Shoulder and Elbow Orthopedic MCQS online
(OBQ15.201) A 52-year-old, right hand dominant man comes for evaluation of right shoulder pain that has been intermittently bothering him for three months. The pain is worse with overhead activities. He denies any history of trauma. His range of forward elevation in the plane of the scapula is painful and is limited to 145 degrees, compared to 170 degrees on his unaffected side. A radiograph is shown in Figure A. He saw another orthopedist a month ago, who ordered an MRI, which showed a small, partial thickness supraspinatus tendon tear. He received a subacromial injection of lidocaine at that time which temporarily relieved 90 percent of the pain he felt with passive forward elevation of his shoulder past 90 degrees. Today he is requesting a subacromial injection of platelet rich plasma (PRP). You tell him that with regard to pain, function and range of motion, subacromial injection of PRP: Review Topic
-
Will result in improvement in pain but no difference in function or range of motion compared to therapy alone.
-
Will result in greater improvement in pain and function compared to therapy alone, with equivalent range of motion at one year
-
Will prevent development of rotator cuff arthropathy in patients with rotator cuff tears.
-
Will likely have no effect on pain, function or range of motion at one year, compared to therapy alone.
-
Will result in worse outcomes in terms of pain, function and range of motion, compared with therapy alone.
PREFERRED RESPONSE 4
At one year, quality of life, pain, disability and shoulder range of motion are the same for patients treated with therapy and placebo versus patients treated with therapy and
PRP injection.
Platelet rich plasma has been used for the treatment of chronic tendinopathy in different areas with mixed results. No benefit to patients with symptoms of subacromial impingement has been demonstrated for subacromial injection of PRP, when added to a standard therapy program.
Kesikburun et al. conducted a randomized controlled trial in which patients with rotator cuff tendinopathy or partial rotator cuff tears were randomized to receive ultrasound-guided subacromial injection of either PRP or lidocaine, followed by a standard six-week therapy program. The authors found no difference in pain, range of motion or validated outcome scores at one year follow up.
Hall et al. reviewed sports medicine applications for PRP. At that time (2009), with regard to PRP, they concluded that there was "little clinical evidence for its use."
Ketola et al. sought to determine the effectiveness of subacromial decompression for the treatment of subacromial impingement syndrome. They randomized 140 patients to a supervised exercise program or arthroscopic subacromial decompression followed by a supervised exercise program. They found no clinically important differences between the two groups at 24 months follow up.
Figure A shows a right shoulder radiograph without osseous pathology. Incorrect answers:
Answers 1 and 2: PRP injection does not result in superior pain or functional outcome compared to therapy alone for treatment of subacromial impingement. Answer 3: There is no evidence that PRP prevents development of rotator cuff arthropathy
Answer 5: No harmful effect of PRP was found when added to a supervised therapy program.
(SAE08UE.109) A 74-year-old man has had worsening left shoulder pain for the past
3 years. Extensive nonsurgical management has provided only minimal relief. Examination reveals limitations in motion due to pain but good rotator cuff strength. Radiographs are shown in Figures 53a and 53b. What surgical procedure is most appropriate? Review Topic
-
Arthroscopic removal of osteophytes and soft-tissue release
-
Soft-tissue interpositional arthroplasty
-
Reverse total shoulder arthroplasty
-
Hemiarthroplasty
-
Total shoulder arthroplasty PREFERRED RESPONSE 5
The patient has end-stage shoulder arthritis with posterior glenoid erosion and large humeral osteophyte formation. Since the rotator cuff is likely intact, the reverse total shoulder arthroplasty is unnecessary. All the remaining procedures may provide symptomatic relief in appropriate patients; however, for most patients, total shoulder arthroplasty has been associated with the most predictive pain relief and functional improvements.
(SAE08UE.94) A 35-year-old man who is an avid weight lifter competing in local tournaments reports new onset pain and loss of motion in his dominant right shoulder. Examination reveals joint line tenderness, active elevation to 100 degrees, and external rotation to 10 degrees. His contralateral shoulder reveals 170 degrees forward elevation and 50 degrees external rotation. Radiographs are shown in Figures 46a and 46b. What is the next most appropriate step in management? Review Topic
-
Total shoulder arthroplasty
-
Hemiarthroplasty with glenoid interposition
-
Surface replacement hemiarthroplasty
-
Arthroscopic debridement
-
Anti-inflammatory drugs and a range-of-motion stretching program
PREFERRED RESPONSE 5
New onset pain and stiffness in the young arthritic shoulder is a difficult problem to treat. Initial management should be aimed at reducing pain and improving motion in all planes. This patient’s activities and age preclude a shoulder arthroplasty at this time. If nonsurgical management fails to provide relief, then arthroscopic debridement and capsular release may be beneficial.
(SAE11UE.122) While performing a total shoulder arthroplasty, excessive retraction is placed on the "strap muscles" (short head of biceps and coracobrachialis). Neurovascular examination would reveal weakness of which of the following? Review Topic
-
Shoulder abduction
-
Shoulder external rotation
-
Shoulder internal rotation
-
Elbow extension
-
Forearm supination
PREFERRED RESPONSE 5
The musculocutaneous nerve can be as close as 3 cm to the coracoid process; therefore, this relationship is important to keep in mind when performing surgery in
this area. Excessive traction on the musculocutaneous nerve could lead to a neurapraxia with resultant weakness of elbow flexion and forearm supinaton because of the loss of biceps function.
(SAE08UE.14) A 72-year-old man who underwent total shoulder arthroplasty 2 years ago slipped on ice and fell on his shoulder 3 weeks ago. Immediately after falling he was unable to elevate his arm. Motor examination reveals deltoid 5-/5, subscapularis 5-/5, external rotation 4-/5, and supraspinatus 2/5. Radiographs are shown in Figures 8a and 8b. What is the most likely diagnosis? Review Topic
-
Anterior shoulder dislocation
-
Humeral component loosening
-
Glenoid component loosening
-
Glenoid component catastrophic fracture
-
Rotator cuff tear
PREFERRED RESPONSE 5
The patient has a traumatic rotator cuff tear. The history of the fall, the weakness on examination, and normal radiographic findings make a traumatic rotator cuff tear the most likely diagnosis. An MRI scan can be obtained to further evaluate the integrity of the rotator cuff. The axillary radiograph shows a reduced, nondislocated total shoulder arthroplasty. His radiographs show a well-seated humeral stem and no signs of loosening. The glenoid is a cemented all-polyethylene component with no evidence of radiolucent lines surrounding the cemented pegs. The polyethylene glenoid component is radiolucent; however, the space between the metallic humeral head and the glenoid bone is the thickness of the polyethylene glenoid component. If the humeral head were directly against the glenoid bone, then catastrophic fracture of the glenoid would be the working diagnosis.
(OBQ14.101) A 31-year-old right handed pitcher felt a pop in his throwing elbow during a game. He is diagnosed with a rupture to the medial ulnar collateral ligament complex of the elbow. During which phase of the overhead throwing cycle did this pitcher most likely sustain his injury? Review Topic
-
Wind-up
-
Early cocking
-
Early acceleration
-
Ball release
-
Follow-through
PREFERRED RESPONSE 3
The medial ulnar collateral ligament is subjected to the greatest tensile stress during the late cocking/early acceleration phase of throwing.
The medial ulnar collateral ligament, or medial collateral ligament of the elbow, is composed of three bundles: an anterior bundle, a posterior bundle, and a variable transverse oblique bundle. During late cocking and early acceleration phases of the overhead throw, the medial UCL is subjected to the greatest amount of valgus stress to the elbow. During this phase, the forearm lags behind the upper arm and generates valgus stress while the elbow is primarily dependent on the anterior band of the UCL for stability. This puts the ligament at greatest risk of injury during this phase.
Fleisig et al. examined the kinetics of baseball pitching and the implications on injury mechanisms. They showed that the UCL contributes to 54% of the varus torque that is generated during the early acceleration of throwing. The position of greatest load occurred when the arm was flexed to 95 +/-14 degrees with an applied valgus load.
Illustration A shows a diagram of the medial ulnar collateral ligament ligament bundles.
Incorrect Answers:
Answer 1: The windup phase is benign for the elbow Answer 2: In early cocking, the rotator cuff and deltoid are active, not the elbow. Answer 4: Ball release is the culmination of cocking and acceleration, but the maximal joint forces have already occurred across the UCL due to the rapid combination of valgus and extension during late cocking and early acceleration. Answer 5: In follow-through, the elbow flexors are most active to prevent hyperextension.
(OBQ15.85) Figure A shows the operative technique used during arthroscopic repair of 25-year-old male patient with vague shoulder pain. The glenoid rim was prepared using the drill bit insertion angles as shown. Three suture anchors, measuring 14mm in length were inserted 4-6 mm deep to the surface. What structure is at the highest risk with this technique? Review Topic
-
Axillary nerve
-
Suprascapular nerve
-
Thoracoacromial artery
-
Thoracodorsal nerve
-
Superior thoracic artery
PREFERRED RESPONSE 2
The suprascapular nerve is at risk of injury during anterior-superior anchor insertion for SLAP repair.
At the scapular spine level, the suprascapular nerve is approximately 1.5-2.0 cm from the glenoid cortex. This places the nerve at risk of injury during shoulder surgery, and injuries have been described. Arthroscopic SLAP repair is known to be a safe and relatively simple procedure. However, deep drilling or anchor insertion from the anterior or anteriosuperior portal during SLAP repair can place the suprascapular nerve at risk of iatrogenic injury.
Morgan et al. performed a cadaveric study to compare the risk of injury to the suprascapular nerve during suture anchor placement in the glenoid when using an anterosuperior portal versus a rotator interval portal. Standard 3 × 14 mm suture anchors were placed in the glenoid rim (1 o’clock, 11 o’clock, and 10 o’clock positions for the right shoulder). They showed that the distance from the far-posterior anchor tip to the suprascapular nerve averaged 8 mm (range, 3.4 to 14 mm) for the anterosuperior portal and 2.1 mm (range, 0 to 5.5 mm) for the rotator interval portal (P = .001).
Koh et al. evaluated the risk of suprascapular nerve injury during the drilling and anchor insertion for anterior SLAP repair. They inserted 1 suture anchor
arthroscopically from the anterior portal at 00:30-1:00 o’clock in right shoulders (11-11:30 in left). Using a mean drill depth was 14.2 (±2.8) mm, all suture anchors perforated the glenoid wall and the tips were measured to be approx. 3.1 (±2.7) mm from the suprascapular nerve.
Figure A shows an arthroscopic view and corresponding schematic image of arthroscopic drill position for SLAP repair. Illustration A shows the close proximity of anchor tip to the suprascapular nerve, as the nerve nerve enters the supraspinatus fossa below the superior transverse scapular ligament and then enters the infraspinatus fossa between the neck of the scapula and base of the spine of the scapula.
Incorrect Answers:
Answers 1,3-5: These nerves are not at risk during this procedure as illustrated.
(SBQ11UE.106) A 60 year-old diabetic man presents with increasing right shoulder pain and stiffness for 10 weeks. He works as a lawyer and has been treating the pain with non-steroidal anti-inflammatory drugs with little improvement. He had a previous injury to his right shoulder 15 years ago while playing hockey, but cannot recall any recent precipitants for this presentation. Physical examination shows significant reduction in right shoulder range of motion, with the greatest loss in external rotation. His MRI images are seen in Figures A-C. What would be the most appropriate treatment? Review Topic
-
Open supraspinatus cuff repair
-
Arthroscopic supraspinatus cuff repair
-
Reverse shoulder arthroplasty
-
Arthroscopic posterior capsular release
-
Physical therapy and medical management
PREFERRED RESPONSE 5
The patient is presenting with right shoulder adhesive capsulitis. The most appropriate treatment at this time would be physical therapy and medical management (i.e., nonsteroidal anti-inflammatories +/- intra-articular steroid injections).
Adhesive capsulitis (aka frozen shoulder) is defined by pain and loss of both active and passive motion, especially loss of external rotation in comparison to the contralateral side. Risk factors include diabetes, thyroid disease, age, previous surgery and prolonged immobilization. Initial treatment options include NSAIDs, physical therapy, and intra-articular steroid injections. Surgical treatment is not commonly recommended, but may be used late in the course to address residual stiffness if extensive therapy has failed.
Griggs et al. prospectively reviewed seventy-five consecutive patients (seventy-seven shoulders) with adhesive capsulitis. They showed that 90 percent of the patients reported a satisfactory outcome with shoulder stretching exercises. However, male gender and diabetes mellitus were associated with worse motion at the final evaluation.
Hannafin et al. reviewed the treatment options for adhesive capsulitis. They suggest
that benign neglect, home-based and supervised physical therapy, and intra-articular corticosteroid injections should be considered in the early treatment of adhesive capsulitis. Duration of treatment should be considered for at least 3 months before more aggressive treatments are considered.
Levine et al. retrospectively reviewed the treatment and outcome of 234 patients with adhesive capsulitis. No significant difference was found for success of nonoperative treatment versus operative treatment or patient gender. With supervised treatment, most patients with adhesive capsulitis experience resolution with nonoperative measures in a relatively short period.
Figures A (Sagittal oblique T1 MRI), and Figure B (coronal oblique fat-saturated T2 MRI) show the anterior rotator cuff interval in the right shoulder demonstrate high-signal soft tissue thickening of the coracohumeral ligament (arrow). Figure C (Coronal oblique fat-saturated T2 MRI) demonstrates a thickened inferior glenohumeral ligament of the right shoulder.
Incorrect Answers:
Answer 1,2: There is no evidence of a rotator cuff tear in the MRI images Answer 3: Reverse shoulder arthroplasty would not be indicated in this patient. There is no evidence of shoulder arthritis. Answer 4: Arthroscopic posterior capsular release would be indicated in persistent adhesive capsulitis with reduction in internal rotation after 3-6 months of nonoperative managements have failed.
(SAE08UE.61) A 28-year-old man sustained a shoulder dislocation 2 years ago. It remained dislocated for 3 weeks and required an open reduction. He now reports constant pain and has only 60 degrees of forward elevation and 10 degrees of external rotation. He desires to return to some sporting activities. An AP radiograph and intraoperative photograph (a view of the humeral head through a deltopectoral approach) are shown in Figures 31a and 31b. What is the best treatment option to decrease pain and improve function? Review Topic
-
Resurfacing hemiarthroplasty
-
Resurfacing hemiarthroplasty with fascial glenoid resurfacing
-
Resurfacing hemiarthroplasty with cemented glenoid component
-
Stemmed hemiarthroplasty
-
Stemmed total shoulder arthroplasty
PREFERRED RESPONSE 4
The radiograph and intraoperative photograph show osteonecrosis with near complete head loss/collapse. A stemmed implant is more appropriate in this patient because there is very little bone to support a resurfacing implant. In a younger patient, a glenoid implant should be delayed as long as possible because of the eventual need for revision secondary to glenoid loosening and wear, especially in a young active male. The hemiarthroplasty may be converted to a total shoulder arthroplasty in the future.
(SAE11UE.60) A 74-year-old man underwent a hemiarthroplasty with acromioplasty for rotator cuff tear arthropathy 2 years ago. Despite continued therapy, he is still unable to elevate his arm beyond 40 degrees. Attempted elevation is painful and demonstrates bulging in the anterosuperior aspect of his shoulder. Radiographs show a well-positioned hemiarthroplasty without signs of loosening. What is the most appropriate treatment for this patient? Review Topic
-
Conversion to a total shoulder arthroplasty
-
Conversion to a reverse shoulder arthroplasty
-
Continued physical therapy
-
Cortisone injection
-
Anti-inflammatory medication
PREFERRED RESPONSE 2
The patient is experiencing anterosuperior escape with attempted shoulder elevation. A conversion to a reverse shoulder arthroplasty will provide the stability to allow active elevation without subluxation. Further physical therapy, cortisone injection, or anti-inflammatory medication will not resolve this instability. A total shoulder arthroplasty is contraindicated because of the anterosuperior escape.
(SAE13BS.52) Reverse total shoulder arthroplasty improves kinematics in the rotator cuff deficient joint by what directional change to the center of rotation? Review Topic
-
Medial
-
Lateral
-
Posterior
-
Proximal
PREFERRED RESPONSE 1
Surgical indications for reverse total shoulder arthroplasty are expanding. In the setting of rotator cuff tear arthroplasty in which the native humeral head migrates superiorly, these implants impart several kinematic advantages. Implant center of rotation medial to the former joint surface improves glenoid component stability as the resultant force vector passes through the component throughout the arc of motion. A stable and fixed fulcrum for elevation is provided by matched radius of curvature between the glenoid and humeral components. A more distal center of rotation increases resting length and tone of the deltoid muscle, improving its effectiveness as a shoulder elevator. Medialized joint center of rotation increases the moment arm of the deltoid, requiring less muscle force to produce a given torque. This results in decreased articular shear stress.
(SAE11OS.52) A 21-year-old minor league pitcher reports decreasing velocity and ability to target his pitches over the last 2 months. He notes that his arm will start to feel heavy in the later innings and notes pain in the posterior aspect of his shoulder in the late cocking phase of his motion. He denies any specific event that initiated his symptoms. Examination reveals symmetric rotator cuff strength and no increased anterior or posterior translation of either shoulder. Supine range of motion of the right shoulder in 90 degrees of abduction reveals external rotation to 100 degrees and internal rotation to 25 degrees. The left shoulder has 95 degrees of external rotation and 60 degrees of internal rotation. He has pain with an O'Brien's maneuver and a negative apprehension sign. What is the next most appropriate step in management? Review Topic
-
Subacromial corticosteroid injection
-
Use of a sling until the pain resolves
-
Posterior capsular stretching
-
Arthroscopic SLAP repair
-
Arthroscopic anterior-inferior capsulolabral plication with posterior capsular release
PREFERRED RESPONSE 3
The patient has glenohumeral internal rotation deficit with posterior capsular tightness; therefore, initial management should be directed at physical therapy and posterior capsular stretching. The total arc of motion (external rotation + internal rotation) should be equal between the shoulders. He has a deficit of 30 degrees in his throwing shoulder. A "sleeper stretch" is a common way for patients to stretch the posterior capsule and involves lying on the involved side with the shoulder abducted 90 degrees, the elbow flexed 90 degrees, and pushing the forearm toward the table. Subacromial injection is not indicated because the pathology of an internal rotation contracture is located within the glenohumeral joint space and not the subacromial space. A sling might be useful for comfort but will not resolve his symptoms. There is no indication for arthroscopy, SLAP repair, or anterior-inferior capsulolabral plication at this time.
(OBQ14.272) A 32-year-old male electrical worker complains of isolated left shoulder pain after a fall from 6 feet. Radiographs of the shoulder are seen in Figures A and B. The radiology technician was unable to obtain a good axillary view due to significant pain and muscle spasm. What would be the next most appropriate step in management? Review Topic
-
Examination under anesthesia
-
MRI shoulder
-
Zanca view radiograph
-
Stryker notch view radiograph
-
Velpeau view radiograph
PREFERRED RESPONSE 5
This patient presents with risk factors of posterior shoulder dislocation. The next most appropriate step in the management of this patient would be to obtain orthogonal shoulder radiographs using a Velpeau view of the right shoulder as seen in Illustration A.
Risk factors for posterior shoulder dislocation include epilepsy, electrocution and high-energy trauma. To make a diagnosis, standard views of the shoulder are required. These include an anteroposterior (AP) view, lateral scapular view and an axillary view. The axillary view is essential for diagnosis, but this requires the arm to be positioned in 20 - 30 degrees of abduction. If pain and muscle spasm restrict arm movement, the next most appropriate view would include a modified axially view,
such as a Velpeau view.
Robinson et al. reviewed posterior shoulder dislocations and fracture-dislocations. They state that apical oblique, Velpeau, or modified axial radiographs are preferable to other alternative axillary views, as they can be obtained with the arm in a sling. When an osseous injury is suspected, a CT scan and three-dimensional reconstruction can be useful in planning operative management.
Millet et al. wrote a JAAOS article on recurrent posterior shoulder instability. They state that 5 radiographic views, or advanced imaging, is essential to evaluate the shoulder. Characteristics to consider include, joint location, humeral head position, glenoid morphology (e.g., retroversion, hypoplasia, posterior glenoid rim), and impaction fracture of the humeral head.
Figure A and B show a normal shoulder radiograph with the shoulder positioned in internal rotation and external rotation. Illustration A shows the correct positioning of a patient to obtain a Velpeau view of the shoulder. Illustration B shows the correct positioning of a patient to obtain a Stryker notch view of the shoulder. This is used to asses for humeral head defects.
Incorrect Answers:
Answer 1: An examination under anesthesia and muscle relaxation in the operating room would be indicated in a locked shoulder dislocation. Answer 2: When a soft-tissue injury is suspected an MRI may be considered. MRI is not first-line imaging in a traumatic shoulder. Answer 3: Zanca view of the shoulder assesses for clavicle fractures or AC joint injuries.
Answer 4: The Stryker notch view of shoulder, seen in Illustration B, requires the hand to be placed on top of the head and the elbow pointing upwards. This image is used to asses for humeral head deformity.
(SAE07SM.49) A 29-year-old quarterback falls onto his dominant shoulder and sustains the injury shown in Figures 14a and 14b. Management should consist of Review Topic
-
an arm sling.
-
nonsteroidal anti-inflammatory drugs and a rapid return to activity.
-
arthroscopic partial claviculectomy.
-
acromioclavicular joint reduction and stabilization.
-
acromionectomy.
PREFERRED RESPONSE 4
Type V acromioclavicular dislocations are characterized by elevation of the clavicle of 100% to 300% and involve extensive soft-tissue stripping. The treatment of choice is surgical reduction of the acromioclavicular joint and some type of stabilization. Treatment of type III injuries is controversial.
(OBQ14.116) A 14-year-old elite basketball player develops acute medial elbow pain after a fall. Physical examination reveals medial elbow tenderness over the submlime tubercle, but full range of motion. The provocative tests seen in Figure A exacerbate his elbow pain. Radiographs of the elbow are normal. What would be the next best step in treatment? Review Topic
-
Supervised elbow stretching program
-
Therapeutic elbow arthroscopy
-
Static elbow external fixation for 3 to 6 weeks, then MR arthrography if pain continues
-
Activity avoidance for 6 weeks
-
Serial inflammatory markers and rheumatology referral PREFERRED RESPONSE 4
Figure A shows a moving valgus stress, which is a provocative test for ulnar collateral ligament (UCL) injury and elbow valgus instability. The initial treatment would be a short period of immobilization, rest and flexor pronator strengthening in this patient population.
Adolescent UCL injuries can be effectively treated with a short period of rest and NSAIDs to control pain. As the acute inflammation resolves, the patient can be started on a supervised therapy program. This should target flexor pronator muscles, as they are important secondary dynamic stabilizers of valgus stress. Once symptoms have improved and the athlete has regained full range of motion and strength, a mediated throwing program may be initiated. Throwing athletes should be educated to avoid provocative activities during this period.
Chen et al. wrote a JAAOS article on shoulder and elbow injuries in the skeletally immature athlete. They state that surgery is reserved for older athletes with persistent valgus instability despite > 6 months of non-surgical management.
Murthi et al. reviewed recurrent elbow instability. They state the anterior bundle of the medial ulnar collateral ligament complex is the primary valgus stabilizer of the elbow. The anterior band is taut for the first 60° of elbow flexion, and the posterior band is taut from 60° to 120° of flexion. The secondary valgus stabilizers of the elbow joint include the radial head, the anterior and posterior aspects of the capsule, and the muscular forces around the joint.
Figure A is showing a moving valgus stress. Illustration A shows provocative tests for valgus instability of the elbow. The image on the left shows a valgus stress test. This assesses the anterior bundle of the medial ulnar collateral ligament complex by
flexing the elbow to 25-30 degrees and applying a valgus load across the elbow. The image on the right shows milking maneuver. This assesses the posterior bundle of the medial ulnar collateral ligament complex by pulling on the patient’s thumb to create valgus stress with the patients’ forearm supinated and elbow flexed beyond 90°.
Incorrect Answers:
Answer 1: This patient has full range of motion of the elbow. Answer 2: Arthroscopy is not indicated in the acute setting, unless there is an indication for operative treatment, such as intra-articular loose body. Answer 3: While a brace would be reasonable, application of a static elbow external fixation would be too invasive for this injury. Advanced imaging may be indicated to better identify the location and severity of injuries with persistent symptoms at 6-12 weeks. These include, MRI, MR Arthrogram, ultrasound, or stress radiographs. Note, plain MRI sensitivity is 57% and specificity is 100%, while MR arthrography is 92% sensitive and 100% specific. Answer 5: Rheumatology referral and inflammatory workup my be indicated in patients with atraumatic elbow pain with systemic involvement.
(SAE08UE.97) A 20-year-old college pitcher reports medial elbow pain after 3 innings of hard throwing. He recalls no injury and reports no pain with light throwing. The examination shown in the clinical photograph in Figure 48 reproduces the elbow pain. What is the most likely diagnosis? Review Topic
-
Flexor-pronator avulsion
-
Ulnar nerve subluxation
-
Medial collateral ligament injury
-
Lateral ulnar collateral ligament rupture
-
Triceps tendon subluxation
PREFERRED RESPONSE 3
The milking test, as seen in the photograph, elicits pain when a tear is present in the medial collateral ligament. Complete rupture is possible but unlikely when there is no history of trauma and the patient is able to throw pain-free for several innings. Subluxation of the ulnar nerve and triceps tendon subluxation present as a painful snapping over the medial aspect of the elbow.
(SAE08OS.101) A 21-year-old basketball player reports increased left shoulder pain with all lifting and overhead activities. He denies any history of dislocations. Axial MRI arthrogram images are seen in Figures 34a and 34b. An expected finding on physical examination of the shoulder would be positive findings for which of the following tests? Review Topic
-
O'Brien's test
-
Speed's test
-
Jerk test
-
Neer impingement test
-
Apprehension test
PREFERRED RESPONSE 3
An MRI arthrogram is a sensitive imaging study used to identify intra-articular shoulder pathology, especially abnormalities of the labrum. Posterior labral tears, although generally less common than anterior tears, can cause significant morbidity, especially in the athlete. Pain, grinding, or gross subluxation often can be elicited with a "jerk" test of the involved shoulder. This test consists of placing an axial load through the humerus, with the shoulder forward flexed to 90 degrees. The shoulder is
then abducted, while maintaining the axial load, and the patient's subjective and objective response is observed. Comparison to the contralateral shoulder is important, especially if painless subluxation is noted, to determine potential evidence of generalized joint laxity.
(SAE11UE.79) What is the most common complication associated with the treatment of the distal biceps ruptures as shown in Figures 79a and 79b? Review Topic
-
Re-rupture
-
Radioulnar synostosis
-
Posterior interosseous nerve injury
-
Lateral antebrachial cutaneous nerve irritation
-
Radial fracture
PREFERRED RESPONSE 4
The patient shown underwent distal biceps repair with a button technique. Among the reports in the literature, the most commonly noted complication associated with this technique is lateral antebrachial cutaneous nerve irritation. Re-rupture, radioulnar synostosis, and posterior interosseous nerve injury can occur, but are not as common as lateral antebrachial cutaneous nerve injury.
(OBQ15.135) An 85-year-old right hand dominant male sustains a fall onto his elbow while trying to get out of the car. His past medical history is significant for diabetes, congestive heart failure, and COPD. He lives in a nursing home, and is able to ambulate at baseline without and assistive devices. What would be the ideal definitive surgical treatment options in this patient? Review Topic
-
ORIF through an olecranon osteotomy with 90-90 plating
-
Total elbow arthroplasty through an anterolateral approach between the brachialis and biceps
-
Acute elbow fusion
-
ORIF through a tricep splitting approach with parallel plating
-
Total elbow arthroplasty through a direct posterior tricep sparing approach
PREFERRED RESPONSE 5
This elderly low-demand patient would most benefit from a total elbow arthroplasty (TEA) done through a direct posterior tricep sparing approach.
TEA is a preferred alternative for ORIF in elderly patients with complex distal humeral fractures that are not amenable to stable fixation. Elderly patients appear to accommodate to objective limitations in function with time, and suffer less complications with TEA than ORIF in select cases.
Sorensen et al evaluated short- to medium term outcome of total elbow arthroplasty (TEA) in complex fractures of the distal humerus by evaluating 24 consecutive cases. Twenty of the patients were followed for almost 2 years, and their MEPS were 15 excellent, 4 good and 1 fair result. They concluded that TEA in complex fractures of the distal humerus in elderly patients can result in acceptable short- to medium term outcome.
Mckee et al conducted a prospective, randomized, controlled trial to compare
functional outcomes, complications, and reoperation rates in elderly patients with displaced intra-articular, distal humeral fractures treated with open reduction-internal fixation (ORIF) or primary total elbow arthroplasty (TEA). They randomized 42 patients and found that TEA for the treatment of comminuted intra-articular distal humeral fractures resulted in more predictable and improved 2-year functional outcomes compared with ORIF, based on the MEPS.
Illustration A shows radiographs of a TEA. Incorrect Answers:
Answer 1: This fracture is not amendable to reliable stable ORIF.
Answer 2: The anterolateral approach to the elbow utilizes the neurovascular plane between brachialis (musculocutaneous nerve) and brachioradialis (radial nerve) proximally and between brachioradialis (radial nerve) and pronator teres (median nerve) distally. The radial nerve, lies between the brachialis and brachioradialis at the level of the elbow. The location of the neurovascular structures does not allow appropriate access to perform a Total Elbow Arthroplasty. Answer 3: This elderly patient with an acute fracture is not an appropriate candidate for elbow fusion. Answer 4: This fracture is not amendable to reliable stable ORIF.
(OBQ15.253) Which of the following patients has the highest risk of developing recurrent instability after an arthroscopic Bankart procedure for anterior shoulder instability? Review Topic
-
30-year old female, recreational soccer player with ligamentous laxity and an x-ray showing a Hill-Sachs lesion and loss of glenoid contour
-
16-year old male, who plays hockey recreationally with no laxity and an x-ray showing a loss of glenoid contour
-
18-year old female, competitive tennis player with no laxity and x-ray findings of a Hill-Sachs lesion and loss of glenoid contour
-
45-year old male, recreational basketball player with ligamentous laxity and x-ray findings of a Hill-Sachs lesion
-
28-year old male, competitive football player with no laxity and no abnormal x-ray findings.
PREFERRED RESPONSE 3
The 18-year old competitive tennis player with no ligamentous laxity and x-ray findings consistent with a Hill-Sachs lesion and loss of glenoid contour has an instability severity index score (ISIS) of 9, which is associated with a >70% chance of recurrent instability after a arthroscopic Bankart procedure.
The surgical management of anterior shoulder instability consists of both arthroscopic and open approaches. The guiding principles for treatment are the restoration of the normal glenoid labrum anatomy and retensioning of the inferior glenohumeral ligament which is achieved via soft-tissue reconstructions (repair of any labral detachment +/- capsular shift) or bony procedures (such as transfer of the coracoid process).
Ahmed et al. reviewed 302 patients who had undergone arthroscopic Bankart repair and capsular shift for the treatment of recurrent anterior glenohumeral instability. The prevalence of patient and injury-related risk factors for recurrence was assessed. The rate of recurrent glenohumeral instability after arthroscopic Bankart repair and capsular shift was 13.2%. The risk of recurrence was independently predicted by the patient’s age at surgery, the severity of glenoid bone loss, and the presence of an engaging Hill-Sachs lesion.
Balg et al. identified risk factors for recurrent instability after arthroscopic Bankart procedure in 131 consecutive patients. Age under 20 years at the time of surgery; involvement in competitive or contact sports or those involving forced overhead activity; shoulder hyperlaxity; a Hill-Sachs lesion present on an AP radiograph of the shoulder in external rotation and/or loss of the sclerotic inferior glenoid contour were all identified as risk factors. These factors were integrated into a 10-point preoperative instability severity index score (ISIS). Patients with a score over 6 points had an unacceptable recurrence risk of 70%.
Illustration A summarizes the components of the ISIS as developed by Balg and Boileau.
Incorrect Answers:
Answer 1 & 2: These patients have an instability severity index score of 5. This is associated with a 10% risk of recurrence. Answer 4: This patients instability severity index score is 4. This is associated with a 10% risk of recurrence. Answer 5: This patients instability severity index score is 3. This is associated with a 5% risk of recurrence.
(SAE07SM.51) A 22-year-old professional baseball catcher has posterior shoulder pain and severe external rotation weakness with the arm in adduction. Radiographs are normal. MRI scans are shown in Figures 15a through 15c. Management should consist of Review Topic
-
aspiration and steroid injection.
-
rest.
-
acromioplasty.
-
arthroscopic repair and decompression.
-
rehabilitation.
PREFERRED RESPONSE 4
The MRI scans reveal a large posterior paralabral cyst associated with a posterior-superior labral tear. The cyst appears as a well-defined, smoothly marginated mass with low signal intensity on T1-weighted MRI scans and with high signal intensity on T2-weighted MRI scans. MRI also reveals changes in the supraspinatus and infraspinatus muscles secondary to denervation, including decreased muscle bulk and fatty infiltration. MRI has the added advantage, compared with other imaging modalities, of detecting intra-articular lesions, such as labral tears, which are frequently associated with ganglion cysts of the shoulder. In this case of a professional baseball player with a space-occupying lesion causing nerve compression with an associated labral tear, the treatment of choice is arthroscopic decompression of the cyst and repair of the tear. Acromioplasty would not address the primary pathology in this patient.
(SAE08UE.3) Baseball pitchers who have internal impingement will most likely demonstrate what changes in range of motion? Review Topic
-
Increase in internal rotation, decrease in external rotation
-
Increase in internal rotation, increase in external rotation
-
Decrease in internal rotation, decrease in external rotation
-
Decrease in internal rotation, increase in external rotation
-
Decrease in forward flexion, increase in external rotation PREFERRED RESPONSE 4
Pitchers tend to have a decrease in internal rotation and an increase in external rotation. The increase in external rotation is felt to be multifactorial. An increase in humeral retroversion occurs from repeated throwing. This results in increased soft-tissue stretching and results in a posterior capsular contracture.
(OBQ14.149) A total shoulder arthroplasty (TSA) would be the most appropriate treatment in which of the following arthritic patients? Review Topic
-
A 75-year-old female with a longstanding history of brachial plexus palsy
-
A 63-year-old male with a 6 month history of shoulder pain and inability to abduct past 30 degrees
-
A 67-year-old female with chronic shoulder pain and evidence of significant proximal migration of the humerus on x-ray
-
A 70-year-old female with severe shoulder pain and radiographic evidence of glenoid erosion to the coracoid process
-
A 72-year-old male who is 9 months status post right TKA for OA with debilitating shoulder pain and an MRI demonstrating an intact rotator cuff
PREFERRED RESPONSE 5
A total shoulder arthroplasty (TSA) is indicated in the 72 year old male with debilitating shoulder pain and an intact rotator cuff on MRI. The other patient scenarios are examples of contraindications for TSA.
A TSA involves replacement of the humeral head with a metal head and resurfacing of the glenoid to a cemented all-polyethylene surface. In order to achieve optimal results, patients must be selected carefully. Patients with an irreparable rotator cuff tear, non-functioning deltoid, inadequate glenoid bone stock and brachial plexopathy
are poor candidates for TSA.
Edwards et al. conducted a multicenter randomized controlled trial to compare TSA versus hemiarthroplasty in patients with primary osteoarthritis of the shoulder. They found that TSA provided better scores for pain, mobility, and activity than hemiarthroplasty at 2 year follow-up.
Boileau et al. followed 45 consecutive patients who underwent reverse TSA (rTSA) for cuff tear arthropathy (CTA), post-traumatic arthritis, and failure of revision arthroplasty. After a mean follow-up of 40 months, they found that the reverse prosthesis improved function and was able to restore active elevation in patients with incongruent cuff-deficient shoulders. They also found that the results were less predictable and complication and revision rates were higher in patients undergoing revision surgery as compared to those patients undergoing rTSA for CTA.
Illustrations A and B show the preoperative and postoperative x-rays of a patient with characteristic OA of the glenohumeral joint that was treated with TSA.
Incorrect Answers:
Answer 1: Brachial plexus palsies lead to substantial motor and sensory deficits. As a result, TSA is contraindicated in these patients. Answer 2: An inability to abduct past 30 degrees is suggestive of a non-functioning deltoid, which is a contraindication for TSA. Answer 3: Significant proximal migration of the humerus is indicative of a rotator cuff tear, which is a contraindication for TSA. Answer 4: Glenoid erosion to the coracoid process suggests that there is inadequate glenoid bone stock, which is a contraindication for TSA.
(SAE11UE.84) A 51-year-old woman with shoulder pain responds transiently to a subacromial injection and physical therapy exercise program. When her symptoms recur, an arthroscopic subacromial decompression is recommended. During the surgery, a partial-thickness articular-sided supraspinatus tear is noted. The supraspinatus footprint is exposed for 3 mm from the articular margin. The remaining intra-articular structures are normal. Inspection from the bursal surface reveals the tendon to be intact. What is the most appropriate course of management? Review Topic
-
Completion of the tear from the bursal surface and rotator cuff repair
-
Arthroscopic long head biceps tenotomy
-
Arthroscopic glenohumeral synovectomy
-
Arthroscopic tendon debridement and subacromial decompression
-
Transtendinous rotator cuff repair
PREFERRED RESPONSE 4
The patient has a partial articular supraspinatus tendon avulsion (PASTA) lesion. Outcome studies suggest that articular-sided tears of this magnitude do well with arthroscopic decompression and debridement alone. Determination of lesion thickness is important in recommending treatment, and may be done with a variety of methods. Tears that involve exposure of less than 5 mm of the rotator cuff footprint likely measure less than half of the tendon thickness. In the absence of other associated pathology, bicipital tenotomy or synovectomy would be unnecessary. Completion of the tear or transtendinous repair would be considered for lesions of greater than 50% thickness.
(SAE10SM.56) Figure 56 shows an arthroscopic view of the long head of the biceps; it has an incompetent biceps sling and is unstable, and an axial glenohumeral MRI scan reveals that it is dislocated medially out of the intertubercular groove. What structure is also most likely injured? Review Topic
-
Middle glenohumeral ligament
3 Infraspinatus
4 Subscapularis
5 Bankart tear
PREFERRED RESPONSE 4
It is important to recognize that rotator cuff tears are a common finding in the setting of a dislocated long head of the biceps tendon (LHB) from the intertubercular groove of the shoulder. If a LHB tendon dislocation is found on examination or radiographic work-up (ultrasound or MRI), it is imperative to rule out associated rotator cuff
pathology, specifically of the subscapularis tendon. Although very rare, injury to the lesser tuberosity should also be ruled out. There are a variety of methods to treat the dislocated biceps (tenotomy versus tenodesis); however, the entire rotator cuff, especially the subscapularis, should be carefully inspected and treated if necessary. The corollary is also true - if you find a tear of the subscapularis tendon insertion, especially the superior half, the LHB should be carefully inspected to ensure that it is not unstable as it exits the shoulder. If the LHB is unstable, this is also addressed surgically with either tenotomy or tenodesis. The middle glenohumeral ligament and Bankart tears are not stabilizers of the biceps.
(SAE11OS.83) Figure 83a shows an axillary radiograph and Figures 83b and 83c show axial MR arthrograms of a 20-year-old collegiate offensive lineman who has shoulder pain while pass-blocking. He sustained a shoulder injury 3 months earlier when he "jammed it." Prior to this injury, he denies any pain or instability in either shoulder. Despite undergoing rehabilitation with a physical therapist and trainer and abstaining from playing for 6 weeks, he is currently unable to play because of his symptoms. Examination reveals full active range of motion, a positive jerk test which reproduces his symptoms, and a grade 2 posterior translation of the humeral head with load and shift testing which also reproduces his symptoms. What is the best management option to allow him to return to his pre-injury function next season? Review Topic
-
Arthroscopic posterior capsulolabral repair
-
Thermal capsulorrhaphy
-
Open anterior capsulorrhaphy
-
Intra-articular injection of corticosteroid
-
Immobilization for 6 weeks in external rotation
PREFERRED RESPONSE 1
Arthroscopic posterior capsulolabral repair is most likely to return him to competitive athletics. The patient has symptomatic posterior instability that is preventing him from performing high-level athletic activities. Posterior subluxation of the humeral head is seen on the axillary radiograph and a posterior labral tear is seen on the axial MR arthrograms. Because nonsurgical management has failed to provide relief, treatment should consist of posterior capsulolabral repair. This can be performed either arthroscopically or open with similar excellent results. An intra-articular injection may help his pain but will not likely allow him to return to his pre-injury functional level. Thermal capsulorrhaphy has limited use in the shoulder because of the high rate of complications reported, and anterior capsulorrhaphy will not correct the posterior instability. Whereas a trial of immobilization in external rotation may have benefitted him with the acute injury, it is unlike to help with this recurrent instability.
(SAE08UE.79) The best candidate for a reverse total shoulder arthroplasty is a patient with rotator cuff tear arthropathy with Review Topic
-
anterior superior escape.
-
rheumatoid arthritis.
-
an acromial stress fracture.
-
a centered head and an external rotation lag sign of 50 degrees.
-
active forward elevation of 130 degrees.
PREFERRED RESPONSE 1
Reverse total shoulder arthroplasty is relatively contraindicated in patients with acromial stress fractures and rheumatoid arthritis. A patient with active forward elevation to 130 degrees is better treated with a hemiarthroplasty because the motion already exceeds the average forward elevation attained in most studies using the reverse prosthesis. A centered case of rotator cuff tear arthropathy is also better treated with a hemiarthroplasty, especially in patients with a large external rotation lag sign because the reverse prosthesis has been shown to decrease active external rotation. However, hemiarthroplasties have not performed well in patients with anterior superior escape and in this group of patients, the reverse prosthesis is best.
(OBQ13.255) A 62-year-old man complains of shoulder pain for 2 years. He has had 1 course of intra-articular sodium hyaluronate and 6 weeks of physical therapy with little relief. Examination reveals diminished arm flexion and abduction secondary to pain. Radiographs of his shoulder are shown in Figures A and B. According to the American Academy of Orthopaedic Surgeons Clinical Practice Guidelines, what is the next best step? Review Topic
-
Humeral head replacement arthroplasty
-
Hemiarthroplasty and ream-and-run glenoid procedure
-
Cuff tear arthropathy (CTA) prosthesis
-
Total shoulder arthroplasty with a metal-backed cemented glenoid component
-
Total shoulder arthroplasty with an all-polyethylene cemented glenoid component
PREFERRED RESPONSE 5
This patient has end-stage glenohumeral osteoarthritis (GH OA). According to the AAOS CPG, total shoulder arthroplasty (TSA) is recommended using an all-polyethylene cemented glenoid component.
TSA is indicated for cases of end-stage GH OA. It is preferred to hemiarthroplasty. It is contraindicated in cases with insufficient glenoid bone stock (glenoid wear to the level of the coracoid), rotator cuff arthropathy or irreparable cuff tears and deltoid dysfunction. It provides good pain relief and has good survival at 10 years (>90%).
Radnay et al. performed a systematic review involving 1952 patients comparing TSA with humeral head replacement (HHR). They found that TSR provided greater pain relief, range of motion, patient satisfaction, and had lower revision rates. They recommend TSA over HHR for GH OA.
Izquierdo et al. described the AAOS Clinical Practice Guidelines (CPG) regarding treatment of GH OA. This is summarized in Illustration A.
Figures A and B show end-stage GH OA with large osteophytes and subchondral sclerosis. There is significant glenoid wear and posterior subluxation (Walch B glenoid deformity). Illustration A is a table summarizing the AAOS CPG on treatment of GH OA. Illustration B shows a CTA humeral component. It is not paired with a glenoid component.
Incorrect Answers:
Answer 1: The AAOS CPG does not recommend humeral head replacement arthroplasty (resurfacing).
Answer 2: Although the AAOS CPG recommends both hemiarthroplasty and TSA as options, TSA is preferred. Answer 3: The AAOS CPG does not recommend use of a CTA humeral component. Answer 4: The AAOS CPG does not recommend the use of metal-backed glenoid components. Metal-backed glenoids have higher rates of revision than all-polyethylene glenoids.
(SAE11UE.24) A 74-year-old woman with rheumatoid arthritis has pain in the shoulder that has failed to respond to nonsurgical management. AP and axillary radiographs are shown in Figures 24a and 24b. Active forward elevation is 120 degrees and external rotation is 30 degrees. At the time of surgery, a 1-cm rotator cuff tear is found, which is repairable. Which of the following treatment options will result in the most predictable pain relief and function? Review Topic
-
Total shoulder arthroplasty and rotator cuff repair
-
Rotator cuff repair
-
Reverse total shoulder arthroplasty
-
Interpositional arthroplasty and rotator cuff repair
-
Hemiarthroplasty and rotator cuff repair
PREFERRED RESPONSE 1
Most studies have shown that total shoulder arthroplasties yield better pain relief and improved forward elevation when compared with hemiarthroplasty in patients with rheumatoid arthritis. Patients with repairable rotator cuff tears should undergo repair at the time of surgery because good results have been shown. Reverse arthroplasties are not indicated with rotator cuff tears that are repairable, and interpositional arthroplasties are not indicated for elderly patients.
(SAE10SM.9) If the quality of the tendon is poor at the lateral attachment of a partial articular side rotator cuff tear (more than 6 mm of footprint exposure or greater than 50% thickness), what should the surgeon do? Review Topic
-
Use an autogenous fascial graft.
-
Use an allograft augmentation.
-
Complete the tear and then repair the tendon.
-
Perform a trans-tendon repair.
-
Biopsy the tissue.
PREFERRED RESPONSE 3
Generally, partial articular side rotator cuff tears are treated by either debridement or repair. The decision to repair depends on the "thickness" of the tear and the retraction of the undersurface of the rotator cuff as well as the quality of the remaining tissue. More than 6 mm of footprint exposure suggests a 50% thicknes tear. If it is poor quality as in this case, the surgeon should complete the tear and repair the tendon as in a small full-thickness tear. Intrasubstance tears with an intact footprint can be treated with trans-tendon repair.
(SAE07SM.1) A 22-year-old college baseball pitcher reports the recent onset of anterior and posterosuperior shoulder pain in his throwing shoulder. Examination shows a 15-degree loss of internal rotation, tenderness over the coracoid, and a positive relocation test. Radiographs are normal, and an MRI scan without contrast shows no definitive lesions. A rehabilitation program is prescribed. Which of the following regimens should be initially employed? Review Topic
-
Stretching the posterior capsule and pectoralis minor tendon
-
Stretching the posterior capsule and strengthening the subscapularis
-
Stretching the posterior capsule and using shoulder plyometrics
-
Stretching the anterior capsule and strengthening all components of the rotator cuff
-
Stretching the anterior capsule and improving pitching mechanics
PREFERRED RESPONSE 1
Throwing athletes, particularly pitchers, have a high incidence of shoulder pain. Recent evidence suggests that posteroinferior capsular tightness and scapular dyskinesis may play a substantial role in the pathologic cascade, culminating in the development of articular surface rotator cuff tears and tearing of the posterosuperior labrum. These patients have posterosuperior shoulder pain primarily. Furthermore, these athletes are susceptible to a muscular fatigue syndrome, the SICK (Scapular malposition, Inferior medial border prominence, Coracoid pain and malposition, and dysKinesis of scapular movement) scapula syndrome. This patient has an internal rotation deficit and tenderness over the coracoid. The internal rotation deficit is addressed by stretching the posterior capsule. The tenderness over the coracoid has been attributed to a contracture of the pectoralis minor tendon secondary to scapular malposition. The initial phase of the rehabilitation regimen is directed at stretching the posterior capsule and pectoralis minor tendon.
(OBQ14.178) Figure A is a glenoid CT 3D reconstruction of a 26-year-old accountant who has recurrent shoulder instability. His first dislocation occurred after a fall while skiing. He has now sustained his third dislocation, which was reduced in the emergency department prior to being sent to your office. What is the most appropriate definitive treatment? Review Topic
-
Immobilization in external rotation for 6 weeks
-
Arthroscopic bony Bankart repair
-
Arthroscopic Remplissage procedure
-
Glenoid augmentation using coracoid transfer
-
Glenoid augmentation using tricortical iliac crest graft
PREFERRED RESPONSE 2
This patient has recurrent shoulder instability with a small bony defect of the anterior glenoid and no previous surgery. The most appropriate definitive management in this
patient would be arthroscopic bony Bankart repair.
Older (>20 years old), recreational athletes with minor glenoid bone loss (<20% of the glenoid surface area) may be treated with soft tissue stabilization procedures using suture anchors. Goals of this procedure include tightening and repairing the torn ligament and labrum to the glenoid. Younger, contact sports athletes with large glenoid defect (>20%) may require bony augmentation type of procedures.
Lynch et al. review the clinical presentation, assessment and treatment algorithm for surgical management of bone loss associated with anterior shoulder instability. While defects larger than 25% of glenoid width should be managed with bony augmentation, they recommend soft-tissue stabilization in smaller defects.
Balg et al. analyzed 131 patients following Bankart procedure and identified following risk factors for failure: age <=20, competitive participation in contact sports, shoulder hyperlaxity, Hill-Sachs on AP radiograph, glenoid bone loss of contour on AP radiograph.
Using human cadaveric shoulders with various anterior glenoid defects sizes, The MOON Shoulder Group compared radiography, MRI and CT to determine the most reliable imaging modality for predicting bone loss. Three-dimensional CT, followed by regular CT were the most reliable and reproducible imaging modalities for predicting glenoid bone loss.
Figure A shows an en face sagittal 3D reconstruction of a glenoid with 10% surface area loss.
Incorrect Answers:
Answer 1: While closed reduction and immobilization are appropriate initial management, after failing conservative management, definitive management for recurrent shoulder dislocation is surgical. Answer 3: Remplissage procedure is indicated in setting of large Hill-Sachs lesions. Answers 4 & 5: Glenoid augmentation procedures using coracoid transfer (Bristow-Latarjet) and tricortical iliac crest graft would be indicated in setting of larger glenoid bone loss.
(SAE11AN.33) The MRI scan shown in Figure 33 reveals the sequelae of an acute traumatic anteroinferior shoulder dislocation. The image reveals the typical separation of what two commonly injured structures? Review Topic
-
Anteroinferior labrum from the bony glenoid
-
Anteroinferior labrum from the cartilaginous surface of the glenoid
-
Biceps tendon from its origin on the supraglenoid tubercle
-
Anterior capsule from the proximal humerus
-
Posteroinferior labrum from the bony glenoid
PREFERRED RESPONSE 1
The MRI scan reveals the sequelae of an anteroinferior dislocation, specifically separation of the anteroinferior labrum from the bony glenoid. The separation does not classically occur only at the cartilage-labral junction, but extends to the bony surface of the medial glenoid neck. Separation of the biceps tendon from its origin on the supraglenoid tubercle (SLAP lesion) or separation of the anterior capsule with the proximal humerus (HAGL lesion) may occur but are not the most common sequelae and are not demonstrated in this MRI image. Anteroinferior shoulder dislocations normally do not affect the posterior labral structures. In their landmark study, Rowe and associates noted that this demonstrated lesion was the most common lesion, present in 85% of their series.
(SAE11UE.30) Figure 30 shows the radiograph of an 82-year-old woman who reports a 1-month history of shoulder pain. She is able to actively elevate her arm to 150 degrees but is experiencing discomfort. Her sleep is disrupted because of the shoulder pain. What is the most appropriate management? Review Topic
-
Total shoulder arthroplasty
-
Hemiarthroplasty
-
Reverse shoulder arthroplasty
-
Arthroscopic shoulder debridement
-
Trial of anti-inflammatory medication or cortisone injection and/or deltoid strengthening
PREFERRED RESPONSE 5
The patient is experiencing rotator cuff tear arthropathy. Given that this is the first medical treatment she has sought, a nonsurgical treatment plan of anti-inflammatory medication or a corticosteroid injection is warranted. Proceeding to the operating room without a trial of nonsurgical management is not indicated in this patient population. Surgical procedures may be necessary in the future if nonsurgical measures fail.
(SAE07SM.80) A 12-year-old Little League pitcher reports lateral elbow pain and “catching.” Examination reveals painful pronation and supination and tenderness over the lateral elbow. Radiographs are shown in Figures 22a and 22b. Initial management should consist of Review Topic
-
rest and repeat examination and radiographs until complete healing occurs.
-
rest and resumption of play when he is asymptomatic and examination shows restoration of painless range of motion.
-
arthroscopic in situ drilling.
-
arthroscopic drilling and internal fixation.
-
arthroscopy with removal of the loose body, followed by lateral column osteotomy.
PREFERRED RESPONSE 2
Osteochondritis of the capitellum is a common problem in young throwing athletes and gymnasts. The mechanism of injury involves lateral compression and axial loading of the capitellum. Repetitive trauma causes ischemia with resultant osteochondral necrosis and sometimes eventual separation. Initial management includes rest for a minimum of 6 weeks; occasionally bracing is used. At long-term follow-up, there is typically an observed radiographic abnormality indicating incomplete healing even in asymptomatic patients. Arthroscopy with in situ drilling is reserved for symptomatic lesions that have an intact articular surface. Lesions with partial separation often require fixation. Lateral column osteotomy is a new investigational procedure designed to relieve lateral compression forces and may be used in salvage cases.
(OBQ12.141) A 47-year-old, healthy, active patient presents with a sub-acute, full-thickness supraspinatus tear. His physical examination reveals significant weakness and pain with abduction. There was no glenohumeral instability. Radiographs demonstrate a type 1 acromion. An MRI scan shows a crescent shaped tear with 2-cm of tendinous retraction and no tendinous fatty changes. A subacromial corticosteroid injection 6 weeks ago provided him with 24 hours of pain relief but no improvement in strength. What would be the most appropriate treatment option? Review Topic
QID:4501
-
Repeat subacromial corticosteriod injection
-
Biological augmentation of rotator cuff with porcine small intestine xenograft
-
Rotator cuff repair
-
Rotator cuff repair plus acromioplasty
-
Rotator cuff repair, remplissage procedure, bicep tenodesis and distal clavicle excision
PREFERRED RESPONSE 3
This patient has an isolated supraspinatus rotator cuff tear with symptomatic weakness. The most appropriate treatment would be isolated rotator cuff repair.
The primary purpose of rotator cuff repair is to restore muscle function. Secondary outcomes include reduction of pain and prevention of irreversible cuff changes, specifically muscular atrophy. Non-operative treatment (exercise, therapy and pain medications) are recommended for partial thickness tears. The indication of surgical repair includes, isolated supraspinatus weakness +/- pain that correlates with MRI imaging of a respective full thickness tear. Routine acrominoplasty is not recommended in conjunction with rotator cuff repair, especially with no previous symptoms of impingement.
Pedowitz et al. developed clinical practice guidelines for the treatment of rotator cuff pathology. The strongest supporting evidence in current literature was given a grade of 'moderate' with four treatment recommendations. These were,
-
Exercise and non-steroidal anti-inflammatory drugs can be used to manage partial thickness tears,
-
Routine acromioplasty is not required the time of cuff repair,
-
Non-cross-linked, porcine small intestine submucosal xenograft patches should not be used to manage cuff tears, and
-
Surgeons can advise patients that workers' compensation status correlates with a less favorable outcome after rotator cuff surgery.
Illustration A shows the different shapes of rotator cuff tears. Incorrect Answers:
Answer 1- Repeat subacromial corticosteriod injection could be considered, but this would not address this patients weakness. Answer 2- There was moderate evidence against the use xenografts for full thickness rotator cuff repairs. Answer 4- There was moderate evidence AGAINST routine acromioplasty in junction with rotator cuff repair, especially with no previous symptoms of impingement. Answer 5- The remplissage procedure, bicep tenodesis and distal clavicle excision would not be warranted in this patient, as there is no supporting features in the clinical exam or imaging to support any additional pathology in the shoulder.
(OBQ15.213) Which patient would expect to have the longest implant survivorship after undergoing total elbow arthroplasty (TEA)? Review Topic
-
Rheumatoid arthritis with unconstrained TEA
-
Rheumatoid arthritis with semiconstrained TEA
-
Primary osteoarthritis with semiconstrained TEA
-
Primary osteoarthritis with unconstrained TEA
-
Acute distal humeral bicolumnar fracture with semiconstrained TEA
PREFERRED RESPONSE 2
Total elbow arthroplasty (TEA) has the longest average implant survivorship in patients with rheumatoid arthritis, above those seen in TEA done for fractures, flail elbow, osteoarthritis, and post-traumatic arthritis. Implant options have traditionally shown best results with semiconstrained TEA designs.
The primary surgical management of the rheumatoid elbow is mainly synovectomy or total elbow arthroplasty (TEA). The indications for TEA include advanced disease of the elbow (grade III or IV), significant pain, and limitations in activities of daily living. Contraindications include active or historical elbow infection, upper extremity paralysis, and refusal to abide by postoperative instructions. Implant options are traditionally classified as linked (semiconstrained) or unlinked - with improved stability and survivorship of semiconstrained designs.
Aldridge et al. provide a 10- to 31-year survival analysis on total elbow arthroplasty with the Coonrad/Coonrad-Morrey prosthesis. They reviewed 40 patients (41 elbows) who had undergone a Coonrad/Coonrad-Morrey elbow arthroplasty. Of the 41 elbows, 21 were functional between 10 and 14 years after operation, ten between 15 and 19 years and ten between 20 and 31 years.
Skytta et al. reviewed 1,457 primary TEAs for rheumatoid elbow. 87% of the TEAs were performed in women. The most frequent reason for revision was aseptic loosening (47%). They found no differences in survival rates between different TEA designs.
Schneeberger et al. examined 41 post-traumatic osteoarthritic elbows treated with semiconstrained Coonrad-Morrey TEA. 11 patients (27 per cent) had major complications. These were attributed to lifting more weight across the implant than adviced by the surgeon; excessive preoperative deformity of the joint; and/or an unstable traumatic injury.
Cil et al. reviewed 91 patients who underwent total elbow arthroplasty for the treatment of a distal humeral nonunion. The rate of prosthetic survival without removal or revision for any reason was 96% at two years, 82% at five years, and 65% at both ten and fifteen years. They conclude that semiconstrained TEA is a salvage procedure in patients with a distal humeral nonunion that is not amenable to internal fixation.
Incorrect Answers:
Answers 1,3-5: Constrained implants have a higher rate of failure than unconstrained or semi-constrained implants. Validated databases have shown the longest survival in patients with RA compared to fractures, flail elbow, osteoarthritis, and post-traumatic arthritis.
(SAE08UE.9) A 78-year-old woman falls onto her nondominant left elbow and sustains the injury shown in Figure A. What treatment option allows her the shortest recovery time and highest likelihood of good function and range of motion? Review Topic
-
Total elbow arthroplasty
-
Open reduction and internal fixation
-
Radial head arthroplasty
-
Sling and swathe
-
Bone stimulator
PREFERRED RESPONSE 1
Total elbow arthroplasty has become the treatment of choice for complex, comminuted distal humeral fractures in patients older than age 70 years. It yields a faster recovery with more predictable functional outcomes, although limitations of lifting weight of more than 5 pounds must be followed to avoid loosening.
(SAE11UE.82) A 61-year-old man reports right shoulder pain and loss of external rotation since having a seizure 5 months ago. MRI scans are shown in Figures 82a and 82b. What is the most appropriate treatment? Review Topic
-
Closed reduction and application of a shoulder immobilizer
-
Open reduction and lesser tuberosity transfer
-
Hemiarthroplasty placed in anatomic version
-
Hemiarthroplasty placed in anteversion
-
Total shoulder arthroplasty
PREFERRED RESPONSE 3
The patient has a chronic posterior shoulder dislocation with loss of approximately half of the humeral head. Hemiarthroplasty or osteochondral allograft to fill the defect would be required. Given the time since injury, the remaining native head and articular surface may have lost structural integrity, making hemiarthroplasty the preferred choice. The implant should be placed close to the patient's natural version, which normally is in the range of 20 to 30 degrees of retroversion. Excessive anteversion is not recommended to avoid repeat posterior dislocation. Closed reduction is highly unlikely to achieve a reduction and may cause displacement of an unrecognized humeral surgical neck fracture. Open reduction and lesser tuberosity transfer is best suited for smaller head defects and a less chronic dislocation. Glenoid integrity is not affected, thus a glenoid implant is unnecessary.
(SAE11UE.52) What is the most common mode of failure following unconstrained total elbow arthroplasty? Review Topic
-
Polyethylene wear
-
Bushing wear
-
Instability
-
Component fracture
-
Loosening of the humeral component
PREFERRED RESPONSE 3
Elbow instability after placement of an unconstrained implant is most often the result of ligamentous insufficiency that can occur late after the index procedure. Instability can also occur from component malpositioning that creates undue stress to the collateral ligaments during the life of the prosthesis. Instability leads to revision surgery in many patients. Polyethylene wear and bushing wear are more common in linked and semiconstrained elbow arthroplasties. Loosening of humeral components may occur with aseptic or septic disease. Component fracture is uncommon.
(SAE11OS.175) When performing a right proximal humeral hemiarthroplasty, the relative placements of the lesser tuberosity relative to the biceps tendon is best depicted, in Figure 175, by the Review Topic
-
lesser tuberosity at A, biceps at B.
-
lesser tuberosity at B, biceps at C.
-
lesser tuberosity at C, biceps at B.
-
lesser tuberosity at A, biceps at C.
-
lesser tuberosity at C, biceps at D.
PREFERRED RESPONSE 1
The lesser tuberosity should be placed at position A, and the biceps tendon at position
B. One of the most common errors during proximal humeral arthroplasty is the use of the lateral keel of the prosthesis as the landmark, around which the tuberosities are reconstructed. If this is done, the anterior soft tissue/bone element is stretched, while the posterior soft tissue/bone element is lax, with a resultant loss of external rotation of the arm. The biceps should be used as the proper landmark for tuberosity reconstruction and in its absence, the anterior aspect of the prosthesis, where the bicipital groove would have been, should be used as the central juncture of tuberosity reconstruction. The upper border of the pectoralis is best used to gauge appropriate height but knowing that the biceps tendon runs directly underneath the tendon insertion can also aid in estimating the proper location.
(SAE07SM.97) What procedure can eliminate a sulcus sign? Review Topic
-
Rotator interval closure
-
SLAP repair
-
Bankart repair
-
Supraspinatus repair
-
Subacromial decompression
PREFERRED RESPONSE 1
A sulcus sign represents inferior subluxation of the shoulder. The elimination of this sign and correction of the inferior subluxation is best achieved through either an open or arthroscopic rotator interval closure. A SLAP repair stabilizes the biceps anchor but does not affect the sulcus sign. A Bankart repair, which corrects anterior-inferior laxity, is not sufficient to eliminate a sulcus sign. Subacromial decompression and supraspinatus repairs have no effect on inferior subluxation.
(SAE08OS.15) A 45-year-old male laborer injured his right elbow trying to catch a heavy object. He has antecubital pain and forearm ecchymosis. MRI scans are shown in Figures 4a and 4b. Nonsurgical management of this injury is most associated with the loss of Review Topic
-
forearm supination strength.
-
forearm pronation strength.
-
elbow flexion strength.
-
elbow extension strength.
-
wrist extension strength.
PREFERRED RESPONSE 1
Complete ruptures of the distal biceps typically occur at the radial tuberosity. Proximal retraction causes visible deformity and is associated with both pain and weakness in the acute setting. Due to the presence of the brachialis, elbow flexion strength returns to near normal. However, forearm supination strength remains weak with nonsurgical management. With surgical management, iatrogenic injury to the posterior interosseous nerve is a concern during exposure of the radial tuberosity. This complication would result in weakness on wrist extension.
(OBQ14.34) A professional baseball team has several pitchers with complaints of velocity loss with their pitches and shoulder pain of their dominant shoulders during spring training. Pitch counts are properly monitored. The average glenohumeral internal rotation deficit on the pitching staff is 45 degrees. The best intervention would be: Review Topic
-
Pitchers throwing less fastballs and more changeups
-
Evaluate the pitchers elbows for ulnar collateral ligament acute ruptures.
-
Increasing the weight training for the deltoid and latissimus dorsi muscles
-
Focused stretches and therapies that address posterior capsular tightness
-
Firing the general manager for finding pitchers that "lose their stuff"
PREFERRED RESPONSE 4
Glenohumeral Internal Rotation Deficit (GIRD) is a phenomenon that occurs in baseball pitchers and is due to posterior capsular tightness. Treatment should begin with a therapy program addressing the pathologic posterior capsule.
GIRD is a phenomenon that is frequently found in high-level overhead throwing athletes, predominantly baseball pitchers. It is defined as the measured difference in internal rotation between the non-dominant arm and dominant arm. Worsening range of motion deficits are seen with increased repetitions, both over a single season and a career. GIRD > 25º is associated with development of shoulder pathologies or pain requiring periods of inactivity. Cessation of overhead throwing activities and initiation of a stretching program to address posterior capsular contractures is largely effective (90% in some series).
Burkhart et al. reviewed the conditions associated with high-level overhead throwing athletes shoulders, culminating in a theory of pathologic progression to "dead arm syndrome" (loss of velocity and effective pitching). Their theory attributes adaptive hyperexternal rotation (occurs during late-cocking / early acceleration phases of pitching) to lead to posterior-inferior capsular contracture and GIRD. Subsequent injuries to anterior structures - including SLAP lesions - would then occur.
Illustration A is a cartoon depiction of how to perform the sleeper stretch. This is a common component of a pitcher's maintenance stretching program.
Incorrect Answers:
Answer 1: While "overthrowing" pitches is associated with a multitude of pitching pathologies, pitch selection is not a described risk factor for GIRD Answer 2: Elbow UCL insufficiency does cause loss of velocity but would not account for the shoulder internal rotation deficit. Answer 3: Heavy weight training about the shoulder is associated with increased stiffness in pitchers Answer 5: While perhaps the "right answer" for professional baseball executives, this is an obvious distractor
(OBQ04.124) A 47-year-old male with a history of a Putti-Platt procedure 20 years ago presents with right shoulder pain with decreased range-of-motion. Radiograph is shown in Figure A. What is the most accurate diagnosis? Review Topic
-
Primary osteoarthritis
-
Post-capsulorrhaphy arthropathy
-
Post-traumatic arthritis
-
Arthritis from poor placement of coracoid transfer
-
Avascular necrosis
PREFERRED RESPONSE 2
With a history of a Putti-Platt procedure with the radiograph, the patient most likely has post-capsulorrhaphy arthropathy.
Post-stabilization procedure arthritis is thought to occur due to changes in contact loading in the shoulder joint due to fixing the joint in an incongruent posistion. It can be severe and debilitating, and lead to arthroplasty as a salvage procedure. The Putti-Platt procedure involves a division of the subscapularis tendon and anterior capsule, and realignment of the lateral tendon stump and capsule sewn into the anterior glenoid neck capsular insertion. The "pants-over-vest" style of repair is then finished by sewing the medial tendon stump into the tuberosity, so that external rotation is significantly limited by the soft tissue imbrication. There is no coracoid transfer for this stabilization procedure.
Bigliani et al. reported on a series of similar patients who developed arthritis following surgery for recurrent glenohumeral dislocation. Authors have theorized that instability repair may excessively tighten the joint in one direction and cause a fixed subluxation in the direction opposite from the side of repair, leading to severe degenerative arthritis due to inappropriate contact loading. 77% of patients following arthroplasty after post-capsulorrhaphy arthropathy had an excellent or satisfactory outcome, with improved pain and range of motion.
Figure A demonstrates severe osteoarthrosis of the affected shoulder, with significant joint space narrowing, periarticular osteophyte formation, and subchondral sclerosis.
Incorrect Answers:
Answers 1, 3, 4, 5: While all of these choices can lead to end-stage arthritis and indications for shoulder arthroplasty, the history given in this question stem indicates Answer 2 as the underlying diagnosis. Of course, the capsulorrhaphy may have been
performed for traumatic dislocation, but the best answer choice for this stem is Answer 2.
(SAE11OS.2) Figures 2a and 2b are the MR arthrograms of a 19-year-old college baseball pitcher who injured his throwing elbow during a game 5 days ago when he felt a pop. Immediately after the throw he reported significant discomfort with pitching and noted that he could not achieve his normal velocity or accuracy in location with his subsequent pitches. On further questioning, he admits to increasing medial elbow pain over the last few seasons with pitching. Examination reveals medial elbow swelling and somewhat diffuse tenderness to palpation medially. Valgus stress at 30 degrees of flexion and resisted wrist flexion produced discomfort. He notes some tingling in his fourth and fifth fingers but Tinel's test posterior to the medial epicondyle is unremarkable. Radiographs of the elbow show no fracture. Because the patient wishes to return to competitive throwing, what is the next step in management? Review Topic
-
Ulnar nerve transposition
-
Ulnar collateral ligament reconstruction
-
Long arm cast for a medial epicondyle fracture
-
Open reduction and internal fixation of the medial epicondyle
-
Elbow arthroscopy and excision of a posteromedial olecranon osteophyte
PREFERRED RESPONSE 2
This high level throwing athlete has a full-thickness injury to the ulnar collateral ligament and is most likely to be able to return to competitive throwing with an ulnar collateral ligament reconstruction. There is no radiographic evidence of a medial epicondyle fracture. The clinical presentation and lack of a posteromedial olecranon osteophyte makes valgus extension overload unlikely, and therefore, makes arthroscopic osteophyte excision a suboptimal choice. Whereas ulnar nerve pathology can coexist with an ulnar collateral ligament injury, isolated ulnar nerve transposition without addressing the ligament injury is not warranted in this patient. Initial nonsurgical management with activity modification and physical therapy is appropriate for partial-thickness injury to the ulnar collateral ligament in a nonthrowing athlete, and in athletes whose sporting activity places them at low risk.
(SAE08UE.106) What neurovascular structure is in closest proximity to the probe in the arthroscopic view of the elbow shown in Figure 50? Review Topic
-
Ulnar nerve
-
Radial nerve
-
Posterior interosseous nerve
-
Superficial radial nerve
-
Median nerve
PREFERRED RESPONSE 2
The image shows a view of the radiocapitellar joint from an anterior medial portal. The radial nerve lies on the elbow capsule at the midportion of the capitellum. It is at risk for injury when capsular excision is performed in this region.
(SAE11UE.16) A 20-year-old man has activity-related deep-seated shoulder pain in his dominant right shoulder. He has taken 3 months off training as a college javelin thrower, and management consisting of physical therapy has failed to provide relief. Shoulder arthroscopic views are shown in Figures 16a through 16c. What is the underlying association with this condition? Review Topic
-
Ehlers-Danlos syndrome
-
Traumatic anterior instability
-
Humeral head osteonecrosis
-
Internal impingement
-
Partial-thickness supraspinatus tear
PREFERRED RESPONSE 4
The patient is involved in overhead athletics and reports deep-seated pain. The arthroscopic views show a SLAP tear with posterior extension that is typical of internal impingement. The history lacks a component of gross instability expected in traumatic anterior dislocations or multidirectional instability associated with a connective tissue disorder, and it also lacks risk factors for osteonecrosis. The images do not show evidence of an unstable humeral cartilage flap or a supraspinatus tear.
(SAE10SM.62) During the cocking and acceleration phases of the overhand throw (pitch), there are several static and dynamic restraints to provide medial elbow support and prevent valgus instability. The dynamic structures found to be most important during these phases of the overhand throw are the flexor digitorum Review Topic
-
profundus and extensor carpi radialis longus.
-
profundus and extensor carpi radialis brevis.
-
superficialis and extensor carpi radialis longus.
-
superficialis and flexor carpi ulnaris.
-
superficialis and flexor carpi radialis.
PREFERRED RESPONSE 4
Biomechanical analysis has demonstrated that local dynamic stability of the elbow is provided by the flexor digitorum superficialis and the flexor carpi ulnaris, especially during the cocking and acceleration phases of the overhand throw. This provides dynamic joint compression across the elbow joint and may be protective to the static restraints such as the ulnar collateral ligament. It also emphasizes the need to strengthen distant muscles in the forearm to assist with elbow biomechanics and potentially prevent injury.
(SAE08UE.57) A healthy 64-year-old man just underwent an uncomplicated shoulder arthroplasty for severe glenohumeral osteoarthritis. Intraoperatively, 60 degrees of external rotation was obtained. Postoperatively, he starts on a range-of-motion program. What limitations are recommended? Review Topic
-
No external rotation stretching for the first 6 weeks.
-
No external rotation stretching for the first 3 weeks.
-
Limit external rotation to the side to 60 degrees for the first 6 weeks.
-
Limit external rotation to the side to 60 degrees for the first 3 weeks.
-
No restrictions on external rotation stretching.
PREFERRED RESPONSE 3
The patient needs restrictions on his external rotation to allow healing of the subscapularis tendon repair. Limitation to 60 degrees is common if the tendon repair is robust and shows no evidence of tension on range-of-motion testing during the
surgery. Restriction from external rotation stretching for even 3 weeks would compromise his ultimate functional recovery.
(SAE10SM.94) If the structure marked by the tip of the probe in Figure 94 is repaired to the bony glenoid with suture anchors during an arthroscopic stabilization procedure, what is the most likely result? Review Topic
-
Loss of external rotation with the glenohumeral joint abducted 90 degrees
-
Loss of external rotation with the arm at the side of the body
-
Loss of internal rotation with the glenohumeral joint abducted 90 degrees
-
Loss of internal rotation up the back
-
Loss of flexion
PREFERRED RESPONSE 2
The probe is on the middle glenohumeral ligament (MGHL), which, in this case, is a cord-like and robust structure, commonly known as a Buford complex. The space between the bony glenoid and the MGHL (in this case, a cord-like Buford complex) is a normal variant and should not be repaired or tightened to the bony glenoid with a soft-tissue anchor or other repair. If this structure is inadvertently repaired, the most common scenario is loss of external rotation with the arm at the side, as the MGHL/Buford complex becomes tight with the arm in this position. The loss of external rotation is more pronounced with the arm at the side than abducted at 90 degrees as the MGHL/Buford complex becomes tighter with the arm at the side than abducted.
(SAE11UE.56) What preoperative patient factor has been shown to most closely correlate with poor results after a latissiumus dorsi transfer for an irreparable rotator cuff tear? Review Topic
-
Age of younger than 70 years
-
Positive lift-off test
-
Previous shoulder surgery
-
Loss of passive external rotation
-
Male gender
PREFERRED RESPONSE 2
Patients with a positive lift-off test have a tear of the subscapularis tendon. Patients with a subscapularis tendon tear did much worse than other patients in the studies by Gerber and associates and Irlenbusch and associates. Latissimus dorsi muscle transfer during the primary surgery when a complete rotator cuff repair could not be performed results in a better outcome than a muscle transfer done as a second surgery, but other prior surgery was not shown to affect transfer results. Iannotti and associates found poor results in patients who were female or had external rotation and forward flexion weakness.
(SAE08UE.93) A 32-year-old patient reports progressively increasing pain and stiffness after undergoing arthroscopic shoulder stabilization 1 year ago. The stabilization procedure was a Bankart repair with anchor fixation and supplemented with the heat probe. Radiographs are shown in Figures 45a and 45b. What is the most likely diagnosis? Review Topic
-
Subscapularis failure
-
Frozen shoulder
-
Recurrent instability
-
Loose body
-
Chondrolysis
PREFERRED RESPONSE 5
Postshoulder stabilization chondrolysis is a rare but devastating complication. It has been implicated with the use of the radiofrequency heat probe in some patients.
(SAE10SM.35) A 36-year-old softball player sustains a shoulder dislocation making a diving catch. The shoulder is successfully reduced in the emergency department. A postreduction MRI is shown in Figure 35. What anatomic lesion is a result of the dislocation? Review Topic
-
Bankart lesion
-
Humeral avulsion of the glenohumeral ligament (HAGL) lesion
-
Superior labrum anterior-posterior (SLAP) lesion
-
Hill-Sach deformity
-
Glenoid fracture (bony Bankart)
PREFERRED RESPONSE 2
The MRI scan reveals a HAGL lesion. It more commonly affects older patients and is associated with more violent trauma.
(SAE11OS.17) A 20-year-old collegiate pitcher has had a 5-month history of shoulder pain while throwing, decreased velocity, and difficulty with location of his pitches despite multiple attempts at rest. He reports no traumatic event. Examination with his throwing arm abducted at 90 degrees reveals external rotation to 110 degrees and internal rotation to 70 degrees when compared with his nonthrowing shoulder which has external rotation to 95 degrees and internal rotation to 85 degrees. He has a positive O'Brien's sign, positive modified Jobe's relocation test, full rotator cuff strength, no obvious muscular atrophy, and no scapular winging. Radiographs of the affected shoulder show no abnormalities. What is the next most appropriate step in management? Review Topic
-
Dynamic ultrasound examination of the rotator cuff
-
Electrodiagnostic testing of the throwing shoulder
-
MR arthrogram of the throwing shoulder
-
Referral to a physical therapist to concentrate on range of motion
-
Laboratory studies to evaluate C-reactive protein and erythrocyte sedimentation rate
PREFERRED RESPONSE 3
The study of choice to evaluate the superior labrum is an MR arthrogram. The patient has symptoms suspicious for superior labral pathology (ie, positive O'Brien's test, Jobe's relocation test, pain with throwing, loss of velocity and location). Whereas he does have increased external rotation and decreased internal rotation of his throwing arm compared with his non-throwing arm, the total arc of motion is 180 degrees and this is considered a normal adaptive change in the overhead throwing athlete; therefore, ultrasound is not considered appropriate management. There are no signs of weakness or rotator cuff pathology to suggest suprascapular nerve compression or a full-thickness rotator cuff tear; therefore, electrodiagnostic testing or physical therapy are inappropriate. There are also no signs or symptoms suggesting infection or rheumatologic issues; therefore, laboratory studies are unnecessary. If the MR arthrogram shows a labral tear, the initial management would include posterior capsular stretching and rotator cuff strengthening.
(SAE11OS.73) A 21-year-old throwing athlete has persistent shoulder pain. Figures 73a and 73b are arthroscopic photographs taken from a posterior viewing portal and an anterior viewing portal. During which phase of the throwing motion did the injury most likely occur? Review Topic
-
Wind-up
-
Early cocking
-
Late cocking
-
Acceleration
-
Deceleration
PREFERRED RESPONSE 3
Five distinct phases of the throwing motion have been identified, each of which places the static and dynamic stabilizers of the shoulder under different stresses. In the late cocking phase, the throwing arm is abducted and maximally externally rotated.
Rotator cuff tears in throwing athletes may be the result of either tensile or compressive forces. Tensile failure is believed to be the result of repetitive eccentric contractions. Compressive failure is thought to result from direct contact of the articular side of the rotator cuff between the greater tuberosity and posterior glenoid. Compressive failure results in tearing of the posterior supraspinatus and anterior infraspinatus, in contrast to the more common partial tearing of the anterior supraspinatus seen in the general population. In addition to tearing of the articular side of the rotator cuff, compressive forces also contribute to the peel-back mechanism and resultant avulsion of the posterosuperior labrum and biceps anchor. Articular-sided posterior supraspinatus and infraspinatus tears in combination with posterosuperior labral and biceps anchor detachment has been termed internal impingement. It is believed to be the primary result of either posterior capsular contracture (GIRD) or anterior capsular laxity.
(SAE11UE.102) An active 66-year-old man who underwent total shoulder arthroplasty 3 years ago now reports pain. Laboratory studies reveal an elevated erythrocyte sedimentation rate and C-reactive protein. Intraoperative frozen section reveals greater than 10 white blood cells per high power field on two slides and the Gram stain reveals gram-positive cocci in clusters. What is the most appropriate surgical treatment to eradicate the infection and maintain function? Review Topic
-
Removal of the components and placement of an antibiotic spacer
-
Removal of the components, placement of an antibiotic spacer, and bone grafting of the glenoid defect
-
Resection arthroplasty
-
Exchange of the humeral head and debridement
-
Arthroscopic debridement
PREFERRED RESPONSE 1
The prosthesis is grossly infected. Removal of the components and placement of an antibiotic spacer is necessary to eradicate the infection and allow for a second stage reimplantation. Resection arthroplasty is an option to treat the infection but the functional outcome would be limited. Bone grafting with concurrent infection is not likely to heal and should be delayed until the second stage. Humeral head exchange and debridement or arthroscopic debridement alone is unlikely to eradicate the infection.
(SAE10SM.47) Early failure of arthroscopic rotator cuff repair most commonly occurs by which of the following mechanisms? Review Topic
-
Anchor pull-out
-
Anchor fracture
-
Suture rupture
-
Knot failure
-
Tissue failure
PREFERRED RESPONSE 5
Arthroscopic repair of the rotator cuff is becoming increasingly popular. Unfortunately, recent objective evaluations have indicated high failure rates even in patients who are clinically improved. Early failure can occur by failure of the suture anchor, suture, or knot. However, the most common cause of failure is when the suture pulls through the tendon. This results in "stretching" of the repair that can lead to gap formation between the repaired tendon and the osseous insertion, and subsequently, poor tendon-to-bone healing.
(SAE11UE.100) A 27-year-old woman underwent shoulder arthroscopy for multidirectional instability 3 years ago. She was unable to regain shoulder range of motion despite therapy and has had progressively worsening pain. A current axillary radiograph is shown in Figure 100. In reviewing the medical records from the index procedure, what factor may be significant in contributing to her current condition? Review Topic
-
Subsequent development of a supraspinatus tear
-
Subscapularis tendon dehiscence
-
Coagulation of the anterior humeral circumflex artery
-
Use of monopolar radiofrequency thermal capsulorrhaphy
-
Lack of compliance with postoperative therapy program
PREFERRED RESPONSE 4
Reports from several centers suggest the potential to develop glenohumeral chondrolysis because of the heat production associated with use of radiofrequency or laser thermal capsulorrhaphy. A tear of the supraspinatus may lead to poor function and progression to rotator cuff tear arthropathy with superior humeral head migration. Subscapularis dehiscence is a risk in open surgery through a deltopectoral approach and can lead to anterior instability. The anterior humeral circumflex artery is the main supply to the humeral head and its coagulation can lead to osteonecrosis. Whereas a lack of postoperative therapy can lead to unresolved pain and stiffness, chondrolysis is not reported.
(SAE11UE.58) Scapular notching following reverse shoulder arthroplasty may be minimized by what technical modification? Review Topic
-
Horizontal humeral cut
-
Superior inclination of the baseplate
-
Inferior inclination of the baseplate
-
Use of a 36-mm glenosphere
-
Use of a retentive polyethylene liner
PREFERRED RESPONSE 3
Biomechanical studies have shown that a 10-degree inferior inclination may decrease scapular notching; whereas superior inclination may worsen notching. Scapular notching has been recognized as a complication following reverse shoulder arthroplasty. Mechanical abutment of the humeral component possibly leads to erosion of the anteroinferior scapular neck, with progressive vulnerability of the inferior baseplate screws. A horizontal humeral cut does not affect notching because the humeral component causes the notching, not the bone on the humerus. Glenosphere size has not been shown to correlate with scapular notching.
(SAE08AN.68) In recurrent posterior shoulder instability, what is the recommended approach to the posterior capsule? Review Topic
-
A teres minor-splitting approach
-
An infraspinatus-splitting approach
-
Between the infraspinatus and teres minor
-
Between the supraspinatus and infraspinatus
-
In the rotator interval
PREFERRED RESPONSE 2
Using an infraspinatus-splitting incision allows for excellent exposure of the posterior capsule and minimizes the risk of injury to the axillary nerve which lies inferior to the teres minor in the quadrilateral space.
(SAE11UE.34) A 47-year-old man undergoes a posterior cervical procedure for a benign tumor. Postoperatively, severe dysfunction with decreased forward elevation and abduction develops and he has lateral winging of the scapula. What is the recommended treatment to best restore motion and function? Review Topic
-
Rhomboids and levator transfer
-
Split pectoralis major transfer
-
Long head of triceps transfer
-
Scapulothoracic fusion
-
Infraspinatus transfer
PREFERRED RESPONSE 1
The patient has sustained a permanent injury to the spinal accessory nerve and has resultant scapular winging (lateral winging) because of trapezius palsy with weakness in abduction and forward elevation. The modified Eden-Lange procedure (transfer of the rhomboid minor, major, and levator scapulae) has been shown to reliably restore range of motion and function. Split pectoralis major transfer is performed to restore serratus anterior function. The long head of the triceps and infraspinatus tendon transfers are rarely used for any shoulder muscle transfer. A scapulothoracic fusion can also be performed for this problem, but the results are not as effective as the Eden-Lange procedure.
(SAE08UE.64) A 17-year-old girl has multidirectional instability of the shoulder. What is the most appropriate initial management? Review Topic
-
Immobilization in a sling and swathe
-
Open capsular shift
-
Arthroscopic capsular plication
-
Thermal capsulorrhaphy
-
Physical therapy and home exercises
PREFERRED RESPONSE 5
Multidirectional instability of the shoulder is defined as symptomatic instability in two or more directions (anterior, posterior) but must include a component of inferior instability. Initial treatment should always include physical therapy and instruction in a home exercise program that emphasizes periscapular and rotator cuff strengthening to improve the dynamic stability of the glenohumeral joint. Immobilization has not been shown to be effective. Open capsular shift and arthroscopic capsular plication remain the surgical options when appropriate nonsurgical management fails (typically a minimum of 6 months of dedicated therapy and home program). Thermal capsulorrhaphy remains controversial but is not recommended by many clinicians because of reported complications including recurrent instability, axillary nerve injury, chondrolysis, and capsular injury.
(SAE08UE.55) A 23-year-old professional baseball pitcher reports shoulder pain and decreased velocity while pitching. Physical examination reveals a side-to-side internal rotation deficit of 25 degrees. The O’Brien sign is negative; Neer and Hawkins signs are negative. Rotator cuff strength is full. Radiographs are unremarkable. What is the next step in management? Review Topic
-
MRI-arthrogram to evaluate the rotator cuff
-
Rotator cuff strengthening program
-
Posterior capsular stretching program
-
Shoulder arthroscopy with SLAP repair
-
Shoulder arthroscopy with posterior capsular release
PREFERRED RESPONSE 3
Throwing athletes with symptomatic internal rotation deficits often benefit from an intensive posterior capsular stretching program. Patients that fail to respond to nonsurgical management may benefit from an arthroscopic posterior capsular release.
(SAE11OS.200) A 20-year-old collegiate volleyball player has vague left, nondominant elbow pain. Five years ago, he sustained a dislocation of the same joint and, while he could participate in his sport, he notes that the elbow 'never felt quite right.` The pain is not severe but prevents him from playing sports and he cannot localize the pain to any specific location. Occasionally he will perceive a catching when pushing himself out of a chair but the elbow never locks in one position. Examination reveals full passive and active range of motion in flexion, extension, supination, and pronation. There is tenderness of the lateral elbow during elbow extension with the forearm supinated and a momentary painful `clunk` is noted. Radiographs and MRI scans are normal. What is the most likely instability? Review Topic
-
Varus
-
Valgus
-
Longitudinal forearm
-
Posteromedial rotatory
-
Posterolateral rotatory
PREFERRED RESPONSE 5
Posterolateral rotatory instability of the elbow is seen in athletes and frequently follows a previous injury such as a dislocation where the lateral ulnar collateral ligament becomes weakened and attenuated. The ulna supinates away from the humerus and the radius subluxates posteriorly on the capitellum with the forearm supinated and the elbow in extension. Posteromedial rotatory instability is more often seen in association with fracture of the coronoid process following a varus stress to the elbow. Valgus instability occurs due to an injury to the medial ulnar collateral ligament seen most commonly in throwers from overuse. Varus instability is rare but results in lateral gapping of the elbow. Longitudinal forearm instability is seen after an Essex-Lopresti injury.
(OBQ15.142) A 34-year-old man presents to clinic with 4 months of right elbow pain. He began going to the gym and playing squash about 3 months ago. On exam, he is tender over the lateral aspect of the elbow and has pain with resisted wrist extension. Which of the following choices lists the correct compartment of the muscle typically involved in this disease and then lists its antagonist muscle? Review Topic
-
posterior; flexor carpi radialis
-
posterior; flexor carpi ulnaris
-
posterior; flexor digitorum superficialis to the long finger
-
mobile wad; flexor carpi radialis
-
mobile wad; flexor carpi ulnaris
PREFERRED RESPONSE 5
The patient presents with lateral epicondylitis, which typically involves the origin of the extensor carpi radialis brevis (ECRB). ECRB is in the mobile wad compartment and its antagonist muscle is flexor carpi ulnaris.
Lateral epicondylitis is an overuse injury, typically secondary to repetitive pronation and supination motion in extension, that leads to inflammation of the ECRB origin at the elbow. Histological analysis typically shows vascular hyperplasia and disorganized collagen. Clinically, patients will have pain over the lateral elbow exacerbated by resisted wrist extension. ECRB, the most commonly involved muscle origin, is innervated by the deep branch of the radial nerve and inserts on the base of the 3rd metacarpal. As it is radial wrist extensor, its antagonist is the ulnar sided wrist flexor.
Brummel et al. reviewed the clinical presentation and management options for lateral epicondylitis. They report acute symptoms in younger patients and chronic symptoms in older patients. NSAIDs, extensor stretching and activity modification are the mainstay of nonsurgical treatment.
Bunata et al. studies 85 cadavar elbows to determine anatomic factors contributing to tennis elbow. They found that the ECRB undersurface rubs against the lateral capitellium in elbow extension leading to tendinosis.
Illustration A is cross-sectional diagram of the forearm with muscle bellies labeled. Notice the location of ECRB in the mobile wad. Illustration B is a coronal T2 MRI showing fluid signal and undersurface tearing near the extensor origin as can be seen in lateral epicondylitis.
Incorrect Answers:
1-4: The ECRB is in the mobile wad and its antagonist is flexor carpi ulnaris. All other answers are incorrect.
(SAE11UE.71) A 50-year-old man fell from a ladder onto his left shoulder and sustained the injury shown in the radiographs in Figures 71a and 71b. He underwent surgery with repair of the coracoclavicular ligaments and deltotrapezial fascia with coracoclavicular screw placement. Which of the following statements regarding postoperative complications is most accurate? Review Topic
-
Hardware migration is more likely than with acromioclavicular pinning.
-
Failure of fixation is usually at the level of the clavicle.
-
Hardware removal is avoided to prevent late displacement.
-
Neurologic injury most likely involves the axilllary nerve.
-
Acromioclavicular arthritis is more likely than with nonsurgical management.
PREFERRED RESPONSE 5
Whereas pain and functional disturbance may persist with nonsurgical management, the lack of articular surface contact prevents arthritic symptoms from developing. Cartilage injury caused by trauma and any persistent joint incongruity following repair would contribute to posttraumatic arthritis. Pinning across the acromioclavicular joint has a high incidence of hardware migration and potential catastrophic consequences. Most cases of lost fixation of coracoclavicular screws are at the level of the thread purchase in the coracoid. Routine hardware removal at 8 to 12 weeks is recommended to avoid screw breakage because of natural movement between the clavicle and scapula. The axillary nerve passes around the inferior edge of the subscapularis and is anatomically distant to the the coracoid. The musculocutaneous nerve would have the closest anatomic position to the coracoid.
(SAE10SM.4) What portion of the pitching phase creates forces approaching the tensile limit of the medial ulnar collateral ligament of the elbow? Review Topic
-
Early cocking
-
Late cocking / Early acceleration
-
Ball release
-
Follow-through
-
Deceleration
PREFERRED RESPONSE 2
The late cocking and early acceleration phases are often combined when discussing medial stress on the elbow of the overhand thrower. This is when the greatest valgus moment across the medial elbow occurs and the forces reach the tensile limits of the medial ulnar collateral ligament.
Fleisig et al. were among the first to elucidate the elbow and shoulder kinetics in healthy adult pitchers using high-speed motion capture analysis. Inability to generate sufficient elbow varus torque may result in medial tension, lateral compression, or posteromedial impingement injury.
According to Lynch et al. the late cocking phase of the overhand throw places a marked valgus moment across the medial elbow. This repetitive force reaches the tensile limits of the medial collateral ligament, subjecting it to microtraumatic injury and attenuation. The anterior bundle of the medial collateral ligament has been identified as the primary restraint to valgus load and is the focus of reconstruction.
Incorrect Responses:
1,4,5: The medial elbow forces are less during these phases. 4: Ball release is not one of the 5 phases of throwing and marks the end of the acceleration and beginning of deceleration phase.
(SAE08UE.24) A 52-year-old man underwent arthroscopic repair of a 1-cm supraspinatus tendon tear 3 weeks ago. He was doing well until he fell down three stairs. One week after the fall he continues to report pain similar to his preoperative pain. An MRI scan reveals a minimally retracted 1-cm supraspinatus tendon tear in the same location as his original tear. Management should now consist of Review Topic
-
continued physical therapy that focuses on stretching and advances to strengthening in 4 weeks.
-
a cortisone injection into the subacromial space.
-
revision rotator cuff repair.
-
a sling with an abduction pillow for 2 weeks, followed by a stretching program.
-
open rotator cuff debridement without repair.
PREFERRED RESPONSE 3
The patient has retorn his rotator cuff repair. This traumatic retear is different from a chronic tear and should be treated similar to an acute rotator cuff tear. Because the patient is younger than age 65 and has a small, single tendon tear, a revision rotation cuff repair is indicated with an expected tendon healing rate of greater than 95%. A physical therapy program is not indicated, and further delay in repair compromises his functional recovery. A cortisone injection is not indicated for this repairable tendon tear. Immobilization will not allow the tendon to heal once it has retorn. A debridement procedure is not indicated on this repairable tendon tear; this procedure is indicated in painful, chronic, irreparable tendon tears.
(SAE11OS.14) Which of the following postoperative rehabilitation techniques causes minimal rotator cuff muscle activation? Review Topic
-
Active forward flexion
-
Passive forward flexion
-
Active-assisted forward flexion
-
Overhead pulley-assisted passive forward flexion
-
Isometric strengthening
PREFERRED RESPONSE 2
Electromyography (EMG) studies have shown that the rotator cuff is least active with passive range of motion and hence this is allowed early in most postoperative rotator cuff rehabilitation protocols. Active forward flexion, active-assisted motion, and isometric strengthening all cause activation of the rotator cuff muscles (as measured by EMG) and therefore should be introduced later in rehabilitation when the repair can withstand these forces. Whereas some authors have felt that pulley-assisted range of motion exercises are safe, EMG analysis has demonstrated that these exercises do cause activation of the rotator cuff musculature and probably should be avoided early in the rehabilitation protocol.
(SAE07SM.64) A 12-year-old boy who pitches on two “select” baseball teams has had pain in his dominant right shoulder for the past 6 weeks. The pain is present only with throwing and is associated with decreased throwing velocity and control. He has no radiation of pain or paraesthesias of the upper extremity. An AP radiograph and MRI scan are shown in Figures 19a and 19b, respectively. Management should consist of Review Topic
-
rest from throwing activities.
-
a subacromial corticosteroid injection.
-
open reduction and internal fixation.
-
arthroscopic labral repair.
-
biopsy of the proximal humerus.
PREFERRED RESPONSE 1
The imaging study demonstrates characteristics of Little Leaguer’s shoulder, including physeal widening. This condition is secondary to overuse (typically throwing) and responds well to rest from the inciting activity. There is no evidence from the patient’s history or examination that he has an impingement syndrome, nor is there any indication of labral pathology on the MRI scan. The changes in the proximal humerus are classic for this condition and are not suggestive of a neoplastic process requiring biopsy for definitive diagnosis.
(SAE11UE.40) Which of the following statements best describes the typical early presentation of osteochondritis dissecans of the elbow? Review Topic
-
Often associated with loss of elbow extension
-
Often associated with catching or locking
-
Involves the capitellum or lateral trochlea
-
Presents in boys younger than age 10 years
-
Outlining of the margins of the lesion on MR arthrogram is a good prognostic sign
PREFERRED RESPONSE 1
This condition is the result of repetitive valgus overload of the radiocapitellar joint in the immature elbow. The clinical presentation is of lateral elbow pain and loss of extension in a juvenile older than age 10 years. Panner's disease typically affects the capitellum in boys younger than age 10 years. Osteochondritis dissecans (OCD) of the elbow affects the capitellum and occasionally the radial head. Fracturing of the OCD region can lead to an unstable fragment with margins outlined on an MR arthrogram and can progress to loose bodies that cause clinical catching or locking. These are typically late signs with a poorer prognosis.
(SAE11UE.41) Two years after undergoing a total shoulder arthroplasty, a patient reports increasing pain, stiffness, and swelling, and has an increased white blood cell count. Radiographs show lucencies around the glenoid and humeral components. You suspect infection. Which of the following is the most likely responsible organism? Review Topic
-
Staphylococcus aureus
-
Staphylococcus epidermidis
-
Propionibacterium acnes
-
Escherichia coli
-
Pseudomonas aeruginosa
PREFERRED RESPONSE 3
The most likely organism to cause late infection in shoulder arthroplasty is Propionibacterium acnes. This is a slow growing organism that is present in over 50% of chronic infections. Staphylococcus epidermidis is the second most likely organism in this setting, present in 15% of cases. The other three organisms are unlikely to present with this clinical picture.
(OBQ13.118) A 30-year-old man undergoes arthroscopic Bankart repair for recurrent anterior dislocation. He continues to experience instability postoperatively. Examination reveals a positive apprehension test. Radiographs of both shoulders are seen in Figure A. CT scan of his left shoulder is seen in Figure B. What is the best treatment option? Review Topic
-
Bankart repair
-
Humeral head bone augmentation
-
Remplissage
-
Coracoid autograft
-
Connolly procedure
PREFERRED RESPONSE 4
This patient has anterior glenoid bone deficiency (inverted pear glenoid) from a large bony Bankart lesion that was not adequately addressed in the index procedure. This is best treated with bony augmentation using the Latarjet vascularized coracoid transfer.
Patients with glenoid bone defects >20-30% have a high recurrence rate (>60%) after Bankart repair alone. Bone grafting is necessary to offer containment. Autograft options include coracoid transfer (such as the Latarjet procedure which extends the articular arc and creates a conjoined tendon sling) and iliac crest bone grafting.
Burkhart et al. addressed glenohumeral bone defects. They advise that significant bone deficits cannot be adequately addressed via arthroscopic Bankart repair alone. The Latarjet transfer creates an extra-articular platform to extend the articular arc of the glenoid.
Hantes et al. assessed Latarjet repairs using CT. They found that there is almost
complete repair of a 25% to 30% glenoid defect when using the Latarjet procedure.
Figure A comprises comparison Bernageau view glenoid profile radiographs of both shoulders. Figure B is a 3D reconstruction CT with showing glenoid bone deficiency (inverted pear deformity) with a large bony Bankart lesion. Illustration A shows the method of obtaining a Bernageau glenoid profile view. Illustration B shows the "cliff sign" of anterior glenoid bone loss. Illustration C depicts the Latarjet procedure. Illustration D depicts reduction in the articular arc with anterior glenoid loss.
Incorrect Answers:
Answer 1: Bankart repair is not likely to be successful with a large bony Bankart lesion.
Answer 2: Humeral head bone augmentation (autograft or allograft) is useful for engaging Hill Sachs lesions. Answer 3: Remplissage is a soft tissue filling procedure (posterior capsulodesis and infraspinatus tenodesis) which also addresses engaging Hill Sachs lesions Answer 5: The Connolly procedure involves transfer of the infraspinatus and attached greater tuberosity bone block into a Hill Sachs lesion. It is analogous to the Neer-McLaughlin transfer (subscapularis and lesser tuberosity for reverse Hill Sachs lesions) in posterior instability.
(OBQ12.96) Which of the following statements is true regarding the posterior oblique portion of the medial collateral ligament of the elbow? Review Topic
-
Can be released to gain flexion in patients with post-traumatic contracture
-
Has the highest tensile strength of any elbow ligament
-
Is reconstructed in the Tommy John procedure
-
Is the primary ligamentous restraint to valgus force during throwing
-
Is responsible for the pivot shift of the elbow
PREFERRED RESPONSE 1
Addressing flexion in a post-traumatic contracture of the elbow requires releasing the posterior oblique ligament (or band) of the medial ulnar collateral complex.
The medial ulnar collateral ligament is one of the primary static stabilizers of the elbow and is composed of three parts: anterior, posterior and transverse. The MCL provides resistance to valgus and distractive stresses. The anterior oblique fibers (of the anterior bundle) are the most important against valgus stresses. The posterior bundle is involved elbow contractures and releasing it can yield significant flexion gains, without creating valgus instability.
Morrey et al. performed a pilot study on 4 specimens and found the valgus stability is equally divided among the medial collateral ligament complex, anterior capsule, and bony articulation in full extension; whereas, at 90 degrees of flexion the contribution of the anterior capsule is assumed by the medial collateral ligament which provides approximately 55% of the stabilizing contribution to valgus stress.
Regan et al. was a subsequent study by the same group that found the posterior medial collateral ligament (PMCL) was taut only when the elbow was in a flexed position. Among the collateral ligaments, the anterior (AMCL) was the strongest and stiffest. (Of note, using present terminology these would be referred to as posterior oblique and anterior oblique portions of the medial ulnar collateral complex).
Wada et al. reported a series of open medial releases for post-traumatic elbow contracture and found scarring on the posterior oblique bundle in all cases. Large increases in flexion were achieved by releasing this structure and the capsule without the need for a lateral incision in most cases.
Illustrations A and B show the posterior oblique portion of the medial collateral ligament. Illustration C shows the most recent terminology and identifies the area to be resected for stiffness.
Incorrect Answers:
Answers 2,3,4: All refer to the anterior oblique bundle of the medial ulnar collateral ligament.
Answer 5: Lateral ulnar collateral (LUCL) insufficiency is responsible for posterolateral rotatory instability and a positive pivot shift of the elbow.
(OBQ14.53) Figure A shows immediate post-operative radiographs of a 75-year-old patient with primary osteoarthritis. She presents 3 years later with increasing pain and weakness in the shoulder despite home physical therapy. Examination reveals limited active range of motion, with forward elevation of 80 degrees and external rotation of 50 degrees. Her deltoid function is intact. Repeat radiographs are seen in Figure B.
Which treatment option would provide the best functional outcome for this patient? Review Topic
-
Open tendon transfer
-
Corticosteriod injection and supervised physical therapy
-
Open rotator cuff repair, subacromial decompression and distal clavical excision
-
Revision to reverse shoulder arthroplasty
-
Revision to cuff arthropathy hemiarthroplasty
PREFERRED RESPONSE 4
This patient presents with failed total shoulder arthroplasty. The best treatment option for functional outcome would be revision to reverse shoulder arthroplasty (rTSA).
RTSA is considered a viable treatment option for patients with failed shoulder arthroplasty. It allows for improved arm elevation and abduction in the setting of nonfunctional rotator cuff muscles, as seen in this example. Despite the expanding indications for rTSA, there are high complication rates in the revision setting. Complication rates for rTSA after failed shoulder arthroplasty have been reported to be between 11-36%. This procedure should, therefore, be performed by surgeons with extensive training in reconstructive shoulder arthroplasty.
Patel et al. retrospectively reviewed 31 patients (mean age, 68.7 years) who underwent rTSA for treatment of a failed shoulder arthroplasty. They found the greatest improvement with active forward elevation from 44° preoperatively to 108° postoperatively (P < .001). Complications occurred in 3 patients with periprosthetic fracture.
Hattrup et al. reviewed a series of 19 patients that underwent open rotator cuff repair after shoulder arthroplasty. Out of the 19 patients only 4 shoulders were successfully repaired. They concluded that successful rotator cuff repair after shoulder arthroplasty is possible but failure is more common.
Figure A shows a left total shoulder arthroplasty that is well reduced in the glenoid. Figure B shows antero-superior escape of the prosthesis, indicative of a massive rotator cuff tear.
Incorrect Answers:
Answer 1: Poor functional outcomes have been reported in patients following tendon transfers in setting of previous shoulder replacements. Answer 2: A well-placed corticosteroid injection may improve pain it will not address the function limitations due to a nonfunctional rotator cuff. Answer 3: Poor results have been reported in patients that underwent rotator cuff repair after shoulder arthroplasty. Subacromial decompression and distal clavicle excision would also not be recommended in this patient as this may further destabilize the shoulder.
Answer 5: Revision to a cuff arthropathy hemiarthroplasty may address the irreparable rotator cuff tear. However, best results for hemiarthroplasty have been shown in patients with marked glenoid bone loss, younger patients, or in patients with rheumatoid arthritis who maintain forward elevation of more than 90 degrees.
(OBQ14.101) A 31-year-old right handed pitcher felt a pop in his throwing elbow during a game. He is diagnosed with a rupture to the medial ulnar collateral ligament complex of the elbow. During which phase of the overhead throwing cycle did this pitcher most likely sustain his injury? Review Topic
-
Wind-up
-
Early cocking
-
Early acceleration
-
Ball release
-
Follow-through
PREFERRED RESPONSE 3
The medial ulnar collateral ligament is subjected to the greatest tensile stress during the late cocking/early acceleration phase of throwing.
The medial ulnar collateral ligament, or medial collateral ligament of the elbow, is composed of three bundles: an anterior bundle, a posterior bundle, and a variable transverse oblique bundle. During late cocking and early acceleration phases of the overhead throw, the medial UCL is subjected to the greatest amount of valgus stress to the elbow. During this phase, the forearm lags behind the upper arm and generates valgus stress while the elbow is primarily dependent on the anterior band of the UCL for stability. This puts the ligament at greatest risk of injury during this phase.
Fleisig et al. examined the kinetics of baseball pitching and the implications on injury mechanisms. They showed that the UCL contributes to 54% of the varus torque that is generated during the early acceleration of throwing. The position of greatest load occurred when the arm was flexed to 95 +/-14 degrees with an applied valgus load.
Illustration A shows a diagram of the medial ulnar collateral ligament ligament bundles.
Incorrect Answers:
Answer 1: The windup phase is benign for the elbow Answer 2: In early cocking, the rotator cuff and deltoid are active, not the elbow. Answer 4: Ball release is the culmination of cocking and acceleration, but the maximal joint forces have already occurred across the UCL due to the rapid combination of valgus and extension during late cocking and early acceleration. Answer 5: In follow-through, the elbow flexors are most active to prevent hyperextension.
(OBQ12.267) A 27-year-old right hand dominant construction worker falls off a scaffold onto his outstretched arm. Figure A exhibits the radiograph taken at a local emergency room. Following treatment, he is placed in a sling and follows up at your office two weeks later. He complains of a feeling that his arm is going to 'pop out'. Which specific physical examination finding is likely to be present? Review Topic
-
Hornblower's Test
-
Jobe's Test
-
Apprehension Sign with shoulder abducted and externally rotated
-
Speed's Test
-
Kim's Test
PREFERRED RESPONSE 5
The patient suffered a posterior shoulder dislocation, likely injuring the posterior capsule and/or labrum. Out of all the answer choices, Kim's test assesses posterior structures. Thus, Kim's test is the physical examination finding most likely to be present.
Posterior dislocations occur less frequently than anterior dislocations, and are often missed. Following closed reduction, persistent instability can occur, usually associated with posterior capsular or labral pathology. Posteriorly directed provocative maneuvers, such as the Kim test can be positive.
Robinson et al. performed an epidemiologic analysis on 120 posterior dislocations. Recurrent instability occurred at a rate of 17.7%. Risk factors for recurrent instability included age less than 40-years-old, dislocation during seizure, and a large reverse Hill-sachs (>1.5 cm3).
Kim et al. describe the Kim lesion, a separation between the posteroinferior labrum and the articular cartilage without complete detachment of the labrum, which cause persistent posterior instability.
Figure A depicts a posterior dislocation on xray. Illustration A depicts the Kim test, which is performed by having the patient seated, arm at 90° abduction, followed by flexing the shoulder to 45° forward flexion while simultaneously applying axial load on the elbow and posterior-inferior force on the upper humerus. The test is positive when there is pain. Video 1 depicts the proper way to perform a Kim Test.
Incorrect answers:
Answer 1: Hornblower's test assesses the teres minor. Answer 2: Jobe's test assesses the supraspinatus. Answer 3: Apprehension sign assesses for anterior instability of the shoulder. Answer 4: Speed's test assesses for biceps pathology.
(SAE11OS.80) A 43-year-old woman has a 2-week history of right shoulder pain. She denies any injury to initiate her symptoms but states that she has shoulder pain with range of motion and lifting objects. Examination reveals mild pain with abduction, empty can testing, and with the Neer and Hawkins impingement tests. Her range of motion with the right shoulder reveals passive forward flexion to 90 degrees, abduction to 90 degrees, external rotation at the side to 15 degrees, and internal rotation to her buttock. The uninvolved left shoulder has forward flexion to 160 degrees, abduction to 150 degrees, external rotation at the side to 60 degrees, and internal rotation to T6. Radiographs of the shoulder are normal. What is the next most appropriate step in management? Review Topic
-
Home exercise program
-
Sling at all times until her pain decreases
-
Closed manipulation under anesthesia
-
Arthroscopic rotator cuff repair
-
Arthroscopic anterior and posterior capsular release
PREFERRED RESPONSE 1
The patient has the recent onset of adhesive capsulitis, which is characterized by loss of both active and passive range of motion. A home exercise program is as helpful as organized therapy to improve her range of motion. While a sling might be appropriate for comfort, continuous use might increase her shoulder stiffness. Surgical treatments, such as a manipulation under anesthesia or arthroscopic capsular release, might be necessary if her motion cannot be restored with physical therapy and home exercises. However, the natural history of idiopathic adhesive capsulitis is self limited and does not usually require surgery. An arthroscopic rotator cuff repair is not indicated because she does not have a rotator cuff tear.
(SAE08UE.70) During treatment of rupture of the subscapularis tendon with associated biceps instability, treatment of the biceps tendon should include which of the following? Review Topic
-
Tenosynovectomy
-
Recentering
-
Deepening of the bicipital groove
-
Tenodesis or tenotomy
-
Lysis of sheath adhesion
PREFERRED RESPONSE 4
With subscapularis tendon ruptures that have biceps tendon pathology, treatment with tenodesis or tenotomy has improved clinical results. Subluxation or dislocation of the biceps tendon is common with subscapularis rupture. Dislocation of the biceps can occur either beneath the tendon, within the tendon, or extra-articularly. In all cases,
the restraints to medial translations of the biceps have been disrupted. Attempts at recentering the biceps have not been successful, and clinical results appear to be improved when tenodesis or tenotomy is employed in the treatment of the unstable biceps associated with subscapularis tears.
(SAE08OS.139) What is the primary sign/symptom with unidirectional posterior instability of the shoulder? Review Topic
-
Instability
-
Dead arm symptoms
-
Locking or catching
-
Decreased range of motion
-
Pain
PREFERRED RESPONSE 5
Posterior labral tears and pathology are usually associated with recurrent chronic injury to the shoulder. Situations such as blocking in football load the humeral head posteriorly, and may predispose to posterior labral injury. Unlike anterior and superior labral pathology, symptoms of posterior labral tears are often vague, though pain with activity is most common. Instability is usually associated with anterior labral pathology. Dead arm symptoms are associated with anterior and superior pathology, especially in throwers. Mechanical locking and catching are less common for any labral pathology. Range of motion with posterior labral pathology is usually unaffected.
(SAE11UE.80) A 16-year-old right-hand dominant male pitcher has had increasing pain in his dominant shoulder for the past 6 months without treatment. A coronal T2-weighted MRI scan is shown in Figure 80. What is the most appropriate treatment plan? Review Topic
-
Decreased pitch count for 4 weeks
-
Continued play with close observation
-
Cessation of all throwing for 6 weeks
-
Arthroscopic repair
-
Mini-open repair
PREFERRED RESPONSE 3
The coronal MRI scan shows an undersurface partial-thickness rotator cuff tear. Initial treatment for this injury should include complete cessation of throwing (or other overhead activities dependent on the athlete). Despite the duration of symptoms, he has had no treatment to date; therefore, nonsurgical management should include activity cessation, a rotator cuff and periscapular strengthening program, and then a slow and supervised return to throwing with particular attention to proper pitching mechanics. Decreasing the pitch count or continued play with observation risks progression of the problem. Surgical intervention is not indicated for initial treatment.
(SAE08AN.23) A previously asymptomatic 40-year-old man injures his shoulder in a fall. Examination shows that he is unable to lift the hand away from his back while maximally internally rotated. An axial MRI scan of the shoulder is shown in Figure
14. What is the most likely diagnosis? Review Topic
-
Pectoralis major tendon rupture
-
Supraspinatus rupture
-
Subscapularis rupture
-
Bankart tear
-
Humeral avulsion of the inferior glenohumeral ligament
PREFERRED RESPONSE 3
The MRI scan shows detachment of the subscapularis from its insertion on the lesser tuberosity. The examination finding is consistent with a positive lift-off test, also indicating a tear of the subscapularis.
(OBQ13.101) Figure A is the MR image of the left shoulder of an active 47-year-old painter who has been experiencing shoulder pain for 9 months. In addition to the finding shown in Figure A, MRI examination of the intra-articular portion of the biceps tendon shows fraying greater than 50%. He has not obtained relief from an 8 month course of non-operative management including non-steroidal anti-inflammatory medications, physical therapy and corticosteroid injection. What is the best next step in treatment? Review Topic
-
New course of physical therapy
-
Activity shutdown with 6 weeks sling immobilization
-
Arthroscopic superior labrum anterior to posterior (SLAP) tear repair
-
Arthroscopic debridement and possible biceps tenotomy versus tenodesis
-
Arthroscopic rotator cuff repair and acromioplasty PREFERRED RESPONSE 4
This patient has a Type II SLAP lesion. These should only rarely be repaired in patients older than 40 years of age. If a source of pain refractory to non-operative management, biceps tenotomy or tenodesis should be considered.
SLAP repair for Type II SLAP lesions is a procedure that has enjoyed a high success rate in young patients. These are generally not indicated for repair in patients greater than 40 years of age due to high rate of stiffness post-operatively. A subset of patients continue to do poorly after SLAP repair. Poor range of motion and the development of post-surgical adhesive capsulitis is often an etiology for poor results. Arthrofibrosis recalcitrant to diligent therapy over many months can be treated with arthroscopic capsular release. This is predicated on failure of a dedicated course of physical therapy as part of a non-operative management course lasting greater than six months. As the propensity for stiffness increases with age, consideration should be treated with SLAP tear debridement and biceps tenotomy or tenodesis in patients greater than 40 years old. Tenotomy or tenodesis, however, can be effective at providing pain relief in the presence of proximal biceps tendon pathology.
Katz et al. reviewed 34 patients who presented to their group for management of failed SLAP repair. 50% were Worker's Compensation cases. The mean age at the time of initial SLAP repair was 43 years. They treated these patients conservatively initially followed by revision surgery in 21 cases. All completed a course of physical therapy initially. They concluded that once a patient has failed SLAP repair, there is a high chance of further conservative treatment failing. Although revision surgery improves outcomes, 32% will continue to have a "suboptimal" result.
Holloway et al. reviewed 50 patients who underwent arthroscopic capsular release for adhesive capsulitis, comparing three groups: (1) post-surgical; (2) post-fracture; and
(3) idiopathic adhesive capsulitis. All patients had completed supervised physical therapy and a home exercise program for at least one year. They concluded that arthroscopic capsular release improved range of motion equally for all three groups but patients in the post-surgical group had poorer subjective pain, function and satisfaction scores.
Figure A is an MRI showing a Type II SLAP tear. Illustration A shows the classification of SLAP lesions.
Incorrect Answers:
Answer 1. The patient has failed a course of non-operative management. Answer 2. This is likely to result in marked shoulder stiffness. Answer 3. Patients over 40 years of age are at a high risk of postoperative stiffness if undergoing SLAP repair Answer 5. This patient does not have a rotator cuff tear.
(SAE08OS.35) A 75-year-old man who is right-hand dominant has had a painful right shoulder for the past 6 months, with no improvement with nonsurgical management. Examination reveals an active motion of 60 degrees of forward flexion and abduction, with severe crepitus and pain. Radiographs reveal a high-riding humeral head with severe glenohumeral arthritic changes. What is the most appropriate treatment? Review Topic
-
Humeral head resurfacing
-
Humeral head arthroplasty
-
Reverse shoulder arthroplasty
-
Total shoulder arthroplasty
-
Arthroscopic shoulder debridement
PREFERRED RESPONSE 3
In an older age group, the most predictable outcome is obtained with a reverse shoulder arthroplasty. Treatment with a standard hemiarthroplasty is more unpredictable in that the pain relief is typically good to excellent in 75% of patients, but the function is poor in most patients. A total shoulder arthroplasty is contraindicated as a result of the significant shearing forces that the glenoid would experience as a result of the rotator cuff deficiency. Arthroscopic lavage and debridement is ineffective in such advanced cases.
(SAE11UE.87) A 25-year-old electrician sustained an injury to his dominant arm while bench pressing at the gym. He reports that he felt a tearing sensation while extending his arms. Examination reveals that he has lost the normal contour of the axillary fold which worsens with resisted adduction. Additionally, there is extensive ecchymosis down the arm and weakness to adduction and internal rotation. Radiographs are normal. What is the most appropriate management? Review Topic
-
Arthroscopic subscapularis repair
-
Repair of the long head of the biceps with tenodesis
-
Open repair of the pectoralis major tendon avulsion
-
Ultrasound and physical therapy to reduce swelling and improve strength
-
Brace immobilization for 6 weeks
PREFERRED RESPONSE 3
This description is classic for an acute pectoralis major humeral avulsion. The loss of contour in the axillary fold confirms this diagnosis. Treatment for a pectoralis tendon avulsion should be open surgical repair in this young patient. Therapy may be considered for injuries within the muscle or at the musculotendinous junction. Examination for subscapularis rupture and biceps injuries would not cause a change in the axillary fold. Bracing will not improve long-term strength.
(SAE10SM.53) Biomechanical in vitro studies of double-row anchor fixation of rotator cuff tears show what initial advantage over single-row anchor fixation? Review Topic
-
Increased peak-to-peak elongation
-
Decreased stiffness
-
Higher ultimate tensile load
-
Decreased contact area
-
Increased conditioning elongation
PREFERRED RESPONSE 3
Biomechanical in vitro studies of double-row fixation of rotator cuff tears during cyclic loading and tensile loading to failure have demonstrated that double-row fixation results in a higher ultimate tensile load when compared to single-row fixation. Peak-to-peak elongation, stiffness, and conditioning elongation for double-row fixation were all similar to single-row fixation. These initial findings, however, may or may not lead to improved clinical outcomes.
(SAE11OS.119) Internal impingement of the shoulder and posterosuperior labral pathology in throwers has been most clearly associated with which of the following? Review Topic
-
Posterior capsular contracture
-
Anterior capsular laxity
-
Coracoacromial arch stenosis
-
Rotator cuff disease
-
Bennet's lesion
PREFERRED RESPONSE 1
Posterior capular contracture has been recognized to be the primary pathologic process resulting in internal impingement. Internal impingement of the shoulder describes contact between the posterosuperior glenoid labrum and the undersurface of the rotator cuff at the level of the posterior supraspinatus when the shoulder comes into abduction and external rotation. This contact may be physiologic or pathologic and is frequently seen in overhead throwing athletes, possibly resulting in articular-sided rotator cuff tears, glenoid labral tears, tendinitis of the long head of the biceps, anterior instability, glenohumeral internal rotation deficit, and dysfunction of scapular rhythm. Nonsurgical management is the initial treatment of choice with an emphasis on increasing range of motion and improving scapular mechanics. Anterior capsular laxity may be present with internal impingement but is variable and less directly associated with internal impingement than posterior capsular contracture.
Coracoacromial arch stenosis is associated with subacromial impingement and unrelated to internal impingement. Bennett's lesion refers to exostosis or calcification at the posterior capsule and while potentially associated with overhead throwing athletes who may have internal impingement, a causal link between the two has not been established and therefore posterior capsular contracture is the preferred response.
(SAE10SM.2) The MRI scans and diagnostic ultrasound shown in Figures 2a through 2c show what pathologic condition? Review Topic
-
Articular-sided supraspinatus tendon tear
-
Bursal-sided supraspinatus tear
-
Superior labral tear
-
Humeral avulsion of the anterior glenoid ligament
-
Avulsion of the anterior inferior glenohumeral ligament
PREFERRED RESPONSE 1
The MRI scans and ultrasound show an articular surface partial-thickness rotator cuff tear of the supraspinatus tendon. This condition most commonly involves the supraspinatus tendon and is usually found on the articular surface where the blood supply is less robust. There are multiple intrinsic and extrinsic factors contributing to this condition which include age-related metabolic and vascular changes that lead to degenerative tearing, subacromial impingement, shoulder instability (typically anterior), internal impingement, and repetitive microtrauma. Acute trauma is less often the cause. The physical examination for this condition is often nonspecific and requires supplemental imaging studies for diagnosis.
(SAE11OS.94) A 16-year-old competitive female swimmer has a 1-year history of left shoulder pain. She denies any specific injury to her shoulder. She reports that the pain is worse with swimming but also has pain with daily activities. She also notes similar occasional symptoms in her right shoulder. Examination reveals symmetric range of motion and rotator cuff strength. Examination of the left shoulder reveals 2+ anterior and posterior translation with pain in both directions and a 2-cm sulcus sign. The right shoulder also has 2+ anterior and posterior translation and a 2-cm sulcus sign with no pain. She also has hyperextension of the elbows and the ability to touch the radial border of her thumb to her forearm. What is the next step in management? Review Topic
-
Open inferior capsular shift
-
Arthroscopic thermal capsulorrhaphy
-
Sling at all times until the pain resolves
-
Arthroscopic anterior and posterior capsular plication
-
Physical therapy for rotator cuff and scapulothoracic strengthening
PREFERRED RESPONSE 5
The patient has symptomatic multidirectional instability. A comprehensive program involving physical therapy to restore dynamic stability to her shoulder is indicated as a first-line treatment. Periscapular strengthening focusing on the serratus anterior and rhomboids and rotator cuff strengthening should be emphasized. A sling might be used occasionally for comfort but will not provide long-term relief of her symptoms. Thermal capsulorrhaphy, although widely used in the past for shoulder instability, has been abandoned because of a high complication rate. Surgical interventions, such as capsular plications or open capsular shift procedures, might be indicated if rehabilitation fails to relieve her symptoms.
(SAE08UE.52) A 20-year-old minor league baseball pitcher is diagnosed with a symptomatic torn ulnar collateral ligament (UCL) in his pitching elbow. Nonsurgical management consisting of rest and physical therapy aimed at elbow strengthening has
failed to provide relief. He has concomitant cubital tunnel symptoms that worsen while throwing. What is his best surgical option? Review Topic
-
UCL repair and nighttime elbow extension splinting
-
UCL repair with ulnar nerve decompression in situ
-
Allograft UCL reconstruction with interference screws
-
Autograft UCL reconstruction with ulnar nerve transposition
-
Autograft UCL reconstruction using a docking technique
PREFERRED RESPONSE 4
High-level pitchers with symptomatic UCL tears require reconstruction, with autograft being the best studied graft selection. With concomitant ulnar nerve symptoms, a simultaneous ulnar nerve transposition provides good results. Ligament “repairs” and allograft reconstructions have not shown good long-term results.
(SAE11UE.22) A 47-year-old man who works as a carpenter reports a 12-month history of painful mechanical locking of his dominant elbow in the mid range of movement. He also has progressive pain at terminal extension that has not responded to medication, rest, and intra-articular cortisone injection. Active range of movement is from 35 degrees to 130 degrees, and he has full pronation and supination. The ulnar nerve is stable, and he has no subjective or objective neurologic dysfunction in the hand. Radiographs are shown in Figures 22a and 22b. What is the most appropriate treatment? Review Topic
-
Oral corticosteroid medication and changes in job activities
-
Soft-tissue interposition arthroplasty
-
Arthroscopic capsular release, loose body removal, and osteophyte decompression
-
Radial head arthroplasty
-
Total elbow arthroplasty
PREFERRED RESPONSE 3
The most appropriate treatment is arthroscopic capsular release, loose body removal, and osteophyte decompression. The patient has moderate osteoarthritis of the dominant elbow, with mechanical symptoms suggestive of loose osteochondral body formation. Because the patient has failed to respond to the typical nonsurgical therapeutic options, it is unlikely that further oral medication will be helpful, and job modification may not be practical at this stage. Soft-tissue arthroplasty may be reasonable to consider when less invasive methods, such as arthroscopy, fail. Isolated radial head arthroplasty would not sufficiently address the symptoms. Total elbow arthroplasty is indicated in cases of more advanced disease in older patients with lower physical demands.
(SAE08OS.48) What adaptations occur in the dominant shoulder of throwers compared to their nondominant shoulder? Review Topic
-
Humeral anteversion with a normal total arc of motion
-
Tight posterior capsule with a normal total arc of motion
-
Lengthening of the anterior capsule with a decreased total arc of motion
-
Increased external rotation with a decreased total arc of motion
-
Decreased internal rotation with an increased total arc of motion
PREFERRED RESPONSE 2
Pitchers change rotation during adolescent growth with external rotation of the proximal humerus. The result is increased external rotation and decreased internal rotation, resulting in a normal total arc of motion. External rotation lengthens the arc of acceleration, resulting in increased velocity. The shorter arc of internal rotation, associated with a tight posterior capsule, makes deceleration of the arm more difficult, which may lead to overuse injuries.
(SAE11UE.111) A baseball player reports a dull pain in the posterior aspect of his throwing arm. Examination reveals decreased internal rotation and prominence of the inferomedial corner of the scapula. An MRI scan suggests a partial-thickness tear of the posterior supraspinatus tendon. Successful treatment would most likely include which of the following? Review Topic
-
Anti-inflammatory medication, posterior capsular stretching, and rotator cuff strengthening
-
SLAP repair
-
Debridement of the partial-thickness rotator cuff tear
-
Rotator cuff repair
-
Imbrication of the labrum and anterior capsule
PREFERRED RESPONSE 1
Internal impingement is related to an internal rotation contracture (GIRD-glenohumeral internal rotation deficit) and an increase in external rotation caused by repetitive overhead throwing. Most patients can be successfully treated with rehabilitation that focuses on internal rotation stretches along with anti-inflammatory medication and strengthening as symptoms improve. SLAP repair and rotator cuff debridement may be considered in refractory cases. Rotator cuff repair is not typically
required, and capsulolabral imbrication is more consistent with the surgical treatment for multidirectional instability.
(SAE10SM.93) What is the theoretical advantage of an open subpectoral technique of tenodesis of the long head of the biceps tendon compared to arthroscopic soft-tissue tenodesis techniques? Review Topic
-
Improved cosmesis
-
Simpler to perform in the lateral decubitus position
-
Shorter surgical time
-
Removal of the biceps tendon from the bicipital groove
-
Superior outcomes when compared to soft-tissue tenodesis in level I studies
PREFERRED RESPONSE 4
A subpectoral biceps tenodesis requires an additional incision at the insertion of the pectoralis major tendon on the humerus. This can be difficult to do in the lateral decubitus position and certainly this technique does not decrease surgical time when compared to arthroscopic soft-tissue techniques. There have been no level I studies comparing the two techniques; however, the theoretical advantage of the open biceps subpectoral tenodesis is that the biceps tendon is removed from the bicipital groove, which may eliminate a source of pain in the biceps tendon.
(OBQ13.158) A 28-year-old professional baseball pitcher sustains a complete rupture of his ulnar collateral ligament. He is neurovascularly intact on exam. Which of the following surgical reconstruction techniques has been shown to result in the lowest complication rate and best patient outcome? Review Topic
-
Splitting of flexor-pronator mass, figure-of-8 graft fixation.
-
Splitting of flexor-pronator mass, docking graft fixation.
-
Splitting of flexor-pronator mass, docking graft fixation, ulnar nerve transposition.
-
Detachment of flexor-pronator mass, figure-of-8 graft fixation, ulnar nerve transposition.
-
Detachment of flexor-pronator mass, docking graft fixation, ulnar nerve transposition.
PREFERRED RESPONSE 2
Ulnar collateral ligament (UCL) reconstruction using a flexor-pronator muscle-splitting approach and a docking graft fixation technique are associated with the lowest complication rate and best patient outcomes.
Vitale et al. performed a systematic review of retrospective cohort studies evaluating UCL reconstruction techniques in overhead athletes. They demonstrated that the flexor-pronator muscle-splitting approach was associated with better outcomes than detachment of the flexor-pronator mass, had a lower rate of postoperative ulnar neuropathy, and a lower overal complication rate. They also found fixation of the graft utilizing the docking technique was associated with better outcomes than the figure-of-8 technique. Abandoning the obligatory ulnar nerve transposition was associated with improved patient outcomes (89% vs. 75%) and a lower rate of postoperative ulnar neuropathy (4% vs. 9%).
Rettig et al performed a case series review of 31 overhead throwing athletes with ulnar collateral ligament injuries managed nonoperatively with 3 months rest followed by rehabilitation exercises. They concluded that 42% of athletes were able to return to their previous level of competition at an average of 6 months from diagnosis (earlier than reconstruction). The authors were unable to identify any patient-specific factors (duration of symptoms, age, acuity of onset) that would predict the success of nonoperative treatment.
Illustration A shows the figure-of-8 (Jobe) graft fixation technique. It is performed by passing the tendon graft through two bone tunnels in the medial epicondyle of the humerus and through one tunnel in the ulnar sublime tubercle. The graft is then sutured to itself in a figure-of-8 configuration. Illustration B shows the docking graft fixation technique. The graft is placed in a triangular configuration through a single humeral tunnel. The suture limbs are then brought out through two separate bone holes and tied over a bony bridge on the superior aspect of the medial epicondyle.
Incorrect Answers:
Answer 1: The figure-of-8 technique is not associated with better patient outcomes when compared to the docking technique. Answer 2: Obligatory ulnar nerve transposition during UCL reconstruction is associated with a higher rate postoperative ulnar neuropathy and worse patient outcomes, and therefore should be avoided. Answer 3 and 5: Detachment of the flexor-pronator mass is not associated with better patient outcomes when compared to the muscle-splitting approach.
(SAE08UE.48) A 74-year-old woman with rheumatoid arthritis reports shoulder pain that has failed to respond to nonsurgical management. AP and axillary radiographs are shown in Figures 23a and 23b. Examination reveals active forward elevation to 120 degrees and external rotation to 30 degrees. What treatment option results in the most predictable pain relief and function? Review Topic
-
Hemiarthroplasty
-
Arthroscopic debridement
-
Total shoulder arthroplasty with a cemented all-polyethelene glenoid component
-
Reverse total shoulder arthroplasty
-
Total shoulder arthroplasty with a metal-backed glenoid component
PREFERRED RESPONSE 3
Most studies have shown that total shoulder arthroplasties yield better pain relief and improved forward elevation when compared to hemiarthroplasty in patients with rheumatoid arthritis. Although rotator cuff tears are more common in this patient population, this patient has good forward elevation and no significant superior migration of the humeral head; therefore, a reverse arthroplasty is not indicated. The arthritis is too advanced in this patient to consider arthroscopy, but in less advanced cases it can improve range of motion and decrease pain. Metal-backed glenoid components have shown higher rates of loosening.
(OBQ14.83) Recent randomized controlled trials comparing early passive range of motion to 6 weeks of immobilization after successful arthroscopic rotator cuff repair concluded that, compared to immobilization, early passive range of motion resulted in: Review Topic
-
Higher Constant scores at 12 months
-
Increased rates of re-rupture as determined by ultrasound
-
Equivalent functional outcomes
-
Less pain at 6 months
-
Inceased range of motion at 12 months
PREFERRED RESPONSE 3
A series of high-quality RCTs have demonstrated that early passive range of motion has equivalent functional outcomes when compared to 6 weeks of immobilization after arthroscopic rotator cuff surgery.
Traditionally, most surgeons recommended early post-operative range of motion exercises for their patients in order to prevent adhesions and ultimately stiffness. However, recent evidence has found that there is no difference in the healing rate, range of motion or functional outcome between patients who undergo early versus delayed (i.e. initial 6 weeks of immobilization) passive range of motion exercises after arthroscopic rotator cuff repair.
Kim et al. conducted a randomized controlled trial comparing early passive range of motion vs. immobilization in 106 patients who underwent arthroscopic repair for full-thickness rotator cuff tears. They found that there was no clinically or statistically significant difference between the two groups in pain, healing or function.
Keener et al. also conducted a randomized controlled trial of 124 patients who were undergoing arthroscopic repair of a full-thickness rotator cuff tear and found no difference between early and delayed range of motion in healing and functional outcome.
Cuff & Pupello also compared early vs. delayed range of motion during the postoperative rehabilitation phase in a randomized controlled trial of 68 individuals undergoing arthroscopic rotator cuff repair and found no significant difference in range of motion or healing.
Incorrect Answers:
Answer 1, 2, 4 & 5: A number of high quality randomized trials have shown that there is no difference between early and delayed passive range of motion in healing, range of motion, pain and function.
(OBQ13.219) A 78-year-old male presents to clinic 4 weeks after left total shoulder arthroplasty. He has not been wearing his sling and reports that he developed increased pain after slipping in the shower. He used the arm to catch himself from falling. On examination, he can flex the shoulder to 70 degrees, limited by pain. Active external rotation with arm at the side is 50 degrees and active internal rotation is 5 degrees. Passive external rotation is to 80 degrees. A radiograph of the left shoulder is shown below in Figure A. What other complaint is the patient most likely to have? Review Topic
-
Pain with palpation of the bicipital groove
-
Pain with palpation over the subdeltoid bursa
-
Sensory loss over the lateral shoulder
-
Sensation of shoulder instability with external rotation
-
Sensation of shoulder instability with internal rotation
PREFERRED RESPONSE 4
The clinical presentation is consistent with a tear of the subscapularis, which is a well-described complication after total shoulder arthroplasty. The most likely additional complaint this patient will have is anterior shoulder instability, noticeable with external rotation of the shoulder.
Total shoulder arthroplasty is the preferred treatment for glenohumeral arthritis in patients with intact rotator cuff and good glenoid bone stock. The surgical approach involves detaching the subscapularis and capsule from the anterior humerus and dislocating the humeral head anteriorly. Post operatively, external rotation is limited to protect the subscapularis repair. If there is suspicion of a postoperative subscapularis tear, and ultrasound can be performed to confirm the diagnosis.
Miller et al. reported 7 cases of subscapularis tendon rupture after total shoulder arthroplasty, all of which were subsequently repaired. Decreased functional outcomes were observed in these patients, with lengthening techniques to address internal rotation contractures and prior surgery involving the subscapularis tendon as risk factors for rupture
Westoff et al. performed static and dynamic ultrasounds on 22 patients after total shoulder arthroplasty evaluating for numerous periarticular pathologies. The authors concluded that sonography is a useful tool for evaluation of peri-implant tissues after TSA.
Figure A shows an intact left total shoulder arthroplasty without evidence of fracture, dislocation, or hardware loosening. Illustration A shows the incision for the subscapularis tendon during TSA.
Incorrect Answers:
Answer 1: Pain with palpation of the bicipital groove would be expected in biceps tendinopathy
Answer 2: Pain with palpation of the subdeltoid bursa would be expected in subdeltoid bursitis
Answer 3: Loss of sensation over the lateral aspect of the shoulder would be expected with axillary nerve injury Answer 5: Sensation of shoulder instability with internal rotation would be expected with posterior instability.
(SAE10SM.31) A 23-year-old man reports a 6-year history of recurrent instability in the right dominant shoulder. He has not undergone surgery and has essentially stopped all of his sporting activities. On examination, he has instability and apprehension in the midrange of motion (abduction of 45 to 60 degrees with external rotation) and a palpable clunk representing a transient dislocation over the anterior glenoid rim. A three-dimensional CT scan is shown in Figure 31. What is the most appropriate surgical intervention to provide him with reliable stability postoperatively? Review Topic
-
Arthroscopic Bankart surgery
-
Bony glenoid augmentation procedure
-
Subscapularis advancement
-
Open capsular shift
-
Hemiarthroplasty
PREFERRED RESPONSE 2
In the setting of significant anteroinferior glenoid bone deficiency (greater than 20% to 25%), both open and arthroscopic Bankart repairs have demonstrated higher rates of failure. Bony glenoid augmentation procedures such as the Bristow-Latarjet, which describe coracoid transfers to reconstruct the deficient glenoid, have led to decreased rates of recurrent shoulder instability. In this scenario, the patient has a significant loss of glenoid bone. There are also several clues in the history to suspect bone deficiency: multiple recurrences, a long history of recurrence, and instability in the midranges of motion. A bony augmentation procedure such as the Latarjet has been well-described to provide a well functioning and stable shoulder joint. A hemiarthroplasty is not indicated in the absence of arthritis. Subscapularis advancement will not address the bone loss.
(SAE11UE.25) A 66-year-old man who underwent shoulder arthroplasty 7 years ago reports progressively worsening shoulder pain for the past 4 weeks after hospital discharge for community-acquired pneumonia. He is afebrile and reports no chills or night sweats. Laboratory studies show a white blood cell count of 11,200/mm3 and an erythrocyte sedimentation rate of 25/h. Shoulder radiographs are negative for fracture, dislocation, or signs of implant loosening. What is the most appropriate management? Review Topic
-
Follow-up in 2 weeks with a repeat white blood cell count and erythrocyte sedimentation rate
-
Shoulder aspiration with Gram stain and culture of fluid
-
Prescription strength nonsteroidal anti-inflammatory drugs
-
Physical therapy for shoulder stretching and modalities
-
Emergent surgical irrigation, debridement, and revision shoulder arthroplasty
PREFERRED RESPONSE 2
The patient may have hematologic spread of the pulmonary infection to the shoulder arthroplasty; however, further work-up is necessary at this point. The elevated laboratory studies may still be secondary to the pulmonary infection. Aspiration of the shoulder joint with stat Gram stain and culture of the fluid is indicated. If the aspirate shows signs of infection and irrigation and debridement is indicated, complete revision of the well-seated implants may not be necessary. Physical therapy and nonsteroidal anti-inflammatory drugs are not indicated until the possibility of a shoulder infection has been ruled out. A wait of 2 weeks to repeat the laboratory values, in the presence of new shoulder pain, is contraindicated.
(SAE11UE.115) A 72-year-old man who underwent an uncomplicated total shoulder arthroplasty 4 weeks ago now reports injuring his shoulder in a fall on the ice. He attempted to catch himself on a railing with his operative arm. He continues to feel pain anteriorly in the shoulder. His range of motion is 140 degrees forward elevation, 90 degrees external rotation with the arm at the side, and internal rotation up the back to L1. Radiographs are normal. What is the most likely diagnosis? Review Topic
1
Deltoid contusion
2
Rupture of the subscapularis repair
3
Traumatic loosening of the glenoid
4
Locked posterior shoulder dislocation
5
Biceps tendon rupture
PREFERRED RESPONSE 2
The patient sustained a rupture of the subscapularis tendon repair. This can occur in the postoperative period with forced internal rotation or excessive external rotation beyond the normal 40 to 60 degrees. On examination, the patient has 90 degrees of external rotation at the side; this is not a normal finding for a 72-year-old man. There is no indication at this time that the glenoid component has loosened or that the patient has a locked posterior dislocation. Both of these would be evident on radiographs. A biceps tendon rupture or a deltoid contusion would not explain the excessive external rotation to 90 degrees as seen on examination.
(SAE11UE.20) A 48-year-old man undergoes arthroscopy to repair a rotator cuff tear. During the arthroscopy, the tear is characterized and found to involve the entire supraspinatus and a majority of the infraspinatus tendons. After mobilization, the posterior rotator cuff can reach the greater tuberosity. However, the supraspinatus tendon cannot reach its insertion point at the greater tuberosity. What is the most appropriate treatment? Review Topic
-
Conversion to a latissimus dorsi muscle tendon transfer
-
Acromioplasty and coracoacromial ligament release
-
Reverse acromioplasty (tuberoplasty)
-
Reverse total shoulder arthroplasty
-
Partial repair of the rotator cuff
PREFERRED RESPONSE 5
If a complete rotator cuff repair is not possible, a partial rotator cuff repair should still be considered and is the appropriate treatment for this patient. In patients with an irreparable massive rotator cuff tear, acromioplasty with coracoacromial ligament
release, reverse acromioplasty, and tenotomy of the biceps tendon may improve shoulder pain. If these procedures fail, then a muscle transfer procedure can also be considered in select patients. If, however, a portion of the rotator cuff can be repaired, even partial repair can balance the coronal and axial forces about the shoulder to restore the kinematics of the joint. Reverse total shoulder arthroplasty is not appropriate for this relatively young patient.
(SAE13SM.99) A 24-year-old collegiate pitcher has had increasing pain over his medial elbow for 3 months. He has point tenderness over his medial epicondyle and reproduction of his symptoms with a valgus stress test. What phase of the throwing cycle most likely will reproduce his symptoms? Review Topic
-
Early cocking
-
Late cocking
-
Acceleration
-
Deceleration
PREFERRED RESPONSE 2
This patient is experiencing soreness over his medial (ulnar) collateral ligament. Valgus overload is likely to reproduce his symptoms and is most pronounced during the late cocking phase of the throwing cycle. In wind up, very little elbow torque is required. In early cocking, the arm is getting loaded and maximum valgus is not yet achieved at the elbow. In acceleration and deceleration, more force is generated at the level of the shoulder joint.
(SAE10SM.54) A 17-year-old pitcher reports pain over the medial aspect of the elbow that occurs during the acceleration phase of throwing, and it prevents him from throwing at the velocity needed to be competitive. What structure is most likely injured in this patient? Review Topic
-
Radial collateral ligament
-
Posterior bundle of the ulnar collateral ligament
-
Anterior bundle of the ulnar collateral ligament
-
Flexor carpi ulnaris
-
Pronation teres
PREFERRED RESPONSE 3
The anterior bundle of the ulnar collateral ligament of the elbow is the primary constraint to valgus force of the elbow. In pitchers and in overhead athletes, injury to this portion of the ligament results in valgus instability. Reconstruction of the anterior band of the ulnar collateral ligament is necessary in many elite athletic throwers to allow them to return to this competitive activity.
(SAE08UE.123) What is the most important stabilizing mechanism in the midrange of motion of the glenohumeral joint? Review Topic
-
Concavity compression
-
Isometric articular ligaments
-
Increased tensile force of the capsule
-
Biceps tendon
-
Deltoid contraction
PREFERRED RESPONSE 1
Concavity compression is a stabilizing mechanism by which muscular compression of the humeral head into the glenoid fossa stabilizes the glenohumeral joint against shear forces. This is dependent on the depth of the concavity and the magnitude of the compressive force.
(OBQ15.112) A 13-year-old baseball pitcher presents with worsening medial-sided elbow pain. He pitches 7 months out of the year, throws 85 pitches per game and plays in two games per week. His fastball speed is approximately 75mph. He regularly plays outfield once he has been relieved of pitching. Which of the following is most likely contributing to his elbow pain? Review Topic
-
The number of months he plays per year
-
The number of pitches he throws per game
-
The number of games he plays per week
-
His fastball speed
-
Playing in another position once being relieved of pitching
PREFERRED RESPONSE 2
Young athletes who throw greater than 80 pitches per game have an increased risk of shoulder and elbow injury. For a 13-year-old, the recommended maximum number of pitches per game is 75.
Little League elbow is a medial-sided overuse injury that occurs in the skeletally immature athlete. During execution of the baseball pitch, tremendous valgus and extension stresses occur at the elbow. Repetitive microtrauma can ultimately injure the medial epicondyle apophysis, ulnar collateral ligament or the flexor-pronator muscle mass. Limiting the number of pitches and innings played per game, as well as the number of months of competitive pitching per year, has been recommended to prevent these overuse injuries in the young athlete.
Olsen et al performed a case control study to determine risk factors associated with the development of shoulder and elbow injuries in adolescent baseball pitchers. Greater than 8 months of competitive pitching per year, more than 80 pitches per game and a fastball speed of greater than 85mph were all associated with increased risk of injury. Continued pitching despite arm fatigue and pain, being a starting pitcher, greater number of warm-up pitches, participating in showcases and regular use of NSAIDs were also associated with injury. The type of pitch (fastball, changeups and breaking balls) and continued play in a different position once being relieved was not associated with increased risk of injury.
Andrews et al authored a review article on ulnar collateral ligament injuries in throwing athletes. According to the USA Baseball Medical/Safety Advisory Committee, young baseball pitchers should avoid breaking pitches, such as curveballs and sliders, and avoid year-round baseball. A minimum of 3 months of complete rest from pitching per year is vital. Youth pitching coaches should be educated to ensure proper pitching mechanics are being reinforced.
Illustration A is a table depicting the recommended maximum number of pitches by age group.
Incorrect Answers:
Answer 1: Competing greater than 8 months per year is associated with increased risk of injury.
Answer 3: Youth pitchers can perform in a maximum of 2 games per week without increased risk of injury. Answer 4: Fastball speed greater than 85mph is associated with increased risk of injury.
Answer 5: Playing in another position once being relieved of pitching is not associated with increased risk of injury.
(SAE11UE.123) With the arm abducted 90 degrees and fully externally rotated, which of the following glenohumeral ligaments resists anterior translation of the humerus? Review Topic
-
Coracohumeral ligament
-
Superior glenohumeral ligament
-
Middle glenohumeral ligament
-
Anterior band of the inferior glenohumeral ligament complex
-
Posterior band of the inferior glenohumeral ligament complex
PREFERRED RESPONSE 4
With the arm in the abducted, externally rotated position, the anterior band of the inferior glenohumeral ligament complex moves anteriorly, preventing anterior humeral head translation. Both the coracohumeral ligament and the superior glenohumeral ligament restrain the humeral head to inferior translation of the adducted arm, and to external rotation in the adducted position. The middle glenohumeral ligament is a primary stabilizer to anterior translation with the arm abducted to 45 degrees. The posterior band of the inferior glenohumeral ligament complex resists posterior translation of the humeral head when the arm is internally rotated.
(SAE10PE.73) A 15-year-old right-handed pitcher reports shoulder pain after throwing. His symptoms have been present for 3 months and have been getting progressively worse. Clinical examination shows no atrophy of the shoulder muscles, but he has pain with resisted motion of the shoulder, especially internal rotation. Radiographs are shown in Figures 73a and 73b. What is the next step in the evaluation and treatment of his shoulder pain? Review Topic
-
MRI/arthrogram of the right shoulder
-
CT of the right proximal humerus
-
Bone biopsy of the right proximal humerus
-
Cessation of throwing for 6 to 8 weeks, followed by a progressive throwing program
-
Arthroscopic evaluation of the right shoulder
PREFERRED RESPONSE 4
The patient has proximal humeral epiphyseolysis, otherwise known as "Little League shoulder." This is an overuse injury of the shoulder in the skeletally immature overhead throwing athlete. Most frequently seen in pitchers, it usually develops after an increase in the amount or intensity of throwing activity. Initial treatment involves cessation of throwing activities so the proximal humeral growth plate injury can heal, followed by a gradual return to throwing.
(OBQ13.227) A 25-year-old lineman is referred to your office for a second opinion. 1 year ago, he underwent an arthroscopic procedure for shoulder instability. He complains of persistent sense of instability despite the surgery. Which of the following is a contraindication to revision arthroscopic labral repair for recurrent anterior glenohumeral instability? Review Topic
-
Glenoid bone loss of 10%
-
Capsular attenuation from prior thermal capsulorraphy
-
Anterior labral periosteal sleeve avulsion (ALSPA ) lesion
-
Glenoid labral articular defect (GLAD) lesion
-
Combined Superior Labrum from Anterior to Posterior tear (SLAP) and recurrent Bankart lesion
PREFERRED RESPONSE 2
Capsular attenuation or postthermal capsular necrosis from prior thermal capsulorraphy is a contraindicated to arthroscopic repair.
Thermal capsulorrhaphy utilizes heat generated by radiofrequency or laser ablation to cause capsular shrinkage in an effort to treat shoulder instability. However, high recurrence rates have been found, especially around two to three weeks after the index procedure, when the capsular tissue is the weakest. In the setting of recurrence following thermal capsulorrhaphy, open revision is recommended.
Creighton et al. reported on a series of 18 patients undergoing revision arthroscopic stabilization. Of the 18, 3 failed with recurrent instability, all with previous thermal capsulorrhaphy.
Miniaci et al. reviewed the outcomes following thermal capsulorrhaphy noting high rates of recurrent instability, especially in the setting of initial treatment for multidirectional instability.
Park et al. reported on a series of 14 patients undergoing revision following thermal capsulorrhaphy. Ten out of 14 patients had signs of capsular thinning, insufficiency and attenuation.
Wong et al. surveyed 379 shoulder surgeons on the complications following thermal capsulorrhaphy. Capsular insufficiency and thinning were commonly associated with recurrent instability.
Hecht et al. performed thermal capsulorrhaphy and biomechanical analysis of the capsule in a sheep model. The authors found that the capsule was weakest at the 2-3 week post-operative timepoint, leading to the highest rate insufficiency, attenuation and mechanical failure at this time.
Incorrect answers:
Answer 2.Glenoid bone loss of 10% can still be addressed arthroscopically. The typical indication for open repair with a bony procedure such as Latarjet procedure is 20%.
Answers 3, 4, and 5: ALSPA lesions, GLAD lesions and combined SLAP and recurrent Bankart lesions can be addressed arthroscopically.
(SAE11OS.132) Figures 132a and 132b are the lateral and anteroposterior radiographs of a 15-year-old boy with a 6-month history of recurrent, activity-related posterior elbow pain when pitching. Two separate 6-week periods of rest have failed to provide relief. What is the next best step to enable him to return to play? Review Topic
-
Physiotherapy
-
Long-arm cast
-
Cannulated screw fixation
-
Plate fixation of the ulna
-
Hinged-elbow bracing
PREFERRED RESPONSE 3
Intramedullary screw fixation of the olecranon stress fracture is most likely to allow him to return to play. Stress fractures through a persistent olecranon apophysis have been well described in the literature. The AP radiograph reveals the other physes of the elbow to be closed. After patients fail to respond to appropriate periods of rest and
cessation from throwing followed by appropriate physiotherapy, surgical management with cannulated screw fixation is appropriate and has been demonstrated to have favorable success rates. Hinged-elbow bracing will not facilitate healing or return to play. Long-arm casting is likely to result in stiffness and would not be unreasonable for a short duration at the onset of symptoms, but is less likely to be helpful at this point. Plate fixation is not indicated for treatment of this injury.
(SAE13SM.33) A 25-year-old recreational soccer player has recurrent shoulder dislocations. He first dislocated his shoulder playing football in high school, was treated in a sling for 6 weeks, and returned to play for the remainder of the season. He did well until 2 years later when he reinjured the shoulder. He says that his shoulder dislocates with little injury and always "feels loose." Examination reveals anterior instability and an MR arthrogram reveals an anterior-inferior labral tear and surgical treatment is recommended. He inquires about the benefits of arthroscopic vs open procedure. Which of the following statements reflects an advantage associated with arthroscopic procedures compared to open stabilization? Review Topic
-
Range of motion might be slightly better after an arthroscopic procedure.
-
Rate of recurrent instability is lower after an arthroscopic procedure.
-
Rates of return to work are higher after an arthroscopic procedure.
-
Rates of return to sports are higher after an arthroscopic procedure.
PREFERRED RESPONSE 1
There is much debate in the literature regarding optimal techniques for treatment of shoulder instability. Most studies have suggested a slightly better range of motion of the shoulder after an arthroscopic repair. Recurrent instability rates have been slightly higher with arthroscopic procedures in some studies, while others show the rates are not statistically different. Return to work and/or sports has been shown to be equal or slightly better with open procedures.
(OBQ14.12) A 72-year-old woman presents for follow-up after elbow surgery. Her radiographs are shown in Figures A and B. Which of the following pre-operative diagnoses is a relative contraindication to the use of this prosthesis design? Review Topic
-
Acute intra-articular distal humerus fracture
-
Malunited intra-articular distal humerus fracture
-
Late-stage rheumatoid arthritis
-
Post-traumatic bony ankylosis
-
Osteoarthritis
PREFERRED RESPONSE 3
This patient has had an unconstrained total elbow arthroplasty (TEA). Unconstrained TEA is least preferred for late-stage rheumatoid arthritis where there is significant capsuloligamentous instability and bony erosion.
Unconstrained (unlinked or resurfacing prosthesis) TEA depend on intact bony and ligamentous constraints for stability. These are appropriate for humeroulnar conditions with intact collateral ligaments and radiocapitellar articulation e.g. osteoarthritis, post-traumatic arthritis, intra-articular distal humerus fracture, and malunion of the distal humerus. Conditions with increased risk of instability (ligamentous injury, rheumatoid arthritis) will benefit from a linked or semiconstrained prosthesis.
Mansat et al. reviewed the Coonrad-Morrey linked (semi-constrained) TEA implant in 70 patients after 5 years. They found that patients with inflammatory arthritis had higher function than those with traumatic conditions (fractures, nonunions and posttraumatic arthritis). Survival rate was 98% and 91% at 5 and 10 years, respectively. They concluded that this implant provided satisfactory treatment for different indications although radiolucent lines and bushing wear were a concern.
Hildebrand et al. reviewed the functional outcome of the Coonrad-Moorey prosthesis in 51 elbows after 50 months. The inflammatory arthritis group had higher performance scores than the traumatic/post-traumatic conditions group. Isometric extensor torque was found to be less than the nonoperated side. Radiolucency was noted in 11 elbows.
Figures A and B show an unconstrained TEA with radial head replacement. Illustration A shows more examples of unconstrained TEA. Illustration B shows a semiconstrained TEA. The arrow points to the anterior flange. Illustration C shows radiolucent lines around the stems. Illustration D shows severe bushing wear leading to locking mechanism failure. Illustration E is a table comparing linked and unlinked implants.
Incorrect Answers:
Answers 1, 2, 4, 5: Primary ligamentous restraints around the elbow are likely to be intact in these conditions. Both linked (semiconstrained) and unlinked (unconstrained) prosthesis are options.
(SAE11OS.93) What prosthetic factor has the most impact on decreasing the rate of scapular notching in a Grammont-style reverse total shoulder arthroplasty? Review Topic
-
Posterior tilt of the glenoid component
-
Inferior tilt of the glenoid component
-
Inferior positioning of the glenoid component
-
Use of a cemented humeral component
-
Use of locking screws in the glenoid component
PREFERRED RESPONSE 3
A low position of the glenoid base plate has been shown to have the greatest effect on decreasing scapular notching with a Grammont-style prosthesis. Scapular notching is the phenomena seen after reverse total shoulder arthroplasty when bone along the inferior scapular neck is lost. It is thought to be the result of repeated contact between the humeral component and the bone. The Grammont-style reverse total shoulder arthroplasty has a medialized center of rotation that decreases strain at the glenoid component but has less space for the humerus to clear the scapula. Scapular notching was seen least in components that are placed low on the glenoid. Posterior and inferior tilt has minimal effect on scapular notching and may even increase notching by bringing the humerus closer to the scapula. The use of locking screws and a cemented humeral stem had no influence on notching.
(SAE11UE.108) A 23-year-old woman with recurrent anterior instability undergoes an open Bankart procedure. Six months after surgery the patient reports shoulder
weakness and is concerned about instability of the shoulder joint. An MRI scan is shown in Figure 108. What is the most appropriate management? Review Topic
-
Physical therapy
-
Biceps tenolysis
-
Subscapularis repair
-
Supraspinatus repair
-
Pectoralis major repair
PREFERRED RESPONSE 3
The axial MRI scan shows rupture of the subscapularis tendon with dislocation of the biceps tendon. Treatment should include a biceps tenotomy or tenodesis in conjunction with a subscapularis repair. A pectoralis major transfer may be necessary in chronic cases where the subscapularis is irreparable, but in this patient the tendon is repairable. As a single operation, biceps tenolysis will not correct the instability, and would likely result in a cosmetic deformity. Physical therapy will not restore subscapularis function.
(OBQ15.12) Figure A is an AP radiograph of a 68-year-old man who presents to clinic with shoulder pain and dysfunction. On examination of his shoulder, he has pseudoparalysis with attempt at forward elevation and a positive hornblower's sign while demonstrating normal belly press test. Treatment should consist of: Review Topic
-
Arthroscopic rotator cuff repair
-
Shoulder hemiarthroplasty
-
Anatomic total shoulder arthroplasty
-
Anatomic total shoulder arthroplasty and pectoralis major transfer
-
Reverse total shoulder arthroplasty and latissmus dorsi transfer
PREFERRED RESPONSE 5
The clinical presentation and radiograph is consistent with a diagnosis of a massive posterosuperior rotator cuff tear and arthropathy. Of the options listed, a reverse total shoulder arthroplasty (RTSA) with latissmus dorsi transfer (LDT) is most appropriate.
RTSA can improve pain and function in shoulders with forward elevation pseudoparalysis secondary to rotator cuff tear arthropathy. Following arthroplasty, the deltoid alone can restore overhead elevation but it does not address active external rotation deficit. LDT is a well described procedure for treatment of irreparable posterosuperior rotator cuff tear. Combining RTSA and LDT can address both deficits and in select patients yields significant pain relief and restoration of function.
Walch et al found that hornblower's sign had 100% sensitivity and 93% specificity for irreparable degeneration of teres minor.
Puskas et al present clinical outcomes of RTSA combined with LDT for treatment of chronic combined pseudoparesis of elevation and external rotation of the shoulder in 40 patients. At a mean follow-up of 53 months, the author report excellent clinical outcomes.
Figure A demonstrates a proximal migration of the humerus resulting in femoralization of the humeral head and acetabularization of the acromion from a massive rotator cuff tear.
Incorrect answers:
Answer 1: Arthroscopic rotator cuff repair alone would not address the degenerative joint process in this shoulder. Answer 2: Shoulder hemiarthroplasty may improve pain but would have poor functional outcomes in patients concomitant massive rotator cuff tear. Answer 3: Anatomic total shoulder arthroplasty in the setting of rotator cuff tear arthropathy would not improve the pseudoparalysis and result in poor functional outcome.
Answer 4: Anatomic total shoulder arthroplasty with pectoralis major transfer may be considered considered in patients with massive irreparable anterior rotator cuff tears and joint degeneration.
(SAE11UE.35) The standard Bankart lesion involves detachment of the labrum along with which of the following capsular ligaments? Review Topic
-
Superior glenohumeral ligament and coracohumeral ligament
-
Superior glenohumeral ligament and middle glenohumeral ligament
-
Middle glenohumeral ligament and inferior glenohumeral ligament
-
Inferior glenohumeral ligament
-
Superior glenohumeral ligament, middle glenohumeral ligament, and inferior glenohumeral ligament
PREFERRED RESPONSE 3
The Bankart lesion involves detachment of the labrum corresponding to the attachment of the middle and inferior glenohumeral ligaments. The superior glenohumeral ligament and the coracohumeral ligament are too superior, inserting near the biceps tendon, and play no role in the Bankart lesion.
(SAE11OS.41) Decreased risk of shoulder and elbow injury in a throwing athlete has been demonstrated with which of the following? Review Topic
-
Rotator cuff strengthening
-
Superior labral repair
-
Posterior capsular stretching
-
Periscapular muscle strengthening
-
Repair of partial-thickness rotator cuff tears
PREFERRED RESPONSE 3
Posterior capsular contracture has been demonstrated to significantly impair the ability of the humeral head to translate anterior and inferiorly during the late cocking and early acceleration phases of the throwing motion. This results in an obligatory posterosuperior translation of the humeral head that may contribute to posterior superior glenohumeral internal impingement with posterosuperior labral and articular-sided rotator cuff pathology. Posterior capsular stretching in throwing athletes has been demonstrated to decrease the likelihood of clinically significant shoulder or elbow injury. Periscapular muscle and rotator cuff strengthening are important for optimal scapulothoracic rhythm, stable scapular position for throwing, and rotator cuff function but less directly established to result in a decreased risk of shoulder and elbow injury than posterior capsular stretching. Partial-thickness rotator cuff repair and superior labral repair may be necessary for treatment of symptomatic lesions unresponsive to nonsurgical management, but these do not necessarily correlate with decreased shoulder and elbow injury risk.
(SAE13SM.1) Figure 1 is the MRI scan of a 19-year-old man who has an acute anterior shoulder dislocation. The bony fragment occupies 10% of the glenoid articular surface. What is the most appropriate treatment? Review Topic
-
Open structural iliac crest graft
-
Open reduction and internal fixation
-
Arthroscopic coracoid transfer
-
Arthroscopic repair incorporating the bone lesion
PREFERRED RESPONSE 4
The MRI scan shows a bony Bankart lesion involving less than 20% of the glenoid joint surface. A recent series reported high success rates after arthroscopic treatment when the defect is incorporated into the repair. Anterior bony deficiencies occupying more than 25% to 30% of the glenoid joint surface treated with soft-tissue repair only are associated with high recurrence rates. In these patients, an open or arthroscopic coracoid transfer or structural iliac crest graft should be considered. Open reduction and internal fixation has been reported for treatment of large acute glenoid rim fractures but is not recommended for recurrent anterior shoulder instability in the setting of a 10% glenoid rim fracture.
(SAE11UE.54) A 38-year-old man reports a 6-week history of shoulder pain and stiffness after falling on the stairs and landing onto the affected side. Radiographs are shown in Figures 54a and 54b. What is the most appropriate treatment? Review Topic
-
Physical therapy including ultrasound and gentle stretches
-
Closed manipulation of the shoulder
-
MRI and possible rotator cuff repair
-
Open glenohumeral reduction, with possible lesser tuberosity transfer
-
Shoulder hemiarthroplasty
PREFERRED RESPONSE 4
The patient has a chronic posterior shoulder dislocation of 6-weeks duration. A CT scan will provide preoperative information regarding the size of the McLaughlin or reverse Hill-Sachs lesion. Open glenohumeral reduction with transfer of the lesser tuberosity and attached subscapularis has been shown to be successful in stabilizing a posterior dislocation. Closed reduction is highly unlikely to achieve a reduction and may cause displacement of an unrecognized humeral surgical neck fracture. Hemiarthroplasty would be considered for lesions involving more than 50% of the humeral head or when the joint has been dislocated for several months and late collapse of the head postreduction is likely. Rotator cuff tears are not commonly associated with posterior shoulder dislocation.
(SAE11UE.103) A 77-year-old woman underwent semiconstrained right total elbow arthroplasty 4 weeks ago through a Bryan-Morrey approach. Her recovery was uneventful until 2 days ago when she began her physical therapy session at an outpatient clinic. During resisted extension exercises, she felt a "pop" in her elbow, accompanied with pain and inability to extend her elbow against resistance. What is the most likely cause of her symptoms? Review Topic
-
Fracture of the ulnar component
-
Disengagement of the axle of the prosthesis
-
Failure of the triceps mechanism repair
-
Periprosthetic fracture of the humerus
-
Periprosthetic fracture of the ulna
PREFERRED RESPONSE 3
During a Bryan-Morrey approach for total elbow arthroplasty, the triceps is dissected free from its ulnar insertion and reflected laterally. At the conclusion of the procedure, the triceps tendon is reattached to the ulna through drill holes. Whereas motion can be initiated postoperatively, 6 to 8 weeks of protection are recommended before initiation of resistance exercises to protect the triceps repair. A periprosthetic fracture or component failure is rare in the absence of more significant trauma, and they are usually late complications.
(SAE10SM.26) Internal impingement is characterized by which of the following anatomic lesions? Review Topic
-
Subscapularis tear
-
Bursal-sided rotator cuff tear
-
Articular-sided rotator cuff tear
-
Tight anterior capsule
-
Laxity of the posterior capsule
PREFERRED RESPONSE 3
Internal impingement is characterized by articular-sided partial-thickness rotator cuff tears and superior glenoid labral tears. The capsule is characterized by laxity anteriorly and tightness posteriorly.
(SAE11UE.19) A 27-year-old man has recurrent anterior shoulder instability following an arthroscopic Bankart repair 4 years ago. Current CT scans are shown in Figures 19a and 19b. Deficiency of what mechanism is most likely to contribute to the current joint instability? Review Topic
-
Synovial fluid adhesion-cohesion
-
Negative intra-articular pressure
-
Concavity-compression of the humeral head in the glenoid
-
Decreased functional arc of motion as a result of a Hill-Sachs lesion
-
Poor rehabilitation of scapulothoracic rhythm
PREFERRED RESPONSE 3
Loss of the anterior glenoid rim can commonly occur as a result of acute fracture or progressive wear following multiple dislocations. This decreases the effective depth of the glenoid. The ability of the rotator cuff to stabilize the joint through production of a joint reactive force is markedly decreased. Synovial fluid adhesion-cohesion and negative intra-articular pressure are maintained in the closed capsular space. The Hill-Sachs lesion in this case is not large enough to be a significant factor in failed Bankart repair. Poor scapulothoracic rhythm can increase the risk of instability but is not typically the primary factor.
(SAE11OS.179) A 56-year-old woman undergoes an arthroscopic rotator cuff repair for a two-tendon retracted tear (supraspinatus and infraspinatus), requiring the use of four suture anchors placed in a double row technique. At her 1 month follow-up visit, what is the appropriate recommendation for her continued rehabilitation program? Review Topic
-
Initiate isometric external rotation strengthening and continue passive range of motion.
-
Initiate eccentric supraspinatus strengthening and continue passive range of motion.
-
Initiate light resistance training to minimize atrophy and continue passive range of motion.
-
Continue passive range of motion and initiate concentric deltoid strengthening.
-
Continue passive range of motion with no active strengthening of the shoulder muscles.
PREFERRED RESPONSE 5
Regardless of the technique of rotator cuff repair, the biology of tendon healing remains the same. Therefore, the repaired muscle tendon(s) must be protected from stress for a minimum of 6 weeks and more likely 8 weeks in a large two-tendon tear such as this patient had repaired. Therefore, at the 1 month follow-up visit, the patient should continue strict passive motion exercises and should perform no strengthening activities. Deltoid strengthening cannot be isolated from rotator cuff strengthening; therefore, deltoid strengthening is inappropriate as well. Because the infraspinatus is the primary shoulder external rotator, it should not be strengthened for 6 to 8 weeks. Supraspinatus strengthening at this time frame would likely ensure its disruption and result in failure of the surgery. Any resistance training at 1 month from surgery would likely result in tendon failure at the tendon-bone interface. The obligatory need to protect the muscles during healing will predictably result in atrophy but it is easier to strengthen healed muscles than it is to strengthen muscle/tendon units that have failed to heal.
(OBQ13.165) The right shoulder exercise seen in Figure A will put the LEAST amount of stretch on which structure? Review Topic
-
Inferior glenohumeral ligament
-
Coracohumeral ligament
-
Anterior-superior capsule
-
Superior glenohumeral ligament
-
Posterior capsule
PREFERRED RESPONSE 5
Shoulder wand exercises, as shown in Figure A, are used to increase external range of motion of the shoulder. With the arm adducted and the elbow flexed, this exercise will put the LEAST amount of stretch on the posterior capsule.
External rotation shoulder wand exercises are commonly used for the treatment of adhesive capsulitis. Adhesive capsulitis is most commonly caused by contracture of the rotator interval. The rotator interval includes the anterior-superior capsule, superior glenohumeral ligament, coracohumeral ligament and long head biceps tendon. The structure most commonly contracted is the anterior-superior capsule, which limits external rotation when the arm is adducted.
Kuhn et al. showed that in the neutral position, each ligament except the posterior capsule significantly affected the torque required for external rotation. The greatest effect on resisting external rotation at 0 degrees of abduction was the entire inferior glenohumeral ligament > coracohumeral ligament > anterior band of the inferior glenohumeral ligament > superior and middle glenohumeral ligament.
Harryman et al. looked at the role of the rotator interval capsule in passive motion and stability of the shoulder. They found operative alteration of this capsular interval was found to affect flexion, extension, external rotation, and adduction of the humerus with respect to the scapula. Limitation of external motion was increased by operative imbrication of the rotator interval and decreased by sectioning of the rotator interval capsule.
Kim et al. reviewed shoulder MRIs to determine if abnormalities of the rotator interval were correlated with chronic shoulder instability. They found a significantly larger rotator interval height, rotator interval area, and rotator interval index in patients with chronic anterior shoulder instability compared to patients without instability.
Figure A shows a patient performing an exercise to increase right shoulder external rotation with a wand/stick. The right arm is fully adducted by her side, and her elbow flexed at 90 degrees.
Incorrect Answers:
Answers 1,2,3,4: These structures are all the static stabilizers of the shoulder. Each will be subjected to a significant degree of stretch when the arm is externally rotated.
(SAE13SM.23) Figure 23 is the T2 axial MRI scan of a 21-year-old man who was injured while playing for his college football team. His pain was aggravated with blocking maneuvers and alleviated with rest, and he had to stop playing because of the pain. What examination maneuver most likely will reproduce his pain? Review Topic
-
Forward elevation in the scapular plane
-
External rotation and abduction
-
Flexion, adduction, and internal rotation
-
Flexion and abduction
PREFERRED RESPONSE 3
This patient has a mechanism of injury and MRI consistent with a posterior labral tear and posterior instability. Flexion, adduction, and internal rotation produce a net posterior vector on the glenohumeral joint and should reproduce this patient's symptoms. Pain or instability with the arm elevated in the scapular plane describes an impingement sign. Pain or instability with the arm in external rotation and abduction describes the apprehension sign. Pain or instability with the arm in flexion and abduction is a nonspecific finding.
(SAE08UE.120) What neurovascular structure is at greatest risk when creating a proximal anterolateral elbow arthroscopy portal? Review Topic
-
Lateral antebrachial cutaneous nerve
-
Radial nerve
-
Posterior interosseous nerve
-
Median nerve
-
Brachial artery
PREFERRED RESPONSE 2
The radial nerve is 4 to 7 mm from the anterolateral portal, which is placed 1 cm anterior and 3 cm proximal to the lateral epicondyle. The posterior interosseous nerve can lie 1 to 14 mm from the portal site.
(SAE08UE.87) Figures 42a and 42b show the radiographs of a 52-year-old man who sustained a fall from a motorcycle 6 months ago and now reports pain and stiffness in his left shoulder. What is the most reliable treatment to improve function and comfort of the shoulder? Review Topic
-
Vigorous physical therapy
-
Manipulation under anesthesia
-
Arthroscopic capsular release
-
Hemiarthroplasty
-
Arthroscopic capsular plication
PREFERRED RESPONSE 4
Appropriate treatment is based on multiple considerations, which include the chronicity of the dislocation, the amount of humeral head involvement, the medical condition, and functional limitations of the patient. It has been shown that shoulder arthroplasty for locked posterior dislocation provides pain relief and improved motion. Transfer of the lesser tuberosity with its attached subscapularis tendon into the defect is recommended for anteromedial humeral defects that are smaller than approximately 40% of the joint surface. Subscapularis transfer as described by McLaughlin and the modification thereof later described by Hawkins and associates in which the lesser tuberosity is transferred into the defect, have yielded good results if the defect is less than 40% of the humeral head. Prosthetic replacement is preferred for larger defects. If the dislocation is less than 3 weeks old and has less than 25% of humeral head involvement, closed reduction with the patient under general anesthesia should be attempted and the stability assessed by internally rotating the arm. If the arm can be safely internally rotated to the abdomen, then 6 weeks of immobilization in an orthosis that maintains the shoulder in slight extension and external rotation can yield a good result. If the dislocation has been present for more than 3 weeks, closed reduction becomes exceedingly difficult.
(SAE08UE.10) An MRI arthrogram of the elbow is shown in Figure 6. Based on these findings, what is the most likely diagnosis? Review Topic
-
Rupture of the medial collateral ligament
-
Rupture of the lateral collateral ligament
-
Intra-articular loose body
-
Flexor-pronator injury
-
Extensor origin avulsion
PREFERRED RESPONSE 1
Carrino JA, Morrison WB, Zou KH, et al: Noncontrast MR imaging and MR arthrography of the ulnar collateral ligament of the elbow: Prospective evaluation of two-dimensional pulse sequences for detection of complete tears. Skeletal Radiol 2001;30:625-632.
(OBQ14.41) A 68-year-old man presents with severe right shoulder pain. He had a prolonged course of physical therapy and received several cortisone injections for his pain without improvement. Examination reveals pseudoparalysis of the right shoulder with a 20-degree external rotation lag with the shoulder adducted. With the shoulder placed in 90 degrees of abduction, he can actively externally rotate his shoulder. The patient was treated with a medialized reverse prosthesis shown in Figure A. Which of the following statement is true regarding this treatment option? Review Topic
-
It is contraindicated in patients with shoulder pseudoparalysis
-
It can be used in patients with deltoid dysfunction when combined with latissimus dorsi transfer
-
It shifts the center of rotation of the shoulder superior and lateral
-
The risk of scapular notching is increased with inferior placement of the glenoid component
-
The risk of instability is increased with an irreparable subscapularis
PREFERRED RESPONSE 5
The clinical presentation is consistent with a patient with pseudoparalysis that was treated with a reverse total shoulder arthroplasty (RTSA). The risk of postoperative instability is increased in patients with an irrepairable subscapularis when a medialized reverse prosthesis is used. Answers 1-4 are false statements.
RTSA is most commonly indicated for rotator cuff arthropathy. However, indications for use now include shoulder pseudoparalysis, anterosuperior escape of the humeral head, acute 3 or 4-part proximal humerus fractures, and greater tuberosity fracture nonunions. Contraindications to RTSA included deltoid dysfunction, insufficient glenoid bone stock, and bony deficiency of the acromion.
Edwards et al. prospectively evaluated the risk of shoulder dislocation after reverse TSA. They found a significantly increased risk of dislocation (p=0.012) in patients with an irreparable subscapularis at time of surgery. There were no dislocations in the reparable group. Dislocations were more likely in patients with proximal humeral nonunions and failed prior arthroplasty.
Mulieri et al. looked at the use of reverse TSA in patients with irreparable massive rotator cuff tears without evidence of glenohumeral arthritis. All outcomes were improved postoperatively, and they advocate for reverse TSA in this subset of patients. Survivorship was over 90% at more than 4 years average follow up.
Boileau et al. evaluated the clinical outcomes of isolated biceps tenotomy/tenodesis in patients with massive rotator cuff tears and a biceps lesion. They found that the procedure can effectively treat pain and improve function in these patients. There was no difference in patients undergoing tenotomy versus tenodesis.
Figure A is a right shoulder radiograph status post RTSA with components in adequate position.
Incorrect Answers:
Answer 1: Pseudoparalysis refers to loss of shoulder active motion with intact passive motion in patients due to pain. This often occurs in patients presenting with arthritis,fractures and rotator cuff tears. It is an indication for RTSA Answer 2: RTSA is contraindicated in patients with deltoid dysfunction. RTSA can be combined with latissimus dorsi transfer in patients with intact deltoid function to restore external rotation. Answer 3: RTSA shifts the center of motion INFERIOR and MEDIALLY giving the deltoid muscle a mechanical advantage. Answer 4: Risk of scapular notching can be DECREASED with inferior placement and adequate inferior tilt of the glenoid component.
(SAE11UE.119) A 35-year-old construction worker continues to have weakness with lifting overhead 2 years after he was treated with physical therapy for a "chest muscle" tear. An obvious deformity noted in his axilla worsens with resisted extension and adduction. A clinical photograph and MRI scan are shown in Figures 119a and 119b. What is the most appropriate treatment? Review Topic
-
Allograft reconstruction with semitendinosis weave to the humerus
-
Latissimus dorsi tendon transfer
-
Electrical stimulation
-
Shoulder arthrodesis
-
Arthroscopic pectoralis major tendon repair
PREFERRED RESPONSE 1
This scenario describes a chronic, symptomatic pectoralis major tendon rupture in a young laborer. Direct repair is difficult at this time; therefore, allograft reconstruction is a good alternative to recover strength. Tendon transfers, electrical stimulation, shoulder arthrodesis, and arthroscopy are not indicated in this patient. They will not offer proper reconstruction of the lost muscle tendon unit and/or cosmetic repair.
(SAE11AN.36) An active 45-year-old man sustained an acute traumatic anteroinferior dislocation. MRI scans and an arthroscopic view are shown in Figures 36a through
36c. The lesion represents compressive injury to which of the following structures? Review Topic
-
Greater tuberosity
-
Lesser tuberosity
-
Posterosuperior humeral head
-
Superior glenoid
-
Central portion of the humeral head
PREFERRED RESPONSE 3
During an anteroinferior dislocation, the posterosuperior portion of the humeral head impacts the inferior rim of the glenoid, resulting in an impaction injury. This lesion is classically referred to as a Hill-Sachs lesion.
(SAE11UE.104) A 47-year-old man who is right-hand dominant reports lateral-sided elbow pain after playing golf. His symptoms developed gradually and without trauma, and he has pain with gripping and repetitive movements with the hand and wrist. Examination reveals his shoulder and wrist to be normal, and the elbow has no effusion and normal range of movement. He is tender near the lateral epicondyle, and symptoms are exacerbated with resisted wrist extension. Radiographs are shown in Figures 104a and 104b. What is the next most appropriate step in management? Review Topic
-
Subtendinous epicondylar corticosteroid injection
-
Corticosteroid injection into the radial tunnel
-
MRI of the elbow
-
Percutaneous extensor carpi radialis brevis tenotomy
-
Physical therapy for an eccentric conditioning and strengthening program
PREFERRED RESPONSE 5
The patient has lateral epicondylitis of relatively short duration. At this early stage of disease, nonsurgical management is indicated. An eccentric physical therapeutic exercise program has been shown to have a beneficial effect on tendon biology; therefore, it would be the most appropriate initial management. While the diagnosis of lateral epicondylitis may be confused with radial tunnel syndrome, the clinical examination and history are most suggestive of the former. Corticosteroid injection has been shown to help with symptoms in short-term follow-up, but does little to affect the natural progression of the condition; it is more appropriate as a second line of treatment. MRI may be beneficial in patients with refractory disease and/or when the diagnosis is in question. Percutaneous surgical treatment is indicated only when nonsurgical measures fail to provide relief.
(SAE08UE.82) An extended head hemiarthroplasty (rotator cuff tear arthropathy head) has what theoretic advantage when compared to a standard hemiarthroplasty? Review Topic
-
Improved superior stability
-
Fixed fulcrum kinematics
-
Creates a metal-to-bone articulation with the acromion
-
Increased deltoid moment arm
-
Increased glenohumeral offset
PREFERRED RESPONSE 3
The theoretic advantage of a metal-to-bone articulation with the acromion is that there is a greater arc in which a smooth metal surface contacts the glenoid and acromion. This may improve pain and function, but no studies have evaluated this to date. One study showed results comparable to that of a standard hemiarthroplasty. There are no other biomechanic advantages.
(SAE08UE.39) The usual presentation of traumatic subscapularis tears is most often seen after forced Review Topic
-
internal rotation.
-
external rotation.
-
extension.
-
abduction.
-
forward flexion.
PREFERRED RESPONSE 2
The typical mechanism of injury is a fall and the patient grasps something to prevent the fall. This maneuver forces the arm into external rotation against resistance.
(OBQ15.209) A 45-year-old male auto mechanic presents to your office with left lateral elbow pain for 6 weeks. On physical exam he has tenderness to palpation over the lateral epicondyle and pain with resisted wrist extension. An MRI is shown in figures A and B. After failing non-surgical treatment modalities, he undergoes arthroscopic surgical management. At 3 months post-operatively, the patient reports persistent left elbow pain and an audible clicking since surgery which occurs when he lifts heavy objects and when he pushes himself up out of a chair. What is the best surgical treatment option? Review Topic
-
Revision elbow arthroscopy with debridement of the extensor carpi radials brevis
-
Primary repair of the medial collateral ligament
-
Primary repair of the lateral ulnar collateral ligament
-
Palmaris longus or gracilis allograft reconstruction of the lateral ulnar collateral ligament
-
Palmaris longus or gracilis allograft reconstruction of the medial collateral ligament
PREFERRED RESPONSE 4
The patient presents with lateral epicondylitis and develops posterolateral rotatory instability (PLRI) of the elbow due to excessive arthroscopic debridement. The correct answer is reconstruction of the lateral ulnar collateral ligament using palmaris longus or gracilis allograft.
PLRI is the result of an incompetent lateral ulnar collateral ligament (LUCL), a component of the elbow lateral collateral ligament complex. The LUCL originates on the lateral epicondyle of the humerus and inserts upon the supinator crest of the ulna. When deficient from acute trauma or from repetitive microtrauma, the elbow becomes rotationally unstable with elbow extension, supination, and an applied valgus force. In this case, the patient has had iatrogenic damage to the LUCL from an arthroscopic release of the extensor carpi radialis brevis (ECRB) for treatment of lateral epicondylitis. This patient exhibits an important manifestation of this: a positive chair pushup test. This test is positive when pushing off of a chair with a supinated forearm causes pain and instability. Due to the chronicity of the injury (3 months) and his persistent symptoms of instability (pain and clicking) the patient would benefit from surgical reconstruction of the damaged LUCL using either palmaris longus or gracilis allograft.
Kelly et. al. reviewed the known major and minor complications of elbow arthroscopy among 473 consecutive cases at their institution from 1980-1998. The most common complications were transient nerve palsies in 10 patients. Among them, the major
nerves involved included the anterior interosseous nerve, posterior interosseous nerve, ulnar nerve, superficial radial nerve, and medial antebrachial cutaneous nerve. The risk of iatrogenic nerve injury was increased among patients with rheumatoid arthritis. The most frequent complication was prolonged drainage from the portal sites.
Calfee et. al. reviewed the management of lateral epicondylitis. The authors suggest open or arthroscopic surgical debridement of the common extensor origin after failure of rest, orthoses, nonsteroidal drugs, physical therapy, cortisone and platelet-rich plasma injections. They do acknowledge that excessive debridement may compromise lateral elbow stability and cause PLRI.
O'Brien et. al. described the surgical techniques for managing PLRI, including an open technique for chronic injuries or revision treatment. In this setting, the authors suggest use of palmaris or gracilis allograft for reconstruction.
Figures A and B are an axial and coronal T2 weighted MRI of an elbow demonstrating signal intensity in the origin of the ECRB, consistent with lateral epicondylitis.
Incorrect Answers:
Answer 1: Further debridement of the ECRB origin is not indicated as the problem is PLRI from excessive debridement. Answer 2, 5: Lateral elbow pain with clicking and a positive chair pushup test suggest PLRI, nota a medial injury. Medial collateral injury would be suggested in the setting of a positive milking maneuver, potentially in a throwing athlete. Answer 3: Due to the chronicity of the injury primary repair is not indicated.
(SAE08OS.139) What is the primary sign/symptom with unidirectional posterior instability of the shoulder? Review Topic
-
Instability
-
Dead arm symptoms
-
Locking or catching
-
Decreased range of motion
-
Pain
PREFERRED RESPONSE 5
Posterior labral tears and pathology are usually associated with recurrent chronic injury to the shoulder. Situations such as blocking in football load the humeral head posteriorly, and may predispose to posterior labral injury. Unlike anterior and superior labral pathology, symptoms of posterior labral tears are often vague, though pain with activity is most common. Instability is usually associated with anterior labral pathology. Dead arm symptoms are associated with anterior and superior pathology, especially in throwers. Mechanical locking and catching are less common for any labral pathology. Range of motion with posterior labral pathology is usually unaffected.
(SAE07SM.21) A 62-year-old man with a long history of right shoulder pain and weakness is scheduled to undergo hemiarthroplasty. Based on the radiographs shown in Figures 6a through 6c, what preoperative factor will most affect postoperative functional outcome? Review Topic
-
Humeral head erosion
-
Glenoid erosion
-
Rotator cuff integrity
-
Status of the coracoacromial ligament
-
Acromioclavicular arthritis
PREFERRED RESPONSE 3
The radiographs reveal osteoarthritis and proximal humeral head migration. Integrity of the rotator cuff must be questioned based on these radiographic changes. The status of the rotator cuff is the most influential factor affecting postoperative function in shoulder hemiarthroplasty. The coracoacromial ligament provides a barrier to humeral head proximal migration in the face of a rotator cuff tear. The radiographs do not indicate significant humeral head or glenoid erosion. Acromioclavicular arthritis is often asymptomatic.
(SAE08UE.8) A patient reports persistent anterior shoulder pain following a forceful external rotation injury to the shoulder. An MRI scan is shown in Figure 4. The patient remains symptomatic despite 3 months of nonsurgical management. Treatment should now consist of Review Topic
-
repair of the superior labrum.
-
isolated supraspinatus repair.
-
biceps recentering.
-
subscapularis repair and biceps tenodesis.
-
subscapularis repair and recentering of the biceps tendon.
PREFERRED RESPONSE 4
The MRI scan reveals a subscapularis tear with a biceps that is out of the groove. Treatment in this patient is most predictable if the subscapularis is repaired. The biceps should either be tenodesed or tenotomized since it is unstable. Recentering of the biceps has been found to be unpredictable. Treatment of these lesions has been shown to have better results if the biceps is either released or tenodesed. This prevents recurrent biceps symptoms that can be source of surgical failure.
(SAE08AN.5) Figure 4a shows the radiograph of a 20-year-old man who has an injury to the right shoulder. Figure 4b shows an arthroscopic view (posterior portal). The arrow points to a Review Topic
-
rotator cuff tear.
-
bare area.
-
Hill-Sachs defect.
-
Bankart tear.
-
glenoid fracture.
PREFERRED RESPONSE 3
The radiograph shows an anterior dislocation of the shoulder. A frequently encountered sequela of this is a compression fracture of the posterolateral humeral head, commonly referred to as a Hill-Sachs defect. The arthroscopic view of the glenohumeral joint visualizes the posterior aspect of the humeral head. In the image, the area devoid of cartilage to the right is the bare area. The indentation seen to the left is a Hill-Sachs defect.
(SAE08UE.36) A cord-like middle glenohumeral ligament and absent anterosuperior labrum complex can be a normal anatomic capsulolabral variant. If this normal variation is repaired during arthroscopy, it will cause Review Topic
-
anterior translation of the humeral head.
-
loss of external rotation.
-
excessive tightening of the biceps tendon.
-
superior migration of the humeral head.
-
no excessive changes
PREFERRED RESPONSE 2
If the Buford complex is mistakenly reattached to the neck of the glenoid, severe painful restriction of external rotation will occur.
(SAE08UE.119) A 61-year-old woman with a long-standing history of rheumatoid arthritis reports progressive elbow pain for the past 12 months. She denies any recent trauma to the elbow; however, she notes increasing pain and decreased joint motion that are now compromising her function. Radiographs are shown in Figures 57a and 57b. What is the most appropriate treatment at this time? Review Topic
-
Physical therapy for restoration of motion
-
Elbow arthroscopy, removal of loose bodies, excision of osteophytes, and capsular release (osteocapsulectomy)
-
Elbow arthroscopy and synovectomy
-
Constrained total elbow arthroplasty
-
Semiconstrained total elbow arthroplasty
PREFERRED RESPONSE 5
The patient has end-stage arthritis of the elbow with advanced joint destruction. At this point, nonsurgical management is unlikely to provide much relief of symptoms. Arthroscopic procedures can provide relief, but it is likely to be incomplete and unpredictable. The most reliable surgical option is total elbow arthroplasty. Currently, semiconstrained components are generally preferred because constrained components have been associated with a high rate of early prosthetic loosening.
(OBQ13.243) A 47-year-old landscaper presents with worsening left shoulder pain and weakness. Three years ago, he injured the left shoulder in a fall and elected for nonoperative management to minimize time off from work. Physical therapy was effective until 6 months ago when his shoulder function worsened to the point that he is now unable to work. Examination of his active range of motion reveals forward elevation 120° with pain, abduction 100°, IR at neutral to T8 and ER at neutral 5°. He has a positive ER lag sign and Hornblower's sign. Belly press and lift-off tests are normal. A recent radiograph is shown in Figures A. MRI images are shown in Figures B and C. Which of the following is the best treatment option? Review Topic
-
Continue physical therapy
-
Latissimus dorsi transfer
-
Arthroscopic rotator cuff repair
-
Pectoralis major transfer
-
Reverse total shoulder arthroplasty
PREFERRED RESPONSE 2
This patient has a chronic massive posterosuperior rotator cuff tear with marked atrophy, tendon retraction and loss of external rotation strength that is impacting his daily life. The best treatment option for this middle-aged laborer with an irreparable posterosuperior rotator cuff tear is a latissimus dorsi transfer to restore external rotation strength and motion.
Irreparable rotator cuff tears are marked by: (1) Superior displacement of the humeral head (AHI < 5-7mm), (2) Fatty infiltration of the rotator cuff muscles (Goutallier stage 3-4), (3) Increased duration of the tendon tear and (4) Profound external rotation weakness. These findings are predictive of poor-quality tissue and stiffness of the muscle-tendon unit, not amenable to primary repair. In this setting, a latissimus dorsi
transfer can be utilized to restore shoulder strength, function and improve pain. Relative contraindications include subscapularis deficiency, deltoid deficiency, pseudoparalysis of the shoulder and advanced glenohumeral arthritis.
Gerber et al. performed a case series analysis of 67 patients with irreparable rotator cuff tears managed with latissimus dorsi transfer. Patients with an intact subscapularis demonstrated improvement in pain, range of motion and strength postoperatively, while no improvement was noted in patients with subscapularis deficiency. The authors conclude that latissimus dorsi transfer should not be performed in the setting of poor subscapularis function.
Iannotti et al. found that better clinical results following latissimus dorsi transfer were associated with: preserved active shoulder range of motion and strength (specifically forward elevation > 90° and external rotation > 20°), synchronous firing of the transferred latissimus dorsi muscle and male gender.
Figure A is an AP radiograph of the left shoulder with superior migration of the humeral head (AHI < 5mm) and no evidence of glenohumeral arthritis. Figures B and C show a retracted posterosuperior rotator cuff tear and Goutallier stage 4 atrophy (more fat than muscle) of the supraspinatus, infraspinatus and teres minor, rendering this tear irreparable. Illustration A shows a latissimus dorsi transfer. The latissimus dorsi tendon is positioned over the top of the humeral head, covering most of the rotator cuff defect. The tendon is then secured to the subscapularis tendon edge and lesser tuberosity anteriorly, the remnant supraspinatus and infraspinatus tendons medially, and the greater tuberosity laterally.
Incorrect Answers:
Answer 1: This patient has failed conservative management. His persistent pain, weakness and limited ROM is not likely to improve with continued physical therapy. Answer 3: Rotator cuff repair is unlikely to be successful in this patient as discussed above.
Answer 4: Pectoralis major transfer is indicated in chronic subscapularis tears. This patient has an intact subscapularis, indicated by the normal belly press and lift-off tests on exam. Answer 5: Reverse total shoulder is reserved for elderly low-demand patients with cuff tear atrophy or massive rotator cuff tears with pseudoparalysis of the shoulder. This patient is young and active, with preserved forward elevation and no glenohumeral arthritis.
(SAE11OS.111) Figures 111a and 111b show axial MRI scans of a 24-year-old man who injured his right shoulder several years ago and now reports continued difficulty with the shoulder and has pain with activity. He reports that when the injury occurred, he felt that his shoulder "popped" but he never required closed reduction. He wore a
sling for about 6 weeks and went through several months of physical therapy. Which of the following activities is most likely to cause him pain? Review Topic
-
Reaching back to hit a forehand in tennis
-
External rotating the shoulder to spike a volleyball
-
Performing a bench press with large amounts of weight
-
Performing a biceps curl with large amounts of weights
-
Throwing a baseball at the point of late cocking/early acceleration
PREFERRED RESPONSE 3
Performing a bench press with large amounts of weight is most likely to cause pain for a patient with a posterior labral tear. A patient who sustains a first-time posterior dislocation is less likely to have recurrent dislocations compared with first-time anterior dislocations. Patients often do have problems with loading the shoulder in a forward flexed position, such as during a bench press. The other activities listed here might be difficult, but are not as likely to be problematic. A biceps curl might bother a person with a SLAP tear. The late cocking/early acceleration phase of throwing, the overhead portion of a tennis serve, and spiking a volleyball places the shoulder in an abduction/external rotation position, which is likely to be problematic for a person with anterior instability.
(SAE07SM.23) Figure 7 shows the CT scan of a 22-year-old professional baseball pitcher who has had elbow pain for the past 6 months despite rest from throwing. Management should consist of Review Topic
-
cast immobilization for 6 weeks.
-
brief immobilization followed by rest for 6 weeks.
-
internal fixation with a compression screw.
-
internal fixation with a tension band wire.
-
bone stimulation.
PREFERRED RESPONSE 3
The CT scan shows a stress fracture of the olecranon. This injury is the result of repetitive abutment of the olecranon into the olecranon fossa, traction from triceps activity during the deceleration phase of the throwing motion, and impaction of the medial olecranon onto the olecranon fossa from valgus forces. Fractures may be either transverse or oblique in orientation. Initial treatment consists of rest and temporary splinting. Electrical bone stimulation may also be considered. Open fixation with a large compression screw is recommended when nonsurgical management has failed to provide relief.
(SAE10SM.37) A 45-year-old coach sustains a complete distal biceps tendon rupture at the elbow. Surgical repair is most indicated to Review Topic
-
restore full supination strength.
-
restore full elbow flexion strength.
-
restore full range of motion.
-
improve cosmesis.
-
prevent degenerative changes of the elbow.
PREFERRED RESPONSE 1
The biceps is primarily responsible for supination of the forearm. The brachialis muscle is primarily repsonsible for elbow flexion strength. Failure to repair the distal biceps tendon will result in loss of 40% supination strength and 10% loss in flexion strength. Therefore, surgical repair of a complete distal biceps tendon rupture is most indicated to maximize supination strength. Improved cosmesis should not be the primary indication for surgical repair. Degenerative changes of the elbow have no bearing on whether the distal biceps is repaired or not. Loss of terminal extension is common in distal biceps tendon repairs.
(SAE10SM.20) A 23-year-old right-hand dominant professional baseball pitcher has right shoulder pain when releasing the ball. He has noticed his velocity has decreased over the past 2 months. Examination reveals supine abducted external rotation of 110 degrees compared to 100 degrees on the left side. His internal rotation is 30 degrees on the right compared to 70 degrees on the left side. Rotator cuff strength is normal. All other clinical tests are normal. MRI with contrast reveals no intra-articular lesions. What is the best course of treatment? Review Topic
-
Arthroscopic capsular plication
-
Arthroscopic thermal shift
-
Arthroscopic subacromial decompression
-
Posterior capsular stretching
-
Selective external rotation stretching
PREFERRED RESPONSE 4
The examination reveals that the patient has posterior capsular tightness. Surgery should not be considered until the patient has failed to respond to nonsurgical management. The internal rotation contracture (GIRD - glenohumeral internal rotation deficit) should be addressed with appropriate posterior capsular stretching. This
should then be followed by appropriate rotator cuff and scapular stabilization exercises. Only if this management fails to relieve the patient's symptoms should surgery be considered. This patient clearly does not need external rotation stretching given the fact that he has normal external rotation.
(SAE08OS.34) A 16-year-old right-hand dominant pitcher has had pain with throwing for the past 6 months but denies any history of trauma. Figures 9a and 9b show noncontrast MRI scans of the involved shoulder. What is the most likely diagnosis? Review Topic
-
Rotator cuff tear
-
Epiphyseal stress fracture
-
Bicipital tendinitis
-
Internal impingement
-
Loose body
PREFERRED RESPONSE 4
Internal impingement differs from standard, or outlet, impingement because instability of the glenohumeral joint is commonly the primary etiology. Sports with repetitive stress of the glenohumeral joint, such as swimming, volleyball, and baseball, are most often associated with this problem. Pathology identified with diagnostic imaging and arthroscopy include: posterolateral humeral head edema, undersurface partial tearing of the rotator cuff, and increased anterior capsular volume. In throwing athletes, prevention and treatment centers on directed posterior capsular stretching and dynamic strengthening of the rotator cuff musculature.
(SAE07SM.9) A 50-year-old man reports left shoulder pain and weakness after undergoing a lymph node biopsy in his neck 2 years ago. Examination reveals winging of the left scapula. Electromyography shows denervation of the trapezius. Surgical treatment for this condition involves Review Topic
-
pectoralis transfer to the medial border of the scapula.
-
pectoralis transfer to the inferior border of the scapula.
-
lateral transfer of the levator scapulae only.
-
lateral transfer of the levator scapulae and rhomboid minor and major.
-
latissimus dorsi transfer.
PREFERRED RESPONSE 4
The muscle transfer procedure most commonly performed for trapezius paralysis is the Eden-Lange procedure. Trapezius paralysis in this patient is secondary to iatrogenic injury to the spinal accessory nerve during lymph node biopsy. In this procedure, the levator scapulae and rhomboid minor and major muscles are transferred laterally. Pectoralis transfer to the inferior border of the scapula is used as a dynamic transfer for serratus anterior winging.
(SAE08AN.46) A 23-year-old man reports pain on the superior aspect of his right shoulder with repetitive overhead activities and when lying on his right side. Figure 29 shows an axial MRI scan. What is the most likely diagnosis based on the MRI findings? Review Topic
-
Osteoarthritis of the acromioclavicular joint
-
Acromioclavicular joint separation
-
Os acromiale
-
Partial-thickness rotator cuff tear
-
Superior labral tear
PREFERRED RESPONSE 3
Os acromiale represents a failure of fusion of the anterior acromial apophysis and has been reported in approximately 8% of the population. Patients with a symptomatic os acromiale often report impingement-type symptoms with pain over the superior acromion, especially with overhead activities or sleeping. When nonsurgical management is unsuccessful, surgical options include excision, open reduction and internal fixation, and arthroscopic decompression.
(OBQ15.185) A 67-year-old female presents with shoulder pain for 3 months after falling down stairs. Imaging demonstrates a large rotator cuff tear involving multiple tendons. You perform an arthroscopic rotator cuff repair and biceps tenodesis. At her 2 month follow up, she reports worsening shoulder pain and decreased range of motion. Examination reveals active forward flexion to 80°. Passive range of motion is full. There is a positive external rotation lag sign. An MRI is performed and is pictured in Figure A. Which is the best treatment for this patient? Review Topic
-
Revision repair of subscapularis
-
Revision repair of infraspinatus
-
Latissimus dorsi transfer
-
Rotator interval release
-
Total shoulder arthroplasty
PREFERRED RESPONSE 2
The next best step for this patient's failed rotator cuff repair is a revision repair of the infraspinatus.
Failed rotator cuff repair is multifactorial. Structural failure of repair is the result of both intrinsic and extrinsic factors. Intrinsic factors include advancing patient age, increasing tear size, poor tendon and muscle quality, systemic disease and smoking history. Extrinsic factors include inadequate biomechanical construct or repair configuration and overaggressive postoperative rehabilitation.
Denard et al authored a review article on revision rotator cuff repair. Indications for revision repair are persistent symptoms despite nonoperative management in whom infection and advanced degenerative changes have been ruled out. Satisfactory results have been reported following revision repair of recurrent rotator cuff tears, particularly with arthroscopic techniques. Female sex and preoperative forward flexion < 135° is associated with poorer outcomes.
Lambers Heerspink et al found that increasing age, larger tear size and additional biceps or acromioclavicular (AC) joint procedures have a negative influence on cuff integrity at follow up. Smoking, duration of symptoms, obesity and medical comorbidities were not found to influence cuff integrity in this study. Only AC procedures and workers’ compensation status were associated with worse functional outcomes.
Figure A is a coronal T2 MRI demonstrating a failed rotator cuff repair with retear. Incorrect Responses:
Answer 1: The infraspinatus is torn as evidenced by the ER lag sign and MRI. The subscapularis does not need to be repaired. Answer 3: Revision repair is the better first step. Answer 4: Rotator interval release does not address the patient's torn rotator cuff. Answer 5: MRI does not show evidence of glenohumeral degenerative changes. Furthermore, in the setting of rotator cuff pathology, reverse total shoulder arthroplasty would be preferred.
(OBQ15.26) A 76-year-old woman has longstanding right shoulder pain affecting overhead function. Figures A and B are her original radiographs. She undergoes reverse total shoulder arthroplasty (rTSA) with iliac crest bone grafting behind the baseplate and is discharged from the hospital the following day. She returns for follow-up at 2 weeks. The incisions have healed and she has minimal pain, which is improving. What is the most accurate description for the cause of failure? Review Topic
-
Central peg error
-
Insufficient inferior tilt
-
Excessive inferior tilt
-
Component dissociation
-
Glenosphere too small
PREFERRED RESPONSE 1
The metaglen (glenoid baseplate) failed because the central peg of the baseplate was not implanted in the glenoid, and the metaglen did not have a long enough central peg. This was the technical error during baseplate implantation.
Glenoid complications are the primary concern of rTSA. Historically, rTSA fail at the glenoid interface because of inadequate fixation. The Grammont-style prosthesis has a medialized center of rotation to circumvent this, but this lead to inferior scapular notching. Newer prosthesis include locking peripheral screws and 15° inferior tilt of the base plate. Further, changing the neck-shaft angle from 155 deg to 143 deg or 135 deg has reduced notching and instability, and reduced the need to for subscapularis repair.
Boileau et al. examined revision surgery for rTSA. They found that the most common cause for revision was prosthetic instability (48%). Humeral complications (loosening/derotation/fracture) were 2nd most common and infection was 3rd most common. Underestimation of humeral shortening and excessive medialization were common causes of recurrent prosthetic instability. Proximal humeral bone loss was found to be a cause for humeral loosening or derotation.
Holcomb et al. reviewed failure of the rTSA glenoid baseplate in 14 patients, which was defined as shift of the baseplate with or without screw breakage. Strategies for success include: (1) Incorporating increased inferior baseplate tilt in post-revision prosthesis-scapular neck angle (PSNA) compared to pre-failure PSNA because inferior tilt maximizes compressive force across the baseplate-glenoid interface. (2) Locking peripheral 5.0mm screws (rather than non-locking 3.5mm screws). (3) Using larger glenospheres to tighten the patulous soft tissue envelope and secure the glenoid allograft. Four patients required structural allograft (2 iliac crest, 2 femoral head).
Figures A shows Hamada 4A cuff tear arthropathy. Figure B shows severe static posterior instability of the humeral head and posterior Walch B2 erosion of the glenoid. Figure C shows the same patient treated with rTSA and iliac crest bone graft. Because of technical error, the central peg was not implanted in the native glenoid. Illustration A shows catastrophic failure of the glenoid baseplate. Illustration B shows the PSNA, with increase in the PSNA from initial rTSA to revision rTSA. Illustration C shows baseplate-glenosphere dissociation. Illustration D shows correction of technical error/too short peg with exchange to a baseplate with a longer peg. Illustration E shows and algorithm for treating unstable rTSA.
Incorrect Answers:
Answers 2 and 3: Baseplate tilt did not contribute to migration/dislocation of the baseplate in this case. Answer 4: There was no baseplate component dissociation. See Illustration C. Answer 5: A larger glenosphere is an option for instability because of a medialized humerus.
(SAE11UE.51) Figures 51a and 51b show the AP and lateral radiographs of the elbow of a 26-year-old man who fell. Closed reduction was performed in the emergency department, and management consisted of immobilization for 3 weeks prior to the initiation of motion. At 12 weeks after injury, he reports continued feelings of instability and catching in his elbow when using his arms to rise from a chair. Which of the following procedures needs to be performed, at a minimum, to reestablish stability of the elbow? Review Topic
-
Medial collateral ligament repair
-
Medial collateral ligament reconstruction
-
Hinged external fixation
-
Lateral collateral ligament repair
-
Lateral collateral ligament reconstruction
PREFERRED RESPONSE 5
The patient has chronic posterolateral instability of the elbow following dislocation. The lateral collateral ligament complex is responsible for maintaining stability of the elbow. Because of the chronicity of the injury, the ligamentous tissues are frequently attenuated and not amenable to simple repair; while the native ligament can be imbricated, reconstruction with allograft or autograft is recommended. Medial collateral ligament reconstruction or hinged external fixation is needed only if restoration of the lateral ligamentous complex does not restore elbow stability; however, these procedures are rarely required. Lateral elbow pain when rising from a chair is equivalent to a positive pivot shift test.
(SAE11UE.94) A 78-year-old woman undergoes an uneventful semiconstrained total elbow arthroplasty through a Bryan-Morrey approach. Her immediate postoperative management should include which of the following? Review Topic
-
Five days of intravenous antibiotics for perioperative prophylaxis
-
Use of continuous passive motion beginning on postoperative day one
-
Immediate initiation of active flexion and gravity-assisted passive extension
-
Splinting at 60 to 90 degrees of flexion for 5 to 10 days, followed by initiation of active flexion and gravity-assisted passive extension
-
Splinting at 60 to 90 degrees of flexion until the triceps has healed, followed by initiation of active flexion and extension
PREFERRED RESPONSE 4
Postoperative management of total elbow arthroplasty patients is directed to avoidance of complications commonly associated with this procedure. Following total elbow arthroplasty, 24 hours of perioperative antibiotics should be given, consistent with other arthroplasty procedures. Because of the relatively thin soft-tissue envelope surrounding the elbow, particularly in patients with rheumatoid arthritis, consideration must be given to the surrounding soft tissues postoperatively. The surgical wound should be given several days of quiescence prior to initiation of motion to minimize wound healing complications. Splinting at 60 to 90 degrees allows tension to be removed from the soft tissues. Immediate motion places these tissues under immediate stress; immobilization of the elbow for 6 to 8 weeks until the triceps has healed would result in significant stiffness. Splinting should not be used more than 10 days to avoid stiffness of the elbow.
(SAE08UE.56) A 72-year-old woman who is right hand-dominant has severe pain in the right shoulder that has failed to respond to nonsurgical management. She reports night pain and significant disability. Examination reveals 30 degrees of active forward elevation. An AP radiograph is shown in Figure 27. Which of the following treatment options will provide the best functional improvement? Review Topic
-
Arthroscopic debridement
-
Arthroscopic rotator cuff repair
-
Hemiarthroplasty with rotator cuff repair
-
Total shoulder arthroplasty
-
Reverse shoulder arthroplasty
PREFERRED RESPONSE 5
The patient has end-stage rotator cuff tear arthropathy. The radiograph shows complete proximal humeral migration (acromiohumeral interval of 0 mm), severe glenohumeral arthritis, and acetabularization of the acromion. In addition, she has "pseudoparalysis" with active elevation of only 30 degrees. Reverse shoulder arthroplasty affords her the best opportunity for pain relief and functional improvement. The other procedures have mixed results but typically are better for pain relief than they are for functional gains.
(SAE08UE.116) A 22-year-old female collegiate javelin thrower has shoulder pain. She notes that her pain is primarily located in the posterior aspect of her shoulder, is exacerbated with throwing, and she experiences maximal tenderness in the extreme cocking phase of the throwing cycle. On examination, she reports deep posterior shoulder pain when the arm is abducted 90 degrees and maximally externally rotated to 110 degrees. This reproduces her symptoms precisely. Shoulder radiographs are normal. What is the most likely diagnosis? Review Topic
-
Anterior shoulder instability
-
Early adhesive capsulitis
-
Internal impingement
-
Subacromial impingement
-
Full-thickness rotator cuff tear
PREFERRED RESPONSE 3
The patient has internal impingement. Internal impingement is commonly seen in overhead throwing athletes. When positioned in the extreme cocking phase of the throwing cycle, the posterior glenoid impacts the articular surface of the infraspinatus and posterior fibers of the supraspinatus tendon. This impact can cause partial-thickness rotator cuff tearing and posterosuperior labral lesions. She has no evidence of anterior shoulder instability, and her range of motion is excellent which rules out adhesive capsulitis. Subacromial impingement is identified with anterolateral shoulder pain with internal rotation in the abducted position. A full-thickness rotator cuff tear in a 22-year-old individual would require significant trauma and would likely result in pain at rest and with lifting.
(SAE08UE.26) A 60-year-old right hand-dominant women fell on her outstretched arm and sustained an anterior shoulder dislocation. The shoulder is reduced in the emergency department and she is seen for follow-up 1 week later wearing a sling. Examination reveals that she has significant difficulty raising her arm in forward elevation and has excessive external rotation compared to the contralateral shoulder. What is the next most appropriate step in management? Review Topic
-
MRI
-
Electromyography
-
Open repair of the supraspinatus
-
Arthrography
-
Arthroscopic labral repair
PREFERRED RESPONSE 1
In patients older than age 40 years, a high suspicion of a rotator cuff tear should be kept in those patients with weakness after shoulder dislocation. Both posterior rotator cuff and subscapularis injuries have been documented. The next most appropriate step
in management should be MRI. If the findings are negative, suspicion of nerve injury should lead to electromyography.
(OBQ14.131) A young, healthy male undergoes a distal biceps repair and sustains an iatrogenic nerve injury during the procedure. Which of the following clinical findings are most likely to be seen in this circumstance? Review Topic
-
Inability to extend the thumb
-
Lateral volar forearm numbness
-
Inability to flex the middle finger
-
Medial volar forearm numbness
-
Dorsal thumb numbness
PREFERRED RESPONSE 2
The most commonly injured nerve during a distal biceps repair is the lateral antebrachial cutaneous nerve (LABCN). Injury to this nerve would result in lateral volar forearm numbness.
Distal biceps avulsions can be partial or complete. Indications for surgical management include young, healthy patients who do not wish to sacrifice function, as well as partial biceps avulsions that do not respond to conservative management. Repair of a distal biceps avulsion can be approached through either an anterior one-incision technique or a two-incision technique (Boyd-Anderson). The one-incision technique uses the interval between the brachioradialis (radial nerve) and pronator teres (median nerve), while the two-incision technique uses this same interval in addition to a second posterolateral elbow incision. The lateral antebrachial cutaneous nerve is the most common nerve injured during either approach.
Kelly et al. retrospectively reviewed 74 distal biceps tendon repairs, and found five sensory nerve paresthesias. The lateral antebrachial cutaneous nerve was most commonly injured, followed by the superficial radial nerve.
Cain et al. retrospectively reviewed 198 distal biceps tendon repairs, and found a 36% complication rate. Lateral antebrachial cutaneous nerve paresthesias were found in 26%, while radial sensory nerve paresthesias were found in 6%, and posterior interosseous nerve (PIN) injury in 4%.
Illustration A shows the close relationship between the lateral antebrachial cutaneous nerve (LABCN) and the distal biceps. Illustration B shows the sensory nerves of the upper extremity and their respective areas of innervation.
Incorrect Answers:
Answer 1: Inability to extend the thumb would be the result of a posterior interosseous nerve (PIN) injury. Answer 3: Inability to flex the middle finger would be the result of a median nerve injury.
Answer 4: Medial volar forearm numbness would be the result of a medial antebrachial cutaneous nerve (MABCN) injury. Answer 5: Dorsal thumb numbness would be the result of a superficial radial nerve injury.
(SAE11OS.155) A 30-year-old accountant and recreational softball player, who is seen at the end of his baseball season, reports a several month history of pain along the medial side of his dominant elbow. He cannot identify a specific injury and notes it only hurts when he throws the ball in from the outfield. Besides the pain, he remarks that his speed and distance while throwing have diminished considerably. Examination reveals tenderness along the medial elbow but no weakness or gross instability is found. Radiographs are normal. Based on the history, what is the most likely diagnosis? Review Topic
-
Ulnar neuritis
-
Pronator syndrome
-
Medial epicondylitis
-
Medial collateral ligament sprain
-
Varus extension overload
PREFERRED RESPONSE 4
Throwing athletes frequently develop medial collateral ligament sprain related to the repeated valgus stress that occurs on the medial elbow during the acceleration phase of throwing. This has the effect of not only causing pain, but also resulting in loss of velocity and distance during the throwing activity. The injury is generally well tolerated in most activities of daily living and only becomes problematic during the vigorous, stressful act of throwing. Absence of neurologic signs or symptoms makes ulnar nerve pathology unlikely. Pronator syndrome causes pain on the volar aspect of
the forearm during resisted forearm pronation and is not associated with the throwing motion in particular. Valgus extension overload may mimic medial collateral ligament injury, not varus extension injuries. Medial epicondylitis may be confused with ligament insufficiency but the examination and a history of pain only while throwing make this an unlikely diagnosis.
(SAE11OS.130) A 45-year-old man sustained the injury seen in Figure 130a 6 weeks ago. He denies any prior injury to his shoulder. After treatment of the injury in the emergency department, he was noted to have significant weakness with empty can testing and external rotation at the side. He has full passive range of motion with forward flexion, abduction, and internal and external rotation, but has difficulty initiating abduction with his arm at his side. He has negative apprehension and relocation signs. A detailed neurologic examination shows no deficits. A coronal image from a follow-up MRI scan is seen in Figure 130b. Follow-up radiographs reveal no fractures. What is the most appropriate next step in his treatment? Review Topic
-
Coracoid transfer
-
Rotator cuff repair
-
Reverse total shoulder arthroplasty
-
Arthroscopic anteroinferior labral repair
-
Physical therapy for range of motion and strength improvements
PREFERRED RESPONSE 2
The most likely concern, in a patient older than age 40 having a first-time shoulder dislocation, is a rotator cuff tear. The MRI scan shows a tear of the supraspinatus tendon. Recurrent instability is less likely to be a problem, so an external rotation brace for an extended period of time is unnecessary. The patient already has good passive range of motion, and with a full-thickness rotator cuff tear, physical therapy alone is unlikely to return him to full function. The MRI scan shows no labral tear, so arthroscopic or open repair is not indicated.
(SAE11OS.158) A 19-year-old college pitcher reports posterior shoulder discomfort that started recently with pitching. He is able to throw with normal velocity and control, but his pain in the early acceleration phase of throwing is getting worse. Examination reveals symmetric rotator cuff strength and no increased anterior or posterior translation of either shoulder. He has some discomfort with his shoulder in abduction and external rotation. Supine range of motion of the right shoulder in 90 degrees of abduction reveals external rotation to 100 degrees and internal rotation to 25 degrees. His left shoulder has 95 degrees of external rotation and 45 degrees of internal rotation. He is not playing the next 2 weeks and requests some exercises that he can do on his own. Which of the following exercises will most likely improve his shoulder symptoms? Review Topic
-
Standard and low rowing exercises
-
Lying on his side with the shoulder abducted 90 degrees, elbow flexed 90 degrees, and pushing his forearm toward the table
-
Humeral head depressions while holding a ball against a wall
-
Scapular `punches` in many directions
-
Putting a rolled towel between his shoulder blades while lying supine and having a teammate push posteriorly on the shoulders
PREFERRED RESPONSE 2
The patient has a glenohumeral internal rotation deficit of 20 degrees. Posterior capsular stretching would be beneficial. A sleeper stretch is a common way for patients to stretch the posterior capsule on their own. It involves lying on the side with the shoulder abducted 90 degrees and the elbow flexed 90 degrees and trying to push the forearm toward the table. Closed-chain rotator cuff exercises, such as humeral head depressions while holding a ball against a wall, pectoralis minor stretching, such as lying on a rolled towel and pushing posteriorly on the shoulders, scapular
protraction, such as punches, and scapular retraction, such as row exercises, can all be helpful for the disabled throwing shoulder, but they will not restore the decreased internal rotation.
(SAE08AN.34) What is the structure indicated by the letter “A” in Figure A? Review Topic
-
Annular ligament
-
Lateral ulnar collateral ligament
-
Accessory collateral ligament
-
Radial collateral ligament
-
Transverse ligament
PREFERRED RESPONSE 4
The ligaments shown are the components of the lateral collateral ligament complex, and the structure indicated by the letter “A” is the radial collateral ligament. The lateral ulnar collateral ligament is the structure indicated by the letter “C” and the annular ligament is indicated by the letter “B.” The transverse ligament is a component of the medial collateral ligament complex.
(SAE08UE.50) A 27-year-old woman reports the acute atraumatic onset of burning pain in her right shoulder followed a week later by significant weakness and the inability to abduct her shoulder. One week prior to this incident she had recovered from a flu-like syndrome. Examination reveals full passive motion of the shoulder and the inability to actively raise the arm. Sensation in the right upper extremity is normal. Cervical spine examination is normal. Radiographs of the shoulder and cervical spine are normal. What is the most likely diagnosis? Review Topic
-
Calcific tendinitis
-
Poliomyelitis
-
Diskogenic cervical spine disease
-
Impingement
-
Brachial neuritis
PREFERRED RESPONSE 5
The patient has symptoms and examination findings of acute brachial neuritis which is often a diagnosis of exclusion. The recent viral flu-like symptoms have shown a correlation with the development of this disorder. The acute, severe shoulder weakness excludes calcific tendinitis, impingement, and poliomyelitis. A normal cervical spine examination makes cervical disk disease unlikely.
(SAE11OS.177) Figure 177 is an intra-articular photograph taken while viewing from the anterior superior portal during arthroscopy of a right shoulder. Which of the following findings identified at the time of surgery would be the most predictive for recurrence following arthroscopic repair of the demonstrated pathology? Review Topic
-
Nonengaging Hill-Sachs deformity
-
Intra-articular loose body
-
Anterior glenoid bone deficiency of 35%
-
Subacromial bursitis
-
10% partial-thickness, articular side tear of the supraspinatous
PREFERRED RESPONSE 3
Anterior glenoid bone deficiency of 35% is most predictive of recurrence. Figure 177 shows an acute tear of the anterior inferior glenoid labrum consistent with a Bankart lesion. It has been clearly shown that there is a direct relationship between failure (ie, recurrent dislocation) of arthroscopic Bankart repair and anterior glenoid bone loss. Anterior glenoid bone loss of greater than 25% in the setting of anterior glenohumeral instability is a relative contraindication to performing arthroscopic stabilization and instead is an indication to perform a bony glenoid augmentation procedure to address the articular arc deficit. Therefore, an anterior bony defect of 35% is the most predictive finding at the time of surgery for recurrent dislocation. An engaging Hill-Sachs deformity has a significant effect on the rate of redislocation, but a nonengaging one should not. An intra-articular loose body, subacromial bursitis, and a partial-thickness articular-sided supraspinatous tear should not lead to an increased risk of recurrent dislocation following Bankart repair.
(SAE11UE.110) A 72-year-old woman was evaluated with an MRI scan for a shoulder mass that was confirmed to be a lipoma. Additional MRI findings included a 7-mm full-thickness tear of the supraspinatus tendon. Therefore, the patient was
referred by her internist for evaluation and management of the rotator cuff tear. The patient reports mild "stiffness" with certain motion but denies any limitations in her functional capacity. Examination reveals a slight decrease in internal rotation and mild weakness with resisted abduction of the shoulder. What is the most appropriate management? Review Topic
-
Observation
-
Arthroscopic rotator cuff debridement
-
Arthroscopic rotator cuff repair with acromioplasty
-
Arthroscopic biceps tendon tenotomy
-
Open rotator cuff repair with bone tunnels
PREFERRED RESPONSE 1
In patients older than age 60 years, over 30% of asymptomatic shoulders show MRI findings of full-thickness rotator cuff tears. Therefore, without significant symptoms, surgical treatment is not warranted.
(SAE10SM.61) The sublime tubercle of the elbow serves as the insertion site of the Review Topic
-
anterior bundle of the medial collateral ligament.
-
posterior bundle of the medial collateral ligament.
-
transverse bundle of the medial collateral ligament.
-
annular ligament.
-
lateral collateral ligament.
PREFERRED RESPONSE 1
The anterior bundle originates on the anteroinferior medial humeral epicondyle and inserts on the medial portion of the coronoid, known as the sublime tubercle.
(OBQ14.11) Which of the following provocative tests would most likely be positive in a patient with medial epicondylitis? Review Topic
-
Resisted forearm pronation and wrist flexion with a clenched fist
-
Resisted forearm supination and wrist extension with a clenched fist
-
Dynamic valgus stress test
-
Milking maneuver
-
Pinch grip test
PREFERRED RESPONSE 1
A provocative test for medial epicondylitis can be elicited by applying resistance to a patient with their fist clenched, wrist flexed and pronated.
Medial epicondylitis is an overuse syndrome of the flexor-pronator mass. The pronator teres (PT) and flexor carpi radialis (FCR) are thought to be most affected with this condition. It is most common in the dominant arm and occurs with activities that require repetitive wrist flexion/forearm pronation. Patients are most tender over the origin of PT and FCR at the medial epicondyle. Resisting a patient with their fist clenched, wrist flexed and pronated can cause worsening of their pain. This maneuver can be used as a provocative test for this condition.
Cain et al. reviewed elbow injuries in throwing athletes. They comment that the common flexor-pronator muscle origin provides dynamic support to valgus stress in the throwing elbow, especially during early arm acceleration and help produce wrist flexion during ball release.
Amin et al. reviewed the evaluation and management of medial epicondylitis. They report that medial epicondylitis typically occurs in the fourth through sixth decades of life, the peak working years, and equally affects men and women. Physical therapy and rehabilitation is the main aspect of recovery from medial epicondylitis, once acute symptoms have been alleviated.
Illustration A shows a video of this provocative test for medial epicondylitis. Incorrect Answers:
Answer 2: Resisted forearm supination and wrist extension with a clenched fist would be used to assess for lateral epicondylitis Answer 3: Dynamic valgus stress test is used to asses for MCL elbow injuries
Answer 4: Milking maneuver is used to asses subjective apprehension, instability, or pain at the MCL origin. Answer 5: Pinch grip test is used to assess for anterior interosseous nerve (AIN) injury/palsy.
(SAE07SM.89) A 22-year-old volleyball player reports the insidious onset of superior and posterior shoulder pain. Radiographs are normal. An MRI scan is shown in Figure
25. What is the most specific physical examination finding? Review Topic
-
Positive impingement sign
-
Positive apprehension
-
Positive active compression
-
Weakness of external rotation
-
Weakness of abduction
PREFERRED RESPONSE 4
Overhead athletes are prone to a number of problems involving the shoulder. Pitchers and volleyball players are susceptible to posterior superior labral tears and internal impingement. These patients will have posterior superior shoulder pain, a positive relocation sign, and a positive active compression test. Occasionally, these posterior superior labral tears are associated with a spinoglenoid cyst as seen in the MRI scan. These cysts cause compression of the suprascapular nerve which manifests primarily as weakness of the infraspinatus, resulting in weakness of external rotation.
(SAE07SM.70) A 45-year-old tennis player undergoes surgery for chronic lateral epicondylitis. After returning to play, he notes increasing lateral elbow pain with mechanical catching and locking. Examination shows positive supine posterolateral rotatory instability. What ligament has been injured? Review Topic
-
Annular
-
Anterior band of the medial collateral
-
Lateral orbicular
-
Lateral radial collateral
-
Lateral ulnar collateral
PREFERRED RESPONSE 5
The patient has sustained an iatrogenic injury to the lateral ulnar collateral ligament. This injury has been reported after lateral approaches to the elbow. The orbicular, annular, and lateral radial collateral ligaments have a much less important role in lateral elbow stability. The anterior band of the ulnar collateral ligament is on the medial side of the elbow and is important for valgus stability.
(SAE07SM.86) A 46-year-old woman fell from her bicycle and sustained the injury shown in Figure 24. Which of the following ligaments has been disrupted? Review Topic
-
Acromioclavicular
-
Acromioclavicular and coracoclavicular
-
Coracoclavicular
-
Coracoacromial and sternoclavicular
-
Sternoclavicular
PREFERRED RESPONSE 2
The radiograph shows a type V acromioclavicular joint injury. Type V injuries involve disruption of the acromioclavicular and coracoclavicular ligaments. Type I injuries involve a sprain of the acromioclavicular joint ligaments. Type II injuries involve disruption of the acromioclavicular joint ligaments; the coracoclavicular ligaments are partially injured. Sternoclavicular ligaments stabilize the medial clavicle and the sternum; they are not damaged with acromioclavicular joint dislocations.
(SAE11UE.96) Which of the following clinical tests is used to diagnose medial instability of the elbow? Review Topic
-
Posterolateral rotatory drawer test
-
Lateral pivot-shift test
-
Moving valgus stress test
-
Chair test (apprehension or dislocation on terminal extension of the supinated forearm when rising from a seated position)
-
Pushup sign
PREFERRED RESPONSE 3
The moving valgus stress test is used in the diagnosis of medial collateral ligament instability of the elbow. The other tests apply a varus force to the elbow and are used to diagnose lateral ulnar collateral insufficiency.
(SAE11UE.81) A 36-year-old woman reports vague right shoulder pain. She denies any previous shoulder problems or any recent trauma. MRI scans are shown in Figures 81a and 81b. Weakness of which of the following is the most likely finding in her physical examination? Review Topic
-
Shoulder abduction and internal rotation
-
Shoulder external rotation and scapula protraction
-
Shoulder external rotation with the arm at the side
-
Shoulder internal rotation with the arm at the side
-
Scapula protraction
PREFERRED RESPONSE 3
The MRI scans show a cyst formation within the suprascapular notch that can compress the suprascapular nerve. The suprascapular nerve innervates both the supraspinatus and the infraspinatus muscles. Therefore, patients with compression of this nerve may demonstrate weakness of shoulder abduction and external rotation with the arm at the side. If the nerve is compressed after its innervation of the supraspinatus muscle, however, patients will demonstrate weakness of shoulder external rotation only. Suprascapular nerve does not innervate muscles that control scapula motion or shoulder internal rotation.
(SAE08UE.63) A football lineman who sustained a traumatic injury while blocking during a game now reports that his shoulder is slipping while pass blocking. Examination reveals no apprehension in abduction and external rotation; however, he reports pain with posterior translation of the shoulder. He has full strength in external rotation, internal rotation, and supraspinatus testing. What is the pathology most likely responsible for his symptoms? Review Topic
-
Anterior glenoid rim fracture tear
-
Anterior inferior labral tear
-
Posterior labral tear
-
Total capsular laxity
-
Osteochondral defect of the humeral head
PREFERRED RESPONSE 3
Traumatic posterior instability is a common finding in football players, especially in the blocking positions as well as in the defensive linemen and linebackers. A traumatic blow to the outstretched arm results in posterior glenohumeral forces. Labral detachment at the glenoid rim is common. Patients report slipping or pain with posteriorly directed pressure. Rarely do these patients have true dislocations that require reduction; however, recurrent episodes of subluxation or pain are not uncommon. Posterior repair has been shown to be successful in the treatment of traumatic instability.
(SAE11UE.67) Which of the follow scenarios is most likely to be amenable to a complete repair of a massive rotator cuff tear? Review Topic
-
42-year-old woman with rheumatoid arthritis
-
45-year-old man with a tear associated with an anterior shoulder dislocation
-
49-year-old man who underwent repair of an ipsilateral rotator cuff 3 years ago
-
56-year-old male laborer with superior humeral migration on radiographs
-
59-year-old woman with muscular atrophy noted in the supraspinatus fossa
PREFERRED RESPONSE 2
Whereas a rotator cuff tear associated with an acute anterior dislocation in 45-year old patient may be massive, its acute nature typically means that significant retraction and atrophy of the musculature has not occurred. Therefore, repair is often complete and tension-free. A massive tear associated with rheumatoid arthritis is likely one of chronic attrition with poor tendon tissue because of the underlying disease and chronic corticosteroid use. Repairs of massive chronic rotator cuff tears have been reported to have a 50% rate of retear and this rate would be expected to be higher in the revision setting and with evident supraspinatus atrophy on physical examination. Superior humeral migration on static upright radiographs indicates loss of the superior glenoid rim, leading to rotator cuff tear arthropathy.
(SAE08UE.46) What are the proposed biomechanical advantages of the Grammont reverse total shoulder arthroplasty when compared to a standard shoulder arthroplasty? Review Topic
-
Lateralization of the center of rotation, lengthening the deltoid, and decreasing the deltoid moment arm
-
Lateralization of the center of rotation, shortening the deltoid, and decreasing acromial stress
-
Lateralization of the center of rotation, lengthening the deltoid, and increasing the transverse force couple
-
Medialization of the center of rotation, lengthening the deltoid, and increasing the deltoid moment arm
-
Medialization of the center of rotation, shortening the deltoid, and decreasing acromial stress
PREFERRED RESPONSE 4
The Grammont reverse total shoulder arthroplasty is designed to medialize the center of rotation, thereby increasing the deltoid moment arm and lengthening the deltoid.
(SAE11UE.48) A 55-year-old patient with rheumatoid arthritis reports increasing elbow pain and swelling for the past 2 months. She underwent a cemented, semiconstrained elbow arthroplasty 8 years ago. Laboratory studies show a normal peripheral white blood cell count; however, the erythrocyte sedimentation rate and C-reactive protein level are elevated. Radiographs are shown in Figures 48a and 48b. Which of the following organisms is most difficult to eradicate? Review Topic
-
Streptococcus viridans
-
Staphylococcus epidermidis
-
Escherichia coli
-
Vibrio parahaemolyticus
-
Clostridium difficile
PREFERRED RESPONSE 2
The patient's history and radiographs are suspicious for a relatively aggressive infection. Staphylococcus epidermidis is difficult to eradicate because of its encapsulation. The lytic area surrounding both the ulnar and humeral components suggests that the prosthesis is also loose. This revision will require component removal, antibiotic spacer placement, and parenteral antibiotics.
(SAE11OS.100) Figures 100a and 100b are the MRI scans of a 45-year-old man who has had elbow and proximal forearm pain for the past 8 months. He can recall no specific trauma and symptoms have not lessened despite his adopting job modifications that limit lifting. He has discomfort with resisted elbow extension and pronation. The biceps tendon can be easily palpated. Treatment should consist of which of the following? Review Topic
-
Release of the lacertus and transfer of the biceps to the brachialis tendon
-
Open detachment, debridement, and reattachment of the biceps tendon
-
Anterior exploration and decompression of the posterior interosseous nerve
-
Excision of the anterior intramuscular lipoma
-
Endoscopic debridement of the biceps tendon
PREFERRED RESPONSE 2
The MRI findings are most consistent with a partial tear of the biceps tendon. In the setting of prolonged symptoms that are resistant to nonsurgical interventions like rest,
physical therapy, and modality, surgical treatment is indicated. Exploration, debridement, and reattachment with one of a variety of techniques are the standards of care. No lipomatous mass is seen on the MRI scan. There is no weakness in finger extension to suggest posterior interosseous nerve palsy. Transfer of the biceps would result in loss of supination strength. Endoscopic biceps tendon surgery is reserved for long-head pathology.
(SAE10SM.19) Which of the following clinical findings is most often seen with the MRI scan findings shown in Figures 19a through 19c? Review Topic
-
Atrophy of the lateral shoulder
-
Atrophy of the posterior shoulder
-
Sensory deficit of the lateral shoulder
-
Sensory deficit of the posterior shoulder
-
Sensory deficit of the anterior shoulder
PREFERRED RESPONSE 2
The MRI scans show a large superior labral cyst. Impingement of the cyst on the suprascapular nerve is implied by the visible atrophy of the infraspinatus muscle as seen in Figure 19c. Clinically, this is manifested by atrophy of the posterior aspect of the shoulder inferior to the scapular spine. The suprascapular nerve provides only motor function and does not provide any sensory function to the shoulder girdle; therefore, sensory deficits will not be present in this patient.
(SAE11OS.11) A 52-year-old man who dislocated his dominant shoulder has it reduced in the emergency department and he is placed in a sling. At his 5-day followup evaluation, he reports that this is his first shoulder dislocation and that the pain is mostly gone but he notes difficulty using his arm overhead and away from his body. Examination reveals minimal pain with passive range of motion, a positive apprehension and relocation test, and 3/5 strength with the empty can test and external rotation at the side compared with 5/5 with those tests on the contralateral side. Cutaneous sensation over the lateral aspect of the shoulder is intact. Radiographs show the glenohumeral joint is reduced with no fractures or degenerative changes. What is the next step in management? Review Topic
-
CT of the shoulder
-
MRI of the shoulder
-
Application of a sling for 6 weeks
-
Surgery for diagnostic shoulder arthroscopy
-
Physical therapy to work on range of motion and strengthening
PREFERRED RESPONSE 2
Obtaining an MRI scan to evaluate for a rotator cuff tear is a reasonable next step. The patient sustained a first-time shoulder dislocation, and given his age and clinical presentation, it is likely that he injured the rotator cuff. Large, full-thickness rotator
cuff tears following dislocation in young individuals warrants early surgical intervention. Delay of surgical repair for large, full-thickness tears may lead to irreversible changes, including atrophy and retraction of the tendon. As a result, clinical outcomes may be compromised. CT will demonstrate bony changes, but it is not as effective as MRI for soft-tissue pathology. While in the short term a sling for comfort might be helpful, 6 weeks of immobilization is unnecessary because recurrent instability is rarely an issue. Physical therapy can be beneficial but could potentially delay identification of an acute rotator cuff tear. In the event the MRI does not reveal a large, full-thickness rotator cuff tear, physical therapy would be an appropriate next step. There is no indication for urgent shoulder arthroscopy.
(SAE08UE.67) Figures 34a and 34b show the axial and sagittal MRI scans of a 36-year-old man who reports the insidious onset of pain in the right shoulder. What is the most appropriate description of the acromial morphology? Review Topic
-
Type I acromion
-
Type III acromion
-
Meso os acromiale
-
Meta os acromiale
-
Pre os acromiale
PREFERRED RESPONSE 3
The MRI scans reveal a meso os acromiale with edema at the site in a skeletally mature patient.
(SAE08UE.95) A 23-year-old man who is a competitive overhead athlete has shoulder pain. Based on the pathology shown in Figure 47, what treatment option would yield the highest satisfaction and return to overhead sports? Review Topic
-
Biceps tenodesis
-
Biceps tenotomy
-
Labral debridement
-
Labral repair
-
Rotator cuff debridement
PREFERRED RESPONSE 4
The patient has a classic type II SLAP tear that will respond best to arthroscopic repair. Labral debridement has been shown to lead to predictably poor results, and biceps tenodesis and tenotomy may be appropriate for an older patient who is not a competitive overhead athlete.
(SAE07SM.95) A 15-year-old wrestler sustains an abduction, hyperextension, and external rotation injury to his right shoulder. The MRI scan findings shown in Figures 27a and 27b are most consistent with Review Topic
-
an avulsion of the lesser tuberosity.
-
a midsubstance tear of the capsule.
-
a tear of the anterior inferior labrum.
-
a tear of the subscapularis.
-
a tear of the humeral insertion of the inferior glenohumeral ligament.
PREFERRED RESPONSE 5
An isolated avulsion of the lesser tuberosity occurs very rarely and usually is found in 12- and 13-year-old adolescents. The MRI scans reveal a tear of the humeral attachment of the inferior glenohumeral ligament, a so-called HAGL lesion. This injury to the inferior glenohumeral ligament occurs much less commonly than the classic Bankart lesion (anterior inferior labral tear). A tear of the subscapularis occurs with a similar mechanism of injury but generally occurs in older individuals.
(SAE11UE.75) A 17-year-old quarterback reports shoulder pain localized over the anterior aspect of the shoulder that occurs during the follow through phase of throwing. The pain worsens toward the end of the game, but becomes asymptomatic the next day. He denies any pain during the cocking phase of throwing or during normal daily activities. Examination reveals a negative relocation test and a negative posterior load and shift test. Motion of the shoulder is normal. An MRI arthrogram is shown in Figure 75. Based on the history, examination, and MRI findings, what initial treatment should be recommended? Review Topic
-
Labrum repair
-
Capsular release
-
Labrum debridement
-
Physical therapy emphasizing a throwing program
-
Physical therapy emphasizing an internal rotation stretching program
PREFERRED RESPONSE 4
The MRI scan shows a small amount of contrast between the posterior labrum and the glenoid, suggesting a posterior labral tear. The patient's symptoms are more consistent, however, with rotator cuff deconditioning because of the timing of his pain during the throwing motion and increased severity at the end of the game. Treatment should focus on reconditioning of the rotator cuff and scapular stabilizers, combined with a return to throw program. Posterior labral tears are often found on MRI scans of asymptomatic throwers, and therefore, should not be considered the primary cause of a patient's symptoms unless it is supported by the history and physical examination. Internal rotation contractures can cause a similar pain pattern, but this patient has full and equal range of motion.
(SAE11UE.72) Following total elbow arthroplasty, patients should be instructed to Review Topic
-
return to impact activities such as golf or tennis.
-
permanently limit the load bearing of that arm to 5 pounds or less.
-
aggressively strengthen the triceps immediately following surgery.
-
immobilize the wrist and hand for 4 weeks postoperatively to minimize stress on the surgical site.
-
avoid pronation and supination to reduce torsional stress on the implant.
PREFERRED RESPONSE 2
Current recommendations are for a lifetime restriction of load bearing and avoidance of impact activities following total elbow arthroplasty (TEA). TEA is a very effective procedure in reducing pain or reconstructing previously unreconstructable fractures. However, its usage must be tempered with the limitations of currently available prostheses. Aggressive triceps strengthening must be delayed following TEA to allow healing of the triceps attachment, regardless of the surgical approach. Wrist and hand mobilization should begin immediately postoperatively to prevent stiffness. Pronation and supination should not stress a humeral ulnar arthroplasty.
(SAE08UE.1) A 68-year-old man had a 3-year history of shoulder pain that failed to respond to nonsurgical management. Examination reveals forward elevation to 120 degrees and external rotation to 30 degrees. True AP and axillary radiographs and an axial CT scan are shown in Figures 1a through 1c. What management option would lead to the best long-term results? Review Topic
-
Hemiarthroplasty
-
Total shoulder arthroplasty
-
Reverse total shoulder arthroplasty
-
Arthroscopic debridement
-
Glenoid osteotomy and interposition arthroplasty
PREFERRED RESPONSE 2
The radiographs and CT scan reveal osteoarthritis with posterior subluxation and posterior bone loss. Total shoulder arthroplasty with reaming of the high side to neutralize the glenoid surface has been shown to yield better results than hemiarthroplasty. The amount of bone loss in this patient does not require posterior glenoid augmentation. Reverse total shoulder arthroplasty is indicated for rotator cuff tear arthropathy; therefore, it is not applicable. Arthroscopic debridement has yielded poor results with advanced osteoarthritis and posterior subluxation. Results from glenoid osteotomy have been variable and glenoid osteotomy is not indicated with associated osteoarthritis.
(SAE10SM.89) A 12-year-old gymnast has had elbow pain for 4 weeks. She denies any specific trauma to the elbow. Examination reveals lateral pain and no instability on testing. Range of motion is as follows: 15 degrees, loss of elbow extension, normal flexion, and normal pronation and supination. Radiographs reveal a 3- x 7-mm radiolucency of the capitellum. A T1-weighted MRI scan reveals a single solitary lesion, and T2-weighted images show no signal around the lesion. There are no intra-articular loose bodies. Appropriate management should include which of the following? Review Topic
-
Arthroscopic debridement of the elbow
-
Open repair of the lesion
-
Open biopsy of the lesion
-
Continued participation in gymnastics until symptoms worsen
-
No participation in gymnastics until symptoms resolve and motion recovers
PREFERRED RESPONSE 5
This is a typical presentation for an osteochondral lesion of the capitellum. This patient is young and has, by definition, a stable lesion and has excellent potential to heal this lesion with nonsurgical management. However, the patient should stop her activities (gymnastics) to prevent further damage and the possible development of an unstable lesion that might then necessitate surgery. Surgical procedures are generally not necessary for the treatment of these lesions.
(SAE08AN.74) Figure 43 shows an arthroscopic view of a right shoulder through a lateral portal in the beach chair position. The arrow is pointing to what structure? Review Topic
-
Biceps tendon
-
Coracohumeral ligament
-
Superior glenohumeral ligament
-
Middle glenohumeral ligament
-
Inferior glenohumeral ligament
PREFERRED RESPONSE 1
This view from the lateral portal shows a full-thickness rotator cuff tear. The glenohumeral joint can be visualized through this tear. The glenoid, labrum, and biceps tendon attaching to the superior aspect of the glenoid are easily viewed from this portal, and the arrow is pointing to the biceps tendon. Arthroscopic rotator cuff repair can be performed while visualizing from this portal and using anterior and posterior working portals.
(SAE08UE.30) A 38-year-old left hand-dominant bodybuilder reports ecchymosis in the left axilla and anterior brachium after sustaining an injury while bench pressing 3 weeks ago. Coronal and axial MRI scans are shown in Figures 16a and 16b. What treatment method yields the best long-term results? Review Topic
-
Physical therapy and nonsteroidal anti-inflammatory drugs
-
Local corticosteroid injection and physical therapy
-
Open repair of the long head of the biceps
-
Open repair of the sternocostal portion of the pectoralis major tendon
-
Open repair of the clavicular portion of the pectoralis major tendon
PREFERRED RESPONSE 4
The MRI scans show a rupture of the sternocostal portion of the pectoralis major tendon. This is the most common site of rupture and bench pressing is the most common etiology. Surgical repair yields better functional outcomes and patient satisfaction for tears not only at the tendon/bone interface but also at the myotendinous junction.
(SAE08OS.184) Which of the following best describes the recommended treatment for a 13-year-old pitcher with a painful chronic stress injury to the proximal humeral physis as confirmed on an MRI scan? Review Topic
-
Brief shoulder immobilization followed by avoidance of pitching and throwing for the remainder of the season
-
Brief shoulder immobilization followed by avoidance of pitching and throwing until skeletal maturity
-
In situ screw fixation of the proximal humeral growth plate
-
Arthroscopic shoulder debridement
-
Arthroscopic repair of the Bankart lesion
PREFERRED RESPONSE 1
Little Leaguer's shoulder is a chronic stress injury to the proximal humerus growth plate. Imaging findings demonstrate widening of the proximal humeral growth plate. Treatment consists of rest and avoidance of pitching for the remainder of the season. Surgery is not indicated.
(SAE08UE.114) Which of the following characteristics is seen in patients with osteochondritis dissecans of the elbow? Review Topic
-
MRI reveals separation of cartilage from the capitellum and chondral fissuring
-
Fragmentation of the entire capitellar ossific nucleus
-
Normal capitellar regrowth with no late sequelae
-
Age younger than 10 years
-
Medial ulnar collateral ligament laxity
PREFERRED RESPONSE 1
Osteochondritis dissecans occurs in the older child or adolescent (typically older than age 13 years). It involves the lateral compartment. The etiology is felt to be microtraumatic vascular insufficiency from repetitive rotatory and compressive forces. MRI typically shows separation of cartilage from the capitellum and chondral fissuring. Panner’s disease is usually seen in children younger than age 10 years, involves the entire capitellar ossific nucleus, and resolves typically with no residual deformity or late sequelae. There is no evidence of ligamentous injury.
(SAE11UE.121) A 22-year-old man sustained a shoulder dislocation while playing collegiate football at age 18. Since that time, he has dislocated the shoulder three more times despite physical therapy. His last dislocation occurred 4 weeks ago while sleeping. What is the most appropriate management for this patient? Review Topic
-
Corticosteroid injection
-
Changing the physical therapist to an athletic trainer
-
A 1-month trial of nonsteroidal anti-inflammatory drugs (NSAIDs)
-
Shoulder immobilization for 6 weeks
-
A discussion regarding surgical stabilization procedures
PREFERRED RESPONSE 5
The patient sustained a traumatic shoulder dislocation at age 18 that has subsequently failed to respond to nonsurgical management. Discussion of surgical stabilization procedures is warranted at this time. A corticosteroid injection or a trial of NSAIDs will not provide any stabilizing effect. Further immobilization in this patient population has not been shown to improve stability.
(SAE11UE.13) One week ago a 25-year-old man slipped on the ice and fell, catching himself on a railing. He sustained an anterior shoulder dislocation that was subsequently reduced without difficulty in the emergency department, and he was discharged in a sling. He is now back for follow-up and reports no pain. Examination reveals no weakness on external rotation strength testing. What is the most appropriate management for this patient? Review Topic
-
Arthroscopic Bankart repair
-
MRI for possible rotator cuff tear
-
Physical therapy
-
Sling immobilization for an additional 2 weeks
-
Cortisone injection
PREFERRED RESPONSE 4
On the basis of the patient's age, lack of weakness, and the fact that this is a first-time traumatic shoulder dislocation, he is unlikely to have sustained a rotator cuff tear. Immobilization should be continued for 2 more weeks. Scheduling a surgical stabilization procedure at this time is not indicated. Immediate therapy is contraindicated because of the acuity of the injury. A cortisone injection is not indicated in an acute traumatic shoulder dislocation.
(SAE08UE.89) A 40-year-old right hand-dominant construction worker has had a 6-month history of aching left shoulder pain that is worse after working a long day. Examination reveals limited range of motion and good strength when compared to his asymptomatic right arm. He has not had any orthopaedic intervention to date. Radiographs are shown in Figures 43a and 43b. What is the most appropriate treatment? Review Topic
-
Nonsteroidal anti-inflammatory drugs, cortisone injection, and physical therapy
-
Total shoulder arthroplasty
-
Shoulder fusion
-
Arthroscopic debridement and capsular release
-
Humeral head resurface arthroplasty
PREFERRED RESPONSE 1
The patient is a young laborer with osteoarthritis. Initial treatment should begin with nonsurgical management that may include anti-inflammatory drugs, cortisone injections, and physical therapy to diminish pain and improve motion. The other choices may eventually be necessary but should only follow a course of nonsurgical management.
(SAE10SM.81) Histologic studies of surgically resected tissue in lateral epicondylitis demonstrate which of the following findings? Review Topic
-
Chondroblastic proliferation
-
Angiofibroblastic tendinosis
-
Significant active inflammation
-
Primarily calcium deposition
-
No normal tendon histology
PREFERRED RESPONSE 2
The extensor carpi radialis brevis is most often cited as the anatomic location of pathology in lateral epicondylitis. Histologic examination demonstrates noninflammatory tissue, primarily angiofibroblastic tendinosis though normal tendon histology is also present. There is usually no evidence of acute inflammation or chondroblastic tissue, or significant calcium deposition.
(SAE07SM.56) A professional pitcher reports pain localized to the medial aspect of his throwing elbow. History reveals that he was pitching in a playoff game and heard and felt a pop in his elbow. MRI reveals a complete ulnar-sided avulsion of the medial collateral ligament (MCL). Examination reveals valgus instability and ulnar nerve involvement. What recommendations should be made based on the patient’s desire to return to sport? Review Topic
-
Surgical reconstruction
-
Rest, followed by physical therapy
-
Splinting in 15 degrees of flexion
-
Primary repair
-
Arthroscopic debridement, followed by bracing in full extension for 4 weeks
PREFERRED RESPONSE 1
Injuries to the MCL usually result from repetitive high valgus stress on the medial aspect of the elbow joint due to overhead throwing or racquet sports. Excessive
stresses during the late cocking and acceleration phase of throwing can injure the anterior band of the MCL. Clinically, the injuries may present as chronic or acute, and a pop may be noted in the latter. Associated ulnar nerve involvement is common. Valgus instability is present in about 25% of patients. Patients typically are athletes who participate in throwing and have localized medial elbow pain and tenderness along the course of a ligament that extends from the medial epicondyle of the distal humerus to the sublime tubercle of the ulna. Surgical reconstruction is the procedure of choice in an athlete desiring a return to a high level of throwing.
(OBQ15.242) A 41-year-old male truck driver fell off his truck and tried to break his fall by holding on to the side of the door with his left arm. His MRI is depicted in Figures A. Which of the following special tests would most likely be positive on physical examination? Review Topic
-
Empty can test
-
Hornblower's test
-
Belly press test
-
External rotation lag sign
-
Relocation test
PREFERRED RESPONSE 3
Based on this patient's MRI he has sustained a full-thickness tear of his subscapularis tendon. As a result, he will likely have a positive belly press test on physical exam.
While the subscapularis is the largest of the rotator cuff muscles, the relative
prevalence of injuries to the subscapularis tendon has only recently been recognized. The primary function of the subscapularis is to internally rotate the humerus. Patients with such injury often present with anterior shoulder pain, and increased external rotation compared to the contralateral limb. It is often associated with medial subluxation of the long head of biceps. A number of special tests have been developed to help aid in the clinical diagnosis of this injury including the belly press, lift off and bear hug tests.
Gerber et al. demonstrated the efficacy of a simple clinical maneuver called the ‘lift-off test’ to reliably diagnose or exclude clinically relevant rupture of the subscapularis tendon in 16 patients.
Barth et al. evaluated the diagnostic value of three clinical tests commonly used to diagnose subscapularis tendon tears; the lift-off test, belly-press test, and bear-hug test. They found that the lift-off test was the most difficult for patients to perform. However, when it was performed and found to be positive, it was 74% sensitive of very severe tears. They also found that the bear hug test was the most sensitive of all tests (82%).
Figures A shows an axial MRI arthrogram showing a subscapularis tear with dislocation of the biceps tendon. Illustration A demonstrates how to perform the bear hug test.
Incorrect Answers:
Answer 1: This special test is used to diagnose a supraspinatus tendon tear. Answer 2: This special test is used to diagnose a teres minor tendon tear. Answer 4: This special test is positive in patients with an infraspinatus tendon tear. Patients with subscapularis tears have an internal rotation lag sign Answer 5: This special test is positive in patients who have anterior shoulder instability and is often performed following the apprehension test.
(SAE11OS.45) Figures 45a and 45b show sagittal T1-weighted MRI scans of a 35-year-old man who has had dominant extremity shoulder pain and weakness for the past 6 months. He denies any history of injury. Examination reveals full range of active and passive motion, negative Hawkins and Neer impingement signs, 5/5 abduction strength, 3+/5 external rotation strength with arm adducted at his side, and negative belly press, Hornblower's sign, Gerber lift-off, and O'Brien's test. Radiographs are unremarkable. An MR arthrogram shows no rotator cuff or labral tears and no paralabral cysts. What is the next most appropriate step in management? Review Topic
-
Electromyography (EMG) and nerve conduction velocity (NCV) studies of the extremity
-
MRI scan of the cervical spine
-
Corticosteroid injection of the subacromial space
-
Arthroscopic suprascapular nerve release at the suprascapular notch
-
Laboratory evaluation of C-reactive protein, erythrocyte sedimentation rate, and white blood cell count
PREFERRED RESPONSE 1
The clinical history and physical examination are suggestive of weakness of the infraspinatus. An EMG/NCV study should be obtained to determine the etiology of the atrophy. In this case, the patient was shown to have suprascapular nerve entrapment at the suprascapular notch with atrophy of the infraspinatus and early signs of denervation of the supraspinatus. An MRI scan of the cervical spine would provide information if the EMG study revealed a cervical nerve compression as the etiology of the atrophy. Arthroscopic suprascapular nerve release at the suprascapular notch is the correct treatment for the lesion; however, the EMG needs to be obtained first to determine the location of nerve compression. Laboratory evaluation of C-reactive protein, erythrocyte sedimentation rate, and white blood cell count is unnecessary because there are no signs or symptoms of an infection. Corticosteroid injection of the subacromial space would not help the current problem because there are no signs or symptoms of impingement syndrome.
(SAE08OS.200) Which of the following is considered a potential advantage of arthroscopic repair for anterior instability of the shoulder? Review Topic
-
Decreased healing time at the glenoid-labral junction
-
Completion of the procedure on an outpatient basis
-
Faster return to play than with open procedures
-
Preservation of external rotation
-
Decreased risk of recurrent instability in comparison to open repair
PREFERRED RESPONSE 4
Arthroscopic anterior labral repair spares the subscapularis, and does not require significant mobilization or incision of the anterior capsule. Therefore, it is less likely to result in significant impairment in external rotation of the glenohumeral joint when compared with traditional open stabilization procedures. Recurrent instability rates are either slightly higher or equivalent to open procedures. Both procedures can be performed on an outpatient basis and require generally identical recovery times.
(SAE11UE.7) An active 65-year-old man has pain in the left shoulder 5 years after undergoing a hemiarthroplasty. He has a remote history of two previous instability operations. Examination reveals that forward elevation is 140 degrees and external rotation is 40 degrees. Serologic studies for infection are negative. AP and axillary radiographs are shown in Figures 7a and 7b. What surgical procedure will provide the most predictable pain relief and function? Review Topic
-
Conversion to a reverse total shoulder arthroplasty
-
Conversion to a standard total shoulder arthroplasty
-
Conversion to a glenohumeral fusion
-
Resection arthroplasty
-
Biologic resurfacing of the glenoid
PREFERRED RESPONSE 2
The radiographs show glenoid arthrosis, which is common after a hemiarthroplasty. Conversion to a conventional total shoulder arthroplasty with placement of a glenoid component predictably decreases pain and improves function. There is no indication for a reverse total shoulder arthroplasty because the patient has 140 degrees of elevation with an intact rotator cuff. Biologic resurfacing has more unpredictable results and is usually reserved for younger patients in whom a prosthetic glenoid component might not be desired. Both resection arthroplasty and arthrodesis are associated with poor function.
(SAE11UE.43) A sedentary 60-year-old woman has had good elbow function and pain relief after undergoing an ulnohumeral interposition arthroplasty 10 years ago. However, she currently reports pain and stiffness for the past 6 months, and nonsurgical management has failed to provide relief. Examination reveals range of motion of 40 to 110 degrees of flexion with pain during the entire range. Radiographs are shown in Figures 43a and 43b. What is the next most appropriate step in management? Review Topic
-
Unconstrained total elbow arthroplasty
-
Revision interposition ulnohumeral arthroplasty with allograft
-
Arthroscopic osteocapsular arthroplasty
-
Outerbridge-Kashiwagi procedure
-
Semiconstrained total elbow arthroplasty
PREFERRED RESPONSE 5
The patient had a good outcome after interposition arthroplasty but has now progressed to end-stage arthritis with loss of joint space and instability. A conversion to semiconstrained total elbow arthroplasty is most reliable for pain relief and improved function. In the setting of ligamentous instability, an unconstrained prosthesis will fail. Revision interposition ulnohumeral arthroplasty and arthroscopic debridement or open debridement, as in the Outerbridge-Kashiwagi procedure, are not reliable in this age group with this amount of joint space loss.
(SAE08OS.86) A 30-year-old man sustained an acute injury to his left shoulder while lifting weights. He reports pain with abduction and external rotation of the shoulder, and he has weakness with internal rotation. Inspection shows loss of contour of the axillary fold. Definitive management should consist of Review Topic
-
rest, ice, anti-inflammatory drugs, and a gradual return to physical activity.
-
primary acute surgical repair.
-
delayed repair after failed nonsurgical management.
-
a steroid injection.
-
referral for immediate physical therapy.
PREFERRED RESPONSE 2
Pectoralis major muscle injuries occur primarily in weightlifting, football, and wrestling activities. Initial swelling, pain, and ecchymosis often make diagnosis difficult in the acute setting. Over time, chest ecchymosis, loss of axillary contour, and asymmetry of the anterior chest wall can be diagnostic. MRI can aid in the diagnosis, especially to differentiate between complete and incomplete injuries. Surgical intervention is indicated for most complete tears, especially in the younger, athletic population. Acute repair is technically easier with less surrounding scar tissue, and it minimizes the potential need for late reconstruction and possible allograft use.
(SAE07SM.50) A 27-year-old professional baseball pitcher who underwent arthroscopic olecranon debridement continues to have medial-sided elbow pain during late cocking. Physical examination reveals laxity and pain with valgus stress testing. What is the most likely cause of his pain? Review Topic
-
Ulnar neuritis
-
Excessive olecranon resection
-
Osteochondritis dissecans of the capitellum
-
Olecranon stress fracture
-
Valgus extension overload
PREFERRED RESPONSE 2
Both the medial collateral ligament and the olecranon contribute to valgus stability of the elbow. Excessive olecranon resection increases the demand placed on the medial collateral ligament in resisting valgus forces during throwing. Bone removal from the olecranon should be limited to osteophytes.
(SAE11UE.95) A 55-year-old man who underwent total shoulder arthroplasty 10 years ago recently reports an increase in shoulder pain. Laboratory studies consisting of a white blood cell count, erythrocyte sedimentation rate, and C-reactive protein are all negative, as is joint aspiration. Radiographs are shown in Figures 95a and 95b. If all intraoperative frozen sections are negative, what is the appropriate treatment during revision surgery to provide pain relief and improved function? Review Topic
-
Placement of antibiotic spacer
-
Removal of the glenoid, and possible bone grafting
-
Conversion to reverse shoulder arthroplasty
-
Referral to pain management
-
Shoulder arthrodesis
PREFERRED RESPONSE 2
The radiographs reveal a loose glenoid in the setting of no infection. Glenoid removal may give this patient the best chance of improved function and pain relief if sufficient bone stock remains. Bone grafting of defects may allow future glenoid implantation. Conversion to reverse shoulder arthroplasty would be a salvage procedure in this younger patient. Shoulder arthrodesis would be difficult and unpredictable after shoulder arthroplasty.
(SAE08OS.14) Persistent symptoms and decreased function following distal clavicle resection, coracoacromial ligament transfer, and augmentation (modified Weaver-Dunn) are most likely related to Review Topic
-
anterior-posterior translation.
-
superior-inferior translation.
3inadequate resection of the distal clavicle.
-
excessive reduction of the clavicle relative to the coracoid.
-
unrecognized instability of the sternoclavicular joint.
PREFERRED RESPONSE 1
Although multiple studies have reported good clinical results with the modified Weaver-Dunn reconstruction, others have suggested that the reconstruction does not restore the native stability to the acromioclavicular joint. In particular, persistent horizontal (anterior to posterior) instability may cause persistent symptoms following reconstruction. Anatomic repair and reconstruction techniques that preserve the distal clavicle may offer patients less risk of horizontal instability.
(SAE08OS.178) When comparing arthroscopic and open rotator cuff repairs, which of the following tears shows a decreased recurrent tear rate in the open versus the arthroscopic group? Review Topic
-
Partial-thickness tears
-
Tears less than 1 cm in width
-
Tears between 1 and 2 cm in width
-
Tears greater than 3 cm in width
-
Tears showing retraction medial to the glenoid
PREFERRED RESPONSE 4
As a tool for rotator cuff repair, arthroscopy has afforded surgeons the ability to repair tears without causing potential morbidity to the overlying deltoid. Follow-up studies looking at functional recovery have now shown equivalent or slightly better outcomes with arthroscopic procedures versus standard open procedures. However, small tear size may serve as a better predictor of success with arthroscopic approaches. Imaging studies have shown a higher rate of tear recurrence and/or failure of healing when tears greater than 3 cm in size are repaired arthroscopically versus a standard open approach.
(SAE08UE.13) Figure 7 shows a sagittal T1-weighted MRI scan. What muscle/tendon is identified by the arrow? Review Topic
-
Infraspinatus
-
Teres minor
-
Subscapularis
-
Long head of triceps
-
Latissimus dorsi
PREFERRED RESPONSE 2
The sagittal T1-weighted MRI scan is useful for interpreting the quality of muscle. The arrow is pointing to the teres minor.
(SAE11UE.57) A 22-year-old man reports that he initially dislocated his shoulder while playing basketball 2 years ago and was subsequently treated with an arthroscopic Bankart repair. Despite appropriate rehabilitation, the patient continues to report recurrent instability. An axillary view radiograph and CT scan are shown in Figures 57a and 57b. What is the most appropriate management at this time? Review Topic
-
Supervised physical therapy
-
Arthroscopic capsulorrhaphy and labral repair
-
Open shoulder capsulorrhaphy and labral repair
-
Open shoulder capsulorrhaphy and bone block
-
Shoulder arthrodesis
PREFERRED RESPONSE 4
Although the changes are subtle on the radiograph, an anterior inferior glenoid bone defect is clearly evident on the CT scan. With loss of greater than 20% to 25% of the glenoid width, patients may experience persistent instability despite appropriate labral repair and capsulorrhaphy. Therefore, nonsurgical management with supervised therapy or surgical treatments that do not address the bony defect, such as arthroscopic or open labral repair and capsulorrhaphy, are not likely to stabilize the joint. An open shoulder stabilization procedure with a bone block should address the defect and stabilize the joint. Shoulder arthrodesis is not warranted in this patient at this time because the shoulder is likely salvageable.
(SAE08UE.49) A 69-year-old woman has just undergone an uncomplicated total shoulder arthroplasty for glenohumeral osteoarthritis. A press-fit humeral stem and a cemented all-polyethylene glenoid component were placed. At this point, what is the postoperative rehabilitation plan? Review Topic
-
Maintain sling immobilization for 6 weeks, and then begin a global range-of-motion program.
-
Maintain sling immobilization for 3 weeks, and then begin a global range-of-motion program.
-
Immediately begin an active assisted range-of-motion program emphasizing forward elevation and external rotation to the side.
-
Immediately begin a passive range-of-motion program for forward elevation only; no external rotation is allowed for 6 weeks.
-
Immediately begin active range of motion in forward elevation and external rotation to the side with a progression to full rotator cuff strengthening in 3 weeks.
PREFERRED RESPONSE 3
The patient needs to immediately begin an active assisted range-of-motion program emphasizing forward elevation and external rotation to the side. Sling immobilization without stretching for either 3 or 6 weeks will result in severe stiffness that will compromise her ultimate range of motion. Since she has a good quality subscapularis tendon, there is no need to avoid beginning external rotation to the side. However, starting a strengthening program at 3 weeks risks tearing the subscapularis tendon repair. Active strengthening should not begin for 6 weeks postoperatively to allow the subscapularis tendon repair time to heal.
(OBQ15.272) What is the average version of the humeral head (with respect to the transepicondylar axis)? Review Topic
-
60 degrees retroversion
-
40 degrees retroversion
-
20 degrees retroversion
-
20 degrees anteversion
-
40 degrees anteversion
PREFERRED RESPONSE 3
Although there is considerable variability in humeral head retroversion among individuals, multiple anatomic studies have found mean humeral head retroversion to be approximately 20 degrees.
One of the goals of primary anatomic total shoulder arthroplasty (TSA) is recreation and reconstruction of proximal humeral anatomy. Modular prostheses have evolved to provide surgeons with better capability to recreate proximal humeral morphology based on humeral head inclination, retroversion, offset, height and size. In terms of size, humeral head thickness has been found in cadaver studies to be 70% of its radius of curvature. This can be helpful to avoid 'over-stuffing' the joint or leaving it too loose.
Boileau and Walch took digitized measurements of 65 humeri in order to create a computer model for proximal humeral morphology. They found that retroversion varied from -6.7 to 47.5 degrees, with a mean of 17.9. They advocate for prosthetic adaptability to recreate proximal humeral anatomy in a way that earlier generations of more geometrically constrained TSA implants could not.
Robertson et al. made 3D computed tomographic models of 60 humeri (30 pairs) to study proximal humeral morphology. They found mean retroversion to be 19 degrees, with a range of 9 to 31 degrees. They found that proximal canal version was similar to head version but that canal version in the middle and distal sections of the canal was variable.
Illustration A shows key proximal humeral morphologic parameters found by Robertson et al. in comparison with earlier studies (including Boileau's).
Incorrect Answers:
Answers 1 and 2. Although some native humeri may have significant retroversion consistent with these numbers, on average, retroversion is not this high. Answers 4 and 5. The average humeral head is retroverted.
(SAE08AN.26) A 25-year-old tennis player has shoulder pain and weakness to external rotation. MRI scans are shown in Figures 16a and 16b. What is the most likely cause of his weakness? Review Topic
-
Supraspinatus tear
-
Infraspinatus tear
-
Suprascapular nerve compression
-
C5 radiculopathy
-
Subacromial impingement
PREFERRED RESPONSE 3
The MRI scans show a paralabral cyst, which is most commonly associated with labral tears. Compression of the suprascapular nerve results in weakness of the supraspinatus and/or infraspinatus depending on the level of compression.
(OBQ05.131) A 24-year-old avid volleyball player has noted gradual onset of shoulder fatigue and weakness limiting his game. Radiographs done by his primary care physician were normal and he has failed to improve with 6 weeks of physical therapy. Given the MRI image shown in Figure A, this patients physical exam may reveal weakness with which of the following actions? Review Topic
-
Adduction
-
Internal rotation
-
Abduction and external rotation
-
Abduction
-
External rotation
PREFERRED RESPONSE 3
The MRI demonstrates of a ganglion cyst within the suprascapular notch, leading to atrophy of both the supraspinatus and infraspinatus. Thus, the patient would have weakness with both abduction and external rotation.
Extrinsic compression or traction on the suprascapular nerve can result in suprascapular neuropathy. Compression of the nerve may occur at two distinct locations: the suprascapular notch and the spinoglenoid notch. Extrinsic compression of the suprascapular nerve by ganglion cysts can occur at the spinoglenoid notch or, less commonly, at the suprascapular notch. These cysts may originate from the transverse scapular ligament, the fibrous tissue of the scapula, or the glenohumeral joint.
Mittal et al. reviewed the literature and found that the formation of ganglionic cysts in the spinoglenoid fossa occurs with cumulative trauma and leads to entrapment neuropathy of the suprascapular nerve and denervation of the infraspinatus muscle.
Romeo et al. reported on various etiologies of suprascapular neuropathy including traction injury at the level of the transverse scapular ligament or the spinoglenoid ligament and direct trauma to the nerve. They noted that sports involving overhead motion, such as tennis, swimming, and weight lifting, may result in traction injury to the suprascapular nerve, leading to dysfunction. They also reported that the onset of weakness can be subtle and must be differentiated from cervical radiculopathy and degenerative disease of the shoulder.
Figure A depicts a T2 coronal MRI of the shoulder with a cyst easily visualized occupying the suprascapular notch. Illustration A is an algorithm for the management of suprascapular neuropathy. Illustration B is a sagittal MRI from the same patient depicting the ganglion cyst within the suprascapular notch once again leading to atrophy of both the supraspinatus and infraspinatus (asterisks).
Incorrect Answers:
Answer 1&2: The suprascapular nerve does not innervate the muscles that function to adduct and internally rotate the shoulder. Answer 4&5: Because the suprascapular nerve is being compressed at the suprascapular notch, it will affect the function of both the infraspinatus and supraspinatus.
(SAE07SM.92) Figures 26a through 26c show the MRI scans of a 47-year-old man who underwent arthroscopic shoulder surgery 6 months ago and continues to have pain despite a prolonged course of rehabilitation. Management should now consist of Review Topic
-
rotator cuff repair.
-
revision acromioplasty.
-
fragment excision.
-
open reduction and internal fixation.
-
continued rehabilitation.
PREFERRED RESPONSE 4
The MRI scans show an os acromiale of the mesoacromion type. This represents an unfused acromial apophysis. Pain is thought to be caused by either motion at the site or downward displacement of the anterior aspect of the acromion onto the rotator cuff, causing impingement. Most patients can be treated nonsurgically as they are usually asymptomatic. In those patients with persistent symptoms of pain and tenderness over the acromion, surgery consisting of rigid internal fixation and bone grafting has yielded satisfactory results. Excision may be a viable treatment option for the preacromion type.
(SAE08UE.75) A 55-year-old man who works as a carpenter reports chronic right anterior shoulder pain and weakness. Examination reveals 90 degrees of external rotation (with the arm at the side) compared to 45 degrees on the left side. His lift-off examination is positive, along with a positive belly press finding. An MRI scan reveals a chronic, retracted atrophied subscapularis tendon. What is the most appropriate management of his shoulder pain and weakness? Review Topic
-
Shoulder fusion
-
Arthroscopic subscapularis repair
-
Intra-articular corticosteroid injection
-
Open subscapularis repair
-
Pectoralis major transfer
PREFERRED RESPONSE 5
Chronic subscapularis tendon ruptures preclude primary repair. In such instances, subcoracoid pectoralis major tendon transfers may improve function and diminish pain. The subcoracoid position of the transfer allows redirection of the pectoralis major in a direction recreating the vector of the subscapularis tendon. Shoulder fusion is a salvage procedure, and corticosteroid injection may reduce pain but will not improve function.
(SAE07PE.83) A 6-year-old Little League pitcher has had pain in the right elbow for the past 2 weeks. Examination reveals mild lateral elbow joint tenderness with full range of motion and no effusion or collateral laxity. A radiograph is shown in Figure
38. Initial management should consist of Review Topic
-
cessation of throwing activities.
-
a long arm cast for 3 months.
-
a corticosteroid injection into the elbow joint.
-
excision of the fragment.
-
arthroscopic drilling of the lesion.
PREFERRED RESPONSE 1
The radiograph shows osteochondritis dissecans (OCD) of the capitellum, one manifestation of “pitcher’s elbow.” The lesion is nondisplaced, and healing is possible if the inciting throwing activities are curtailed. Long arm cast treatment may be reasonable for the noncompliant patient but should not exceed 6 weeks duration. Surgical treatment is indicated for loose bodies or cartilage flaps. Elbow OCD lesions are now being seen in younger children as more participate in organized sports, especially baseball and gymnastics.
(OBQ15.51) A 29-year-old male rugby player presents for further evaluation and management of left shoulder instability. He initially dislocated his left shoulder six years ago while snowboarding. Since that time, he has sustained five dislocations requiring reduction. He has participated in multiple rounds of physical therapy without improvement. His CT scan and 3D reconstruction are pictured in Figures A and B. Which of the following is the most appropriate treatment for this patient? Review Topic
-
Open capsular shift
-
Remplissage
-
Arthroscopic Bankart repair
-
Arthroscopic Bankart repair + Remplissage
-
Open coracoid transfer
PREFERRED RESPONSE 5
This patient has recurrent anterior glenohumeral instability with >20% glenoid bone loss and therefore would benefit most from an open coracoid transfer (Latarjet procedure).
Recurrent anterior shoulder instability occurs in 33-67% of patients who sustain an initial traumatic dislocation. Specific risk factors include age < 25 years, male gender, anterior glenoid (Bankart) and/or posterior humeral (Hill-Sachs) osseous defect(s) and participation in contact sports. Surgical management of recurrent instability depends on the presence or absence of glenohumeral bone loss. Patients with < 20% glenoid bone loss can be managed with arthroscopic Bankart repair. An engaging Hill-Sachs lesion, meaning the humeral head defect engages the glenoid rim in abduction (ABD) and external rotation (ER) [see Illustration B], in the setting of minimal glenoid bone loss can be managed with remplissage. Patients with > 20% glenoid bone loss require greater stabilization, mostly commonly in the form of an open coracoid transfer (Latarjet procedure).
Burkhart et al. (2000) found arthroscopic Bankart repairs equivalent to open Bankart repairs if no substantial bone defects were present. However, patients with an “inverted pear” glenoid secondary to significant anteroinferior bone loss or an engaging Hill-Sachs lesion of the humerus had a 67% recurrence rate overall and an 89% recurrence rate if they were contact athletes. Therefore, contact athletes with structural bone deficits require open surgery and often necessitate reconstruction with bone-block procedures.
Itoi et al. determined that an osseous defect with a width of >/= 21% of the glenoid length was associated with anteroinferior instability in ABD and internal rotation (IR), as well as loss of external rotation following Bankart repair. The authors concluded that while Bankart repair in the setting of a substantial bone loss conferred adequate stability in ABD and ER, it did so at the cost of overtightening the anterior structures which limited ER and did not affect stability in IR when the anterior capsuloligamentous structures are lax.
Burkhart et al. (2007) concluded that in the setting of significant glenohumeral bone deficiency, an open Latarjet procedure had only a 4.9% recurrence rate as compared to a 67% recurrence rate following arthroscopic Bankart repair. The Latarjet procedure works to extend the bony glenoid concavity, provide a dynamic sling from the conjoint tendon, preserve the lower third of the subscapularis and repair the capsule.
Figures A and B are the axial CT scan and 3D reconstruction en face view of the glenoid demonstrating significant anteroinferior glenoid bone loss of approximately 40%. Illustration A is an algorithm for the management of anterior shoulder instability based on pertinent risk factors and glenohumeral bone loss. Illustration B compares non-engaging and engaging Hill Sachs lesions. Size and depth of the Hill-Sachs lesion as well as glenoid bone deficiency both contribute to risk of engagement. Illustration C depicts a simplistic version of the Latarjet procedure.
Incorrect Responses:
Answers 1 and 2: Open capsular shift and remplissage alone will not address the significant glenoid bone loss contributing to his recurrent instability. Answers 3 and 4: Arthroscopic Bankart repair in the presence of substantial glenohumeral bone deficits has been shown to have an unacceptably high recurrence rate.
(SAE11UE.11) Which of the following 50-year-old patients with an irreparable rotator cuff tendon is the best candidate for an isolated latissimus dorsi muscle transfer? Review Topic
-
Man with active elevation to 90 degrees
-
Woman with active elevation to 45 degrees
-
Woman with a Hornblower's sign (complete absence of external rotation with abduction)
-
Man with superior escape of the humeral head
-
Man with full motion and a positive lift-off test
PREFERRED RESPONSE 1
Patients with superior escape or a torn subscapularis (demonstrated by a positive lift-off test) will not benefit from a latissimus dorsi transfer, even if combined with a pectoralis muscle transfer. In the study by Iannotti and associates, women had poorer outcomes than men, and patients with preoperative elevation below shoulder level or 90 degrees also had poorer outcomes. Patients with complete loss of external rotator function have worse function after latissimus dorsi transfer than patients with some external rotation function.
(OBQ14.215) Reverse total shoulder arthroplasty with a latissimus dorsi transfer would be the most appropriate treatment for which of the following patients? Review Topic
-
Previous shoulder arthrodesis and complete brachial plexus injury
-
Humeral head avascular necrosis with partial thickness infraspinatus tear
-
Failed hemiarthroplasty with the inability to perform active external rotation with the arm abducted
-
Primary shoulder osteoarthritis with 10 degree of glenoid retroversion
-
Primary rotator cuff arthropathy with active forward shoulder flexion >100 degrees and external rotation >50 degrees
PREFERRED RESPONSE 3
Reverse total shoulder arthroplasty with a latissimus dorsi transfer would be most appropriate in a patient with failed shoulder hemiarthroplasty and the inability to perform active external rotation with the arm abducted
R-TSA has become the mainstay treatment for rotator cuff arthropathy. In the presence of severe loss of active elevation and external rotation, combined latisimus dorsi transfer and reverse total shoulder arthroplasty can restore elevation and external rotation, respectively. This may be used in the primary or revision setting.
Frankle et al. report the results of sixty patients with rotator cuff deficiency and glenohumeral arthritis who were followed for a minimum of two years. All were
treated with R-TSA. Their study showed that forward flexion increased from 55 to
105 degrees, and abduction increased from 41 to 102 degrees.
Boileau et al. followed 45 patients with severe cuff tear arthropathy and advanced atrophy/fatty infiltration of the infraspinatus or teres minor muscles. All patients were treated with R-TSA and a modified L'Episcopo procedure (latissimus dorsi and teres major transfer). Mean active elevation increased from 74 degrees preoperatively to
149 postoperatively, and external rotation increased from -21 to 13 degrees.
Illustrations A and B show the classic findings of rotator cuff arthropathy. There is significant acromial acetabularization and femoralization of humeral head. Other features include: asymmetric superior glenoid wear, osteopenia, "snowcap sign" due to subchondral sclerosis and anterosuperior escape. Illustration C shows a left shoulder after conversion from hemiarthroplasty to reverse total shoulder arthropathy.
Incorrect Answers:
Answer 1: Complete brachial plexus injury and shoulder arthrodesis are contraindications for reverse total shoulder arthroplasty. Answer 2: Humeral head AVN with partial thickness infraspinatus tear would most likely be treated early with conservative treatments (e.g. physiotherapy, stretching, etc). Surgical treatment would most likely involve humeral head resurfacing. Answer 4: Primary osteoarthritis of the shoulder can be treated with total shoulder arthroplasty. Reaming the anterior glenoid to neutral is a technique to be considered by the operative surgeon when presented with a patient undergoing total shoulder arthroplasty with a retroverted glenoid. Answer 5: Primary rotator cuff arthropathy with active forward shoulder flexion >100 degrees and external rotation >50 degrees can be treated primarily with a reverse total shoulder arthroplasty or hemiarthroplasty with a cuff tear arthropathy head. Active external rotation indicates functioning posterior rotator cuff muscles.
(SAE08AN.70) Figure 40 shows the MRI scan of a 23-year-old man with a history of recurrent anterior shoulder instability. What is the most likely diagnosis? Review Topic
-
Humeral avulsion of the inferior glenohumeral ligament (HAGL lesion)
-
Osseous Bankart lesion
-
Perthes lesion
-
Anterior labroligamentous periosteal sleeve avulsion (ALPSA lesion)
-
Glenolabral articular disruption (GLAD lesion)
PREFERRED RESPONSE 4
The MRI scan shows an ALPSA lesion. This is also known as a medialized Bankart with medial displacement of the torn anterior labrum. During surgical stabilization, the labrum and periosteal sleeve must be mobilized and repaired laterally to reduce recurrent instability. A Perthes lesion is a nondisplaced labral tear. A GLAD lesion represents a nondisplaced anterior labral tear with an associated articular cartilage injury.
(SAE11UE.31) A 53-year-old woman reports a 4-month history of gradual onset diffuse shoulder pain and limited function. She has had no prior treatment, and her medical history is unremarkable. Examination reveals globally painful active range of motion to 120 degrees forward elevation, 25 degrees external rotation with the arm at the side, and internal rotation to the sacrum. Passive range of motion is also limited in comparison with the contralateral shoulder. Radiographs are shown in Figures 31a through 31c. What is the most appropriate management? Review Topic
-
Sling immobilization and rest
-
Physical therapy for aggressive stretching
-
Intra-articular corticosteroid injection and stretching program
-
Manipulation of the shoulder under anesthesia
-
Arthroscopic subacromial decompression and capsular release
PREFERRED RESPONSE 3
The patient has stage II adhesive capsulitis. Patients most commonly affected are women between the ages of 40 and 60, and most cases are considered idiopathic. The preferred method of treatment is an intra-articular corticosteroid injection to decrease inflammation in the joint and allow for a gentle stretching therapy program. Sling immobilization is contraindicated because it likely will promote further joint contracture and prolonged recovery. Aggressive capsular stretching in the early stages of the disease is often counterproductive, unless pain can be adequately controlled with medication or injections. Manipulation under anesthesia and arthroscopic surgical treatment are used when symptoms remain refractory despite initial nonsurgical management.
(SAE11UE.106) A 62-year-old man has had worsening pain in the left shoulder for the past 6 weeks without trauma. He participated in physical therapy to "strengthen" his shoulder; however, it failed to provide relief. On examination, his right shoulder motion is 180, 60, and T8 (forward flexion, external rotation, and internal rotation). His left shoulder motion, both active and passive, is 150, 40, and L1. T1- and T2-weighted MRI scans are shown in Figures 106a and 106b with an official diagnosis of partial supraspinatus tendon tear. What is the appropriate treatment? Review Topic
-
Physical therapy for rotator cuff strengthening and scapula stabilization
-
Regimen of stretching exercises for motion
-
Arthroscopic acromioplasty
-
Arthroscopic acromioplasty and rotator cuff repair
-
Open rotator cuff repair
PREFERRED RESPONSE 2
The patient lacks both active and passive motion in all planes of shoulder motion; his primary pathology is adhesive capsulitis. Although the MRI scans reveal a partial-thickness rotator cuff tear, this is not uncommon in asymptomatic patients older than age 60 years. Physical therapy for patients with adhesive capulitis should stress shoulder motion rather than rotator cuff strengthening. Because most cases of adhesive capsulitis improve without surgical management, surgical treatment options are not appropriate at this time.
(SAE11UE.36) Complications following a reverse shoulder prosthesis occur most frequently when performed for what diagnosis? Review Topic
-
Rotator cuff tear arthropathy with superior escape
-
Massive rotator cuff tear with osteoarthritis
-
Fracture-dislocation of the glenohumeral joint
-
Four-part proximal humeral fractures
-
Failed shoulder arthroplasty
PREFERRED RESPONSE 5
Revision following failed shoulder arthroplasty is associated with the highest complication rates, including dislocation, loosening, and decreased function. However, when performed for rotator cuff tear arthropathy or failed rotator cuff repairs, the complication rate is reasonably low. The complication rate is unknown when the reverse total shoulder is used for fracture-dislocation or acute four-part fractures of the proximal humerus.
(OBQ15.89) A 47-year-old man comes for evaluation of his dominant right elbow, which has been bothering him with activity for the past 3 months, especially with activities requiring wrist extension. He is an active squash player and has been unable to continue this sport. Examination shows tenderness at the common extensor origin. What is the next best step? Review Topic
-
Platelet rich plasma injection
-
Eccentric exercises
-
Corticosteroid injection
-
Ultrasonic emulsification/aspiration
-
Surgical debridement and repair of the extensor carpi radialis brevis
PREFERRED RESPONSE 2
This patient has tennis elbow or lateral epicondylitis. First line treatment is conservative.
Lateral epicondylitis is a common problem with numerous non-operative treatments available. There is little scientific evidence to support any of these treatments however and the quality of most reports is low and their conclusions limited by bias and/or study design. Tennis elbow often resolves with time regardless of which conservative treatment is chosen.
Coombes et al. randomized patients with unilateral tennis elbow to receive either (1) corticosteroid injection, (2) placebo injection, (3) corticosteroid injection plus physical therapy or (4) placebo injection plus physical therapy. They found that
corticosteroid resulted in worse clinical outcome and recurrence rate compared to placebo injection. Use of physical therapy did not produce any significant differences.
Dines et al. review the diagnosis and of tennis injuries, including lateral epicondylitis. Although they acknowledge that there is "no long term benefit with regard to tendon healing," they note that steroid injection may be useful to control acute symptoms.
Boyer et al. review the myths surrounding lateral epicondylitis. They note that despite widespread use, corticosteroid injection has repeatedly been shown to have no long term benefit. They conclude based on their review of the literature that if there is any benefit to steroid injection, it is of short duration, and in patients whose symptoms have been of short duration, without any previous treatment.
Incorrect answers:
Answers 1, 3, 4, 5. All of these treatments have been used for lateral epicondylitis but more conservative measures should be tried first.
(SAE08OS.118) Which of the following is helpful on physical examination to diagnose a fixed posterior shoulder dislocation? Review Topic
-
Apprehension sign
-
Jobe relocation test
-
Sulcus sign
-
Jerk test
-
Lack of external rotation
PREFERRED RESPONSE 5
The apprehension sign and Jobe relocation test are helpful for the diagnosis of anterior shoulder instability. The sulcus sign provides information on the status of the rotator interval. The jerk test is helpful for the diagnosis of posterior instability, but a fixed posterior shoulder dislocation is associated with loss of external rotation. Since an AP radiograph may miss this diagnosis, an axillary view should be obtained on patients with a shoulder injury.
(SAE10SM.20) A 23-year-old right-hand dominant professional baseball pitcher has right shoulder pain when releasing the ball. He has noticed his velocity has decreased over the past 2 months. Examination reveals supine abducted external rotation of 110 degrees compared to 100 degrees on the left side. His internal rotation is 30 degrees on the right compared to 70 degrees on the left side. Rotator cuff strength is normal. All other clinical tests are normal. MRI with contrast reveals no intra-articular lesions. What is the best course of treatment? Review Topic
-
Arthroscopic capsular plication
-
Arthroscopic thermal shift
-
Arthroscopic subacromial decompression
-
Posterior capsular stretching
-
Selective external rotation stretching
PREFERRED RESPONSE 4
The examination reveals that the patient has posterior capsular tightness. Surgery should not be considered until the patient has failed to respond to nonsurgical management. The internal rotation contracture (GIRD - glenohumeral internal rotation deficit) should be addressed with appropriate posterior capsular stretching. This should then be followed by appropriate rotator cuff and scapular stabilization exercises. Only if this management fails to relieve the patient's symptoms should surgery be considered. This patient clearly does not need external rotation stretching given the fact that he has normal external rotation.
(SAE11UE.73) Which of the following is considered a contraindication to elbow arthroscopy? Review Topic
-
Osteonecrosis of the elbow (Panner disease)
-
Loose body in the ulnohumeral joint
-
Status post open reduction and internal fixation of a radial head fracture
-
Ulnar neuropathy with prior submuscular ulnar nerve transposition
-
Elbow stiffness
PREFERRED RESPONSE 4
Neurovascular complications are the most common complications reported with elbow arthroscopy. Any distortion in the anatomy of the elbow, especially when it involves neurovascular structures, such as a prior ulnar nerve transposition, increases the risk of neurovascular injury and is generally considered a contraindication to elbow arthroscopy. The other answers listed are either indications for arthroscopy or are not contraindications for the procedure.
(SAE11OS.165) A 43-year-old man who works as a plumber has a painful stiff elbow in his dominant arm. He notes that while he recalls no single event of injury, he thinks the many years of pulling wrenches and soldering pipes have resulted in his problem. He reports that he has pain with any motion in bending his arm and can no longer straighten his elbow. Examination reveals generalized swelling of the elbow, both medial and lateral with a range of motion that lacks 45 degrees of extension and flexes only to 110 degrees. Pronation and supination are also limited to 45 degrees. Audible crepitus is perceived but there is no instability. Radiographs reveal advanced osteoarthritis at the radiocapitellar and ulnohumeral joints with complete loss of articular cartilage. What is the most appropriate initial treatment option? Review Topic
-
Elbow fusion
-
Radial head resection
-
Total elbow arthroplasty
-
Osteophyte resection and capsular release
-
Physical therapy with dynamic extension and flexion splints
PREFERRED RESPONSE 4
Osteoarthritis of the elbow is more common in the middle-age laborer such as this plumber, whereas rheumatoid arthritis is more common in older females. Treatment
must respect the physical demands of the patient while trying to preserve joint motion and function with tolerable symptoms. Osteophyte resection and capsular release have offered many patients significant improvement in their symptoms while allowing them to return to most activities. The osteophyte resection and releases can be done effectively by an open or arthroscopic approach. Whereas total elbow arthroplasty would likely result in better and more thorough pain relief, it would not tolerate the occupational demands of this individual. There is no role for physical therapy initially in the face of advanced, painful arthritis associated with long-standing fixed joint contractures. Elbow fusion results in severe loss of function and its indication is rare and usually considered in the face of unmanageable sepsis. Radial head resection may improve symptoms related to the radial capitellar arthritis but would not improve range of motion or end range impingement pain. Also, radial head resection should be avoided in heavy laborers with elbow arthritis because it would lead to increased loads across the arthritic ulnohumeral joint.
(SAE11OS.76) Figures 76a and 76b are the sagittal T1-weighted MRI scans of an active 27-year-old man who has had left dominant extremity shoulder pain and weakness for the past 5 months. He denies any history of a precipitating event but recalls that the pain began around the time he started lifting weights after a year off from lifting. Examination reveals full range of active and passive motion, negative Hawkins and Neer impingement signs, 5/5 abduction strength, 5/5 external rotation strength with arm adducted at his side, and a negative belly press, Gerber lift-off, and O'Brien's test. He does have weakness with resisted external rotation with the arm abducted to 90 degrees. Radiographs are unremarkable. An MRI arthrogram shows no rotator cuff tear or labral tears. What is the most likely diagnosis? Review Topic
-
Scapular dyskenisia
-
Quadrilateral space syndrome
-
Subacromial impingement syndrome
-
Suprascapular nerve compression by a spinoglenoid notch
-
Suprascapular nerve compression at the suprascapular notch
PREFERRED RESPONSE 2
Examination reveals weakness of the teres minor muscle, and the MRI scan shows moderate isolated atrophy of the teres minor muscle belly. This is consistent with quadrilateral space syndrome, which is compression of the axillary nerve and posterior circumflex humeral artery in the quadrilateral space (bounded by the teres minor, teres major, long head of triceps and the humerus). This syndrome has been related to compression of the neurovascular structures by muscle hypertrophy consistent with the patient's history of lifting weights near the onset of symptoms. The next step in confirming the diagnosis is a subclavian arteriogram with the arm in adduction as well as in abduction and external rotation. Suprascapular nerve compression would be manifested by atrophy and weakness of both the supraspinatus and infraspinatus (if occurring at the suprascapular notch) or just infraspinatus (if occurring at the spinoglenoid notch). The patient does not demonstrate signs or symptoms of either impingement syndrome or scapular dyskenisia.
(SAE11OS.57) Figures 57a and 57b are the MRI scans of a 61-year-old man who is unable to elevate his dominant arm following a golf injury 24 hours ago. He has moderate pain during attempted arm elevation. Examination reveals significant spinati atrophy and he is only able to elevate his arm fully overhead while supine. The neurologic examination is normal. What is the next most appropriate step in management? Review Topic
-
Lidocaine injection test
-
Supraspinatus strengthening
-
Reverse shoulder arthroplasty
-
Conventional total shoulder arthroplasty
-
Arthroscopic rotator cuff repair/subacromial decompression
PREFERRED RESPONSE 1
The patient unknowingly has a chronic massive rotator cuff tear. Because of excellent compensation, he remained functional and was without symptoms. This is evidenced by the significant muscle atrophy. Following even trivial injury, the compensation process of arm elevation fails and the patient suddenly loses the ability to elevate the arm. At this time in management, it is critical to recognize that the rotator cuff had already been torn and that pain now prevents the patient from actively using the arm. To better ascertain a prognosis of return of function, injecting a local anesthetic (lidocaine) into the joint is important. If, with an anesthetized joint, the patient can now elevate the arm, a supine strengthening program will likely return the patient to his pre-injury state. If there is no improvement in the ability to elevate the arm after the injection, surgical considerations may become relevant. There is no role for arthroscopic repair in this chronic, massive rotator cuff tear and decompression would likely lead to superior escape. A reverse shoulder arthroplasty would be contraindicated in a very active 61-year-old patient who 2 days ago was functioning normally. Based on the MRI scan, there is no supraspinatus muscle remaining to strengthen. Total shoulder arthroplasty is contraindicated in patients with a deficient rotator cuff mechanism.
(OBQ05.51) A 20-year-old girl reports a shoulder dislocation while reaching for a high shelf. Her history reveals multiple past dislocations with spontaneous reduction and no obvious traumatic event at onset. A photograph of her hand is shown in figure
A. What is the most likely etiology of her shoulder instability? Review Topic
-
inverted pear glenoid with bone deficiency
-
long thoracic nerve palsy
-
Bankart lesion
-
capsular redundancy
-
Buford complex
PREFERRED RESPONSE 4
The patient has multidirectional instability (MDI). A redundant capsular pouch is a consistent and reproducible finding in shoulders with MDI.
The first line of treatment for shoulder dislocations in patients with MDI should consist of physical therapy aimed at strengthening the rotator cuff and scapular stabilizers. For those who fail to respond to 3 to 6 months of nonsurgical treatment, surgical intervention can be considered with inferior capsular shift being the procedure of choice for multidirectional instability. Good results have been achieved with surgical procedures for posterior and multidirectional instability, but results have been less predictable than those achieved with procedures for traumatic anterior instability.
Ide et al. evaluated the results of an 8-week rehabilitation program with shoulder-strengthening exercises and a novel scapular-stabilizing shoulder orthosis in 46 patients with MDI. There was a significant increase in mean total scores on the modified Rowe grading system and mean torque of internal and external rotation with a normalization of mean external/internal torque ratios at the completion of the program. The authors concluded that shoulder strengthening exercises represent a
useful treatment option for patients with MDI.
Levine et al. reviewed treatments of multidirectional shoulder instability in athletes. Nonoperative management remains the initial treatment of choice. Open capsular shifts remain the operative treatment of choice, however, arthroscopic electrothermal capsulorrhaphy has become increasingly used as an alternative to an open approach.
Beasley et al. reviewed multidirectional instability in the shoulder of female athletes. The authors note that women tend to have greater ligamentous laxity than men and female athletes have a greater risk of converting laxity to symptomatic instability.
Figure A demonstrates metacarpophalangeal hyperextension, which is a sign of generalized ligamentous laxity.
Incorrect Answers:
Answer 1 and 3 - Bankart lesions and glenoid bone deficiency would be more indicative of acute traumatic instability. Answer 2 - Long thoracic nerve palsy would result in serratus anterior weakness and medial scapular winging. Answer 5 - Buford complex is a normal anatomic variant in which there is complete absence of the anterosuperior labrum and a cordlike MGHL with attachment to the base of the biceps anchor.
(SAE08UE.83) A 21-year-old right hand-dominant male collegiate swimmer reports painful clicking in the right shoulder. He states that he can occasionally feel his shoulder “slip out” when he is working out. AP, true AP, and axillary radiographs are shown in Figures 39a through 39c. What is the next most appropriate step in management? Review Topic
-
Echocardiography
-
Abdominal ultrasound
-
Skeletal survey
-
Glenoid osteotomy
-
Physical therapy
PREFERRED RESPONSE 5
The radiographs show glenoid hypoplasia. The common radiographic findings of glenoid hypoplasia include an inferior and posterior glenoid deficiency, enlargement of the distal end of the clavicle, and sometimes an indentation in the glenoid. It is usually bilateral and rarely associated with other syndromes; therefore, an echocardiogram, abdominal ultrasound, or a skeletal survey is unnecessary unless the patient has stigmata of a syndrome such as Holt-Oram or Apert’s. Although posterior instability has been reported, the results of glenoid osteotomy have been variable and should not be considered initially. Physical therapy is the mainstay of initial management, but the patient should be counseled that this may be a recurrent problem with early osteoarthritis developing in many patients. Radiographs of the contralateral side should be obtained because this is usually bilateral.
(SAE07SM.81) Medial dislocation of the long head of the biceps tendon in the shoulder is most commonly caused by a Review Topic
-
tear of the subscapularis tendon.
-
tear of the supraspinatus tendon.
-
tear of the transverse ligament.
-
type I SLAP tear.
-
congenitally shallow bicipital groove.
PREFERRED RESPONSE 1
Medial dislocation of the biceps tendon in the shoulder is commonly associated with subscapularis tendon tears. Although type II SLAP tears can result in bicipital instability, type I SLAP lesions do not. Congenitally shallow grooves and tears of the transverse ligaments usually do not lead to dislocation of the biceps tendon. Supraspinatus tendon tears are associated with long head of the biceps tendon ruptures but do not cause biceps tendon dislocations.
(SAE07SM.12) Which of the following is considered an advantage of arthroscopic distal clavicle excision compared with open distal clavicle excision? Review Topic
-
Fewer complications
-
Lower infection rate
-
Evaluation of the glenohumeral joint
-
Preservation of the inferior acromioclavicular ligament
-
Decreased surgical time
PREFERRED RESPONSE 3
Arthroscopic versus open distal clavicle excision has the advantage of allowing evaluation of the glenohumeral joint arthroscopically prior to moving into the subclavicular and subacromial space to perform the distal clavicle excision. This can be of value in both confirming the diagnosis as well as avoiding diagnostic errors. Berg and Ciullo showed that in 20 patients who underwent open distal clavicle excision that resulted in failure, 15 of those patients had a superior labral anterior posterior (SLAP) lesion. Of these 15 patients who had the lesion treated surgically, 9 went on to a good to excellent result after the surgery was performed arthroscopically. Fewer complications, lower infection rate, and decreased surgical time have not been documented in the literature. Arthroscopic technique sacrifices the inferior acromioclavicular ligament and preserves the superior acromioclavicular ligament.
(SAE11UE.39) A 22-year-old javelin thrower reports that he has had increasing discomfort in his right elbow and loss of distance from his throws for the past 3 months. Examination reveals tenderness over the medial elbow. Application of valgus torque to the elbow through a passive range of motion elicits pain from 70 degrees to 120 degrees of flexion, with no pain at the limits of extension. What structure is primarily responsible for the patient's symptoms? Review Topic
-
Anterior bundle of the medial collateral ligament (MCL)
-
Posterior bundle of the MCL
-
Annular ligament
-
Triceps insertion
-
Olecranon osteophytes
PREFERRED RESPONSE 1
The MCL is divided into anterior and posterior bundles; the anterior bundle is subdivided into anterior and posterior bands. Sectioning studies showed that the anterior band of the anterior bundle is the primary restraint to valgus stress at 30 degrees, 60 degrees, and 90 degrees; the posterior band of the anterior bundle is the primary restraint at 120 degrees. Medial elbow pathology in a throwing athlete can present with pain, instability, loss of velocity or control, or with ulnar nerve symptoms. Differentiating between different causes of disability can be largely accomplished through physical examination. The moving valgus stress test is performed by applying a valgus stress to a maximally flexed elbow, then passively extending the elbow. Reproduction of the patient's symptoms in the mid arc of flexion suggests MCL insufficiency. Pain at the end point of extension suggests posterior compartment symptoms, which were not present in this patient. The posterior bundle
is a secondary stabilizer at 30 degrees of flexion, and not susceptible to valgus load when the anterior bundle is intact. The annular ligament and triceps insertion are not involved with medial instability of the elbow. Olecranon osteophytes likely cause pain in terminal extension of the elbow.
(SAE08OS.104) A 38-year-old woman has persistent elbow pain but is unable to recall a specific traumatic event. Examination reveals that the patient exhibits apprehension when the elbow is placed in valgus with forearm supination and axial loading. Because of chronicity and failure to respond to nonsurgical management, what is the most appropriate treatment? Review Topic
-
Continued bracing
-
Direct ligamentous repair
-
Arthroscopic electrothermal capsular and ligamentous shrinkage
-
Isolated plication of the capsule and ligaments
-
Free tendon graft reconstruction with capsuloligamentous plication
PREFERRED RESPONSE 5
The maneuver described is the lateral pivot-shift test, where valgus and axial loads are applied to the extended and supinated forearm while the elbow is gradually flexed. The presence of apprehension in an awake patient suggests posterolateral rotatory instability, indicating insufficiency of the lateral ulnar collateral ligament. Treatment for chronic cases involves reconstruction using a palmaris longus tendon graft combined with plication of the lateral capsuloligamentous structures. Direct ligament repair and isolated plication are less reliable. The long-term effects of thermal shrinkage are still unclear. Because of the failure to respond to nonsurgical management, continued bracing is unlikely to resolve the patient's symptoms.
(SAE13SM.6) A 26-year-old weightlifter had increasing pain in his left shoulder for 4 months. Nonsurgical treatment consisting of anti-inflammatory medication,
corticosteroid injections, and rest failed to alleviate his symptoms. He underwent an arthroscopic distal clavicle resection with excision of the distal 8 mm of clavicle (Mumford procedure). Three months after surgery, he reported popping by his clavicle and mild pain. His clavicle demonstrated mild posterior instability on examination without any obvious deformity on his radiographs. What structures were compromised during his excision? Review Topic
-
Anterior and superior acromioclavicular joint ligaments
-
Posterior and superior acromioclavicular joint ligaments
-
Conoid ligament
-
Trapezoid ligament
PREFERRED RESPONSE 2
The posterior and superior acromioclavicular ligaments provide the most restraint to posterior translation of the acromioclavicular joint and must be preserved during a Mumford procedure. Anterior and superior acromioclavicular joint ligaments are the opposite of the preferred response and prevent anterior translation of the clavicle. Injuries to the conoid and trapezoid ligaments are more pronounced with grade III or higher acromioclavicular separations, with superior migration of the clavicle relative to the acromion.
(SAE11UE.124) Which of the following statements regarding the use of thermal shrinkage during arthroscopic shoulder surgery is most accurate? Review Topic
-
The amount of shrinkage is fixed for a given peak temperature, irrespective of the time of application.
-
Denatured capsular tissue does not undergo a healing response.
-
The capsule is typically found to be thick and fibrotic in revision cases following thermal shrinkage.
-
Patients with good results at 1 year are unlikely to develop recurrent instability in the future.
-
High failure rates have been reported in its use for anterior, posterior, and multidirectional instability.
PREFERRED RESPONSE 5
Reports of clinical results at 2- and 5-year follow-up indicate much higher failure rates than traditional stabilization techniques for all common instability patterns. The degree of capsular shrinkage is dependent on the total amount of thermal energy delivered, as well as the rate of delivery. Denatured tissue undergoes a healing response. The capsule typically encountered in revision cases is thin and patulous, rather than thick and fibrotic.
(SAE10SM.27) The clinical photograph in Figure 27 shows a palsy of what nerve/associated muscle? Review Topic
-
Long thoracic/rhomboid
-
Long thoracic/serratus anterior
-
Long thoracic/supraspinatus
-
Dorsal scapular/trapezius
-
Spinal accessory/trapezius
PREFERRED RESPONSE 2
The clinical picture reveals medial scapular winging, which involves the serratus anterior muscle, potentially due to an injury to the long thoracic nerve that innervates this muscle. Injury to the long thoracic nerve is usually due to closed trauma, direct compression, traction or stretching injury, a direct blow, or, very rarely, viral infection
such as Parsonage-Turner syndrome. The nerve is easily injured in surgical dissection of the axilla, and is predisposed to injury due to its relatively long course, it is small in diameter, and it has little surrounding connective tissue. If rehabilitation and time are unsuccessful, both nerve and muscle transfers have been described with mixed results.
(SAE11OS.30) During arthroscopic evaluation of a partial-thickness articular-sided supraspinatus tendon tear, the medial-lateral width of the tear is noted to be 6 mm. This represents what percent partial-thickness tear? Review Topic
1 10%
2 25%
3 50%
4 75%
5 90%
PREFERRED RESPONSE 3
Partial-thickness rotator cuff tears can be bursal-sided, articular-sided, and/or intratendinous. Management of partial-thickness tears requires an understanding of the native anatomy. Dugas and associates and Ruotolo and associates studied cadaveric specimens and reported the medial-lateral width of the supraspinatus tendon averages 12.1 to 12.7 mm. Therefore, a 6- to 7-mm tear represents approximately a 50% tear of the supraspinatus tendon. Most authors agree that tears representing greater than 50% of the medial-lateral width of the supraspinatus tendon should be repaired.
(SAE08UE.103) A 67-year-old woman is seen in the emergency department after falling at home. Radiographs before and after treatment are shown in Figures 49a and
49b, respectively. Which of the following best explains the 8-week postinjury clinical findings seen in Figure 49c? Review Topic
-
Axillary nerve palsy
-
Spinal accessory nerve palsy
-
Deltoid avulsion
-
Rotator cuff tear
-
Unreduced posterior glenohumeral dislocation
PREFERRED RESPONSE 4
Patients older than age 40 years at the time of initial anterior dislocation have low rates of redislocation; however, 15% of these patients experience a rotator cuff tear. Moreover, there is a dramatic increase (up to 40%) in the incidence of rotator cuff tears in patients older than age 60 years. Axillary nerve injury may occur but is less common than rotator cuff tear.
(SAE08OS.126) Which of the following preoperative findings is a predictor of poor outcome following arthroscopic debridement for glenohumeral arthritis? Review Topic
-
Shoulder stiffness
-
History of infection
-
Nonconcentric joint space on preoperative radiographs
-
Intra-articular loose bodies
-
History of instability
PREFERRED RESPONSE 3
Studies have shown that patients with moderate to severe degenerative changes of the glenohumeral joint fare worse than those with a concentric joint space. Preoperative stiffness and presence of loose bodies have correlated with successful results. A history of infection or instability, in the absence of degenerative changes has not been correlated with poorer outcomes.
(SAE10SM.22) What structure is the primary restraint to inferior translation of the shoulder? Review Topic
-
Middle glenohumeral ligament
-
Subscapularis
-
Long head of the biceps
-
Coracohumeral ligament
-
Coracoacromial ligament
PREFERRED RESPONSE 4
The coracohumeral ligament has been shown to be the primary restraint to inferior translation of the shoulder. Although Bigliani and associates have demonstrated that the inferior capsule and inferior glenohumeral ligaments also play a role, none of the other choices provide primary inferior stability of the shoulder. The coracohumeral ligament is an important structure of the rotator interval of the shoulder (the rotator interval contains the long head of the biceps, the superior glenohumeral ligament, the coracohumeral ligament, and a thin layer of capsule). Harryman and associates demonstrated that an open rotator interval closure via imbrication of the coracohumeral ligament improves inferior stability of the glenohumeral joint.
(SAE10SM.52) When performing elbow arthroscopy, it is often necessary to evaluate the posterior compartment. When entering the posterior compartment of the elbow, what are the two safest and most commonly used portals? Review Topic
-
The posterior portal created 3 cm proximal to the tip of the olecranon and the posterior medial portal created 3 cm from the tip of the olecranon and medial to the triceps
-
The posterior portal created 3 cm proximal to the tip of the olecranon and the posterior lateral portal created 3 cm proximal from the tip of the olecranon and just lateral to the triceps
-
The posterior medial portal created 3 cm from the tip of the olecranon and medial to the triceps and the posterior lateral portal created 3 cm from the tip of the olecranon and lateral to the triceps
-
The posterior medial portal created 3 cm from the tip of the olecranon and the lateral portal made through the anconeus
-
The posterior portal created at the tip of olecranon and the posterior medial portal just medial to the triceps
PREFERRED RESPONSE 2
The posterior portal created 3 cm proximal to the tip of the olecranon and the posterior lateral portal created 3 cm proximal from the tip of the olecranon and just lateral to the triceps are the "workhorse" portals of the posterior compartment and although relatively safe, risks exist. The radial nerve proximity averages 4.8 mm (3 to 8 mm) from the posterolateral portal. The central posterior portal is close to 20 mm from the ulnar nerve.
(SAE08UE.43) A 26-year-old right hand-dominant man has had right shoulder pain for the past 6 months. History reveals that he was the starting pitcher for his high school team. Activity modification, physical therapy, cortisone injection, and anti-inflammatory drugs have failed to improve his symptoms. He has a positive O’Brien’s active compression test. What is the next most appropriate step in the diagnosis of this patient? Review Topic
-
Diagnostic arthroscopy
-
MRI-arthrography
-
Stress radiographs
-
CT
-
Weighted radiographs of the arm
PREFERRED RESPONSE 2
MRI-arthrography has been shown to be an accurate technique for assessing the glenoid labrum in patients with suspected labral tears. Often standard MRI technique will not identify labral lesions. The use of MRI-arthrography with an intra-articular injection of gadolinium provides improved visualization of labral lesions. Bencardino and associates demonstrated a sensitivity of 89%, a specificity of 91%, and an accuracy of 90% in detecting labral lesions. SLAP lesions can be visualized on coronal oblique sequences as a deep cleft between the superior labrum and the glenoid that extends well around and below the biceps anchor. Often, contrast will diffuse into the labral fragment, causing it to appear ragged or indistinct.
(OBQ15.167) A 23-year-old male college quarterback presents with acute left shoulder pain after being tackled. A radiograph of the injury is shown in figure A. After successful closed reduction, what shoulder position should be avoided in order to minimize the risk of a repeat injury? Review Topic
-
Abduction to 90 degrees with maximal external rotation
-
Forward elevation to 170 degrees with neutral rotation
-
Cross-body adduction at 90 degrees flexion, with internal rotation
-
Extension with maximal internal rotation
-
Flexion to 90 degrees with maximal external rotation
PREFERRED RESPONSE 3
The patient presents with a traumatic posterior shoulder dislocation and radiographic evidence of a reverse Hill-Sachs type injury. The patient should avoid adduction, 90 degrees flexion, and internal rotation in order to decrease the risk of re-dislocation.
Shoulder stability is achieved through the both dynamic and static stabilizers. The static stabilizers include the bony morphology of the joint, glenoid labrum, capsule, and glenohumeral ligaments. The contributions of the glenohumeral ligaments to shoulder stability are dependent upon the position of the humerus relative to the glenoid. Posterior stability is afforded to the joint by the superior glenohumeral ligament (SGHL) and the posterior band of the inferior glenohumeral ligament (IGHL). The SGHL specifically is taught and provides posterior stability with the shoulder in flexion, adduction, and internal rotation.
Kim et. al. reviewed their experience treating 27 athletes diagnosed with traumatic posterior shoulder instability and treated with arthroscopic posterior labral repair and capsular shift. Most patients were found to have an incompletely stripped posterior capsulolabral complex. After arthroscopic repair and shift, all 26 of the 27 patients treated had improved shoulder function and objective scores, a stable shoulder, and were able to return to sport.
Millett et. al. reviewed posterior shoulder instability. They describe the static restraints of the posterior shoulder as the SGHL, posterior band of IGHL, and the coraohumeral ligament (CHL). The SGHL and CHL are both taught in the position of flexion, adduction, and internal rotation, whereas the posterior band of the IGHL is taught in abduction. They describe posterior instability occuring secondary to overhead sports due to repetitive microtrauma causing gradual capsular failure.
Figure A is an axillary radiograph of the left shoulder demonstrating a posterior dislocation and an engaging reverse Hill-Sachs lesion.
Incorrect Answers:
Answer 1, 2, 4, and 5: These options do not maximally tension the posterior shoulder stabilizers
(OBQ13.166) A 55-year-old male returns for followup 3 months after reverse shoulder arthroplasty. He reports limited function of his right shoulder but no antecedent trauma. A radiograph of his shoulder is shown in Figure A. All of the following variables are associated with this complication EXCEPT: Review Topic
-
History of malunited proximal humerus fracture
-
Proximal humeral bone loss
-
Failed primary arthroplasty
-
Rheumatoid arthritis
-
Fixed preoperative glenohumeral dislocation
PREFERRED RESPONSE 4
Rheumatoid arthritis is not associated with reverse shoulder arthroplasty (RSA) dislocation.
RSA dislocation is a known complication of RSA. Risks include proximal humeral bone loss, chronic fracture sequelae with malunited/ununited tuberosities, failed previous arthroplasty, and fixed glenohumeral dislocation preoperatively. An irreparable subscapularis tears may be less of an issue with newer implant designs.
Trappey et al. studied instability and infection rates after RSA. They found that the rate of instability was similar in primary and revision surgery, but the rate of infection was higher in revision surgery. Instability was highest in the fracture sequelae group because of malunited tuberosities, contractures and proximal humeral bone loss.
Favre et al. examined the effect of component positioning on RSA stability. They found that humeral version was more important than glenoid version. Stability is improved with the humerus in neutral or slight anterversion. They recommend avoiding retroversion >10deg.
Edwards et al. examined subscapularis insufficiency and the risk of RSA dislocation. They found that of 138 RSA, all 7 dislocations occurred in patients with an irreparable subscapularis. Dislocation was also more likely in patients with complex diagnoses, including proximal humeral nonunion, fixed dislocation, and failed prior arthroplasty.
Figure A shows reverse shoulder arthroplasty dislocation. Incorrect Answers:
Answers 1, 2, 3, 5: Proximal humerus fracture malunion, proximal humeral bone loss, failed primary surgery, and fixed dislocation are all risk factors for RSA dislocation.
(OBQ14.181) An active 68-year-old woman undergoes an uncomplicated rotator cuff repair with a double-row construct using biocomposite knotless anchors. At her two month follow up, she is noted to have increased shoulder pain, weakness and limited motion. Imaging reveals failure of the rotator cuff repair. What is the most likely mechanism of failure? Review Topic
-
Anchor fatigue and breakage
-
Anchor pull out from bone
-
Suture rupture secondary to anchor eyelet abrasion
-
Suture pull out from the repaired tissue
-
Infection
PREFERRED RESPONSE 4
Rotator cuff repair (RCR) failure most commonly occurs from a failure of the repaired tissue to heal with suture anchor pull out from the repaired tissue.
The overall complication rate of arthroscopic RCR is roughly 10%. Failed RCR most commonly results from failure to heal (19-94%) secondary to poor rotator cuff tissue, insufficient vascularity or poor bone quality. Other causes of RCR failure include surgical complications (deltoid disruption, infection, foreign body reaction, stiffness, neurologic injury), diagnostic errors (missed lesions of the rotator interval, long head of biceps or subscapularis tear), and technical errors (excessive tension due to lack of proper tissue mobilization, anchor pull out secondary to improper anchor placement).
George et al evaluated the causes of failed RCR and results of revision RCR. While results of revision RCR are inferior to primary RCR, arthroscopic repair yields > 60% good or excellent results. Risk factors for poor results following revision RCR include poor tissue quality, detachment of the deltoid origin and multiple previous surgeries.
Diduch et al reviewed the design and composition of various anchors used in arthroscopic shoulder surgery. Current advancements in the field include high-strength polyethylene sutures, new biocompatible anchor materials (PEEK, biocomposite) and modified designs including knotless systems. With improved strength of the current anchors and repair constructs, the most common mode of arthroscopic RCR failure is now related to tissue failure occurring at the tissue-anchor interface.
Cole et al discussed the different primary rotator cuff repair constructs, including single row, double row, transosseous and transosseous equivalent. The authors concluded that construct selection depends on tear acuity, size and tissue quality. For acute tears < 12mm in anteroposterior length, single-row configuration likely has sufficient strength to maintain the repair and promote healing. For more chronic tears, poor tissue quality, or tears > 12-15mm in the anteroposterior dimension, the authors recommend double-row or transosseous-equivalent repair to better restore the anatomic footprint and provide optimal mechanical stability to achieve healing.
Illustration A is an algorithm from George et al detailing the decision-making process when considering revision RCR for a symptomatic failed RCR.
Incorrect Responses
Answer 1: Anchor breakage is a less common cause of RCR failure. Biocomposite anchors exhibit high load-to-failure and result in fatigue failure less commonly than metal anchors.
Answer 2: Anchor pull out from bone is a less common cause of RCR failure and is associated with poor bone mineral density. Answer 3: Suture rupture secondary to abrasion against the anchor eyelet is a much less common cause of RCR failure and occurs more often with metal anchors, which have relatively sharper and rougher edges as compared to biocomposite anchors. Answer 5: Infection is an uncommon complication of RCR (~1.9%).
(OBQ15.149) An 80-year-old right-hand dominant male presents to clinic with 1 month of left shoulder pain. He has crepitance as well as a positive drop arm test on exam. Radiographs are significant for moderate glenohumeral arthritis and MRI demonstrates Goutallier Stage IV fatty infiltration of the rotator cuff. Which of the following is NOT an appropriate option for treatment of this condition? Review Topic
-
NSAIDs and/or cortisone injection
-
Arthroscopic rotator cuff repair
-
Shoulder hemiarthroplasty
-
Activity modification and/or physical therapy
-
Reverse total shoulder arthroplasty
PREFERRED RESPONSE 2
This patient has moderate glenohumeral arthritis with an irreparable rotator cuff tear. Rotator cuff tears with fatty infiltration are considered to be "irreparable", with arthroscopic repair not indicated as an appropriate option for treatment.
The optimal management of patients with irreparable rotator cuff tears with glenohumeral osteoarthritis is not well defined in literature. Initial management should involve conservative measures, including injection of corticosteroids, physical therapy, activity modification, and NSAIDs, with consideration of operative intervention in those that fail a trial of nonoperative management.
Laudicina et al review the management of irreparable rotator cuff tears in the setting of glenohumeral osteoarthritis(OA). NSAIDs, corticosteroid injection, activity modification, and physical therapy are mainstays of nonoperative treatment. Failure of conservative management may lead to operative intervention. The authors endorse that hemiarthroplasty is currently the procedure of choice for patients with moderate to severe glenohumeral OA and irreparable cuff tears.
Izquierdo et al provide a clinical practice guideline of the treatment of glenohumeral osteoarthritis based on systematic review. Nine of 16 addressed recommendations were inconclusive, illustrating that the management of glenohumeral osteoarthritis remains controversial. The single moderate-rated recommendation was for the use of total shoulder arthroplasty (TSA) rather than hemiarthroplasty. The two recommendations reached by consensus include use of perioperative mechanical and/or chemical DVT prophylaxis for shoulder arthroplasty patients and that TSA should be avoided in patients with glenohumeral OA with irreparable rotator cuff tear.
Illustration A demonstrates the Goutallier staging system of rotator cuff tears.
Incorrect Answers:
Answers 1, 3 and 4: NSAIDs, corticosteroid injection, physical therapy and activity modification are all part of the conservative initial management of glenohumeral arthritis with irreparable rotator cuff tears. Answer 5: Shoulder hemiarthroplasty and reverse total shoulder arthroplasty may be considered for patients who have failed a trial of nonoperative management.
(SAE08UE.11) A 45-year-old woman awakens with the acute onset of burning left shoulder pain that radiates toward the axilla. She denies any history of trauma. On examination, she is unable to abduct her arm but has full passive shoulder motion. Her sensation is intact. Cervical spine examination reveals full range of motion and a negative Spurling’s test. Radiographs and MRI studies are normal for the cervical spine and shoulder. What is the most likely diagnosis? Review Topic
-
Cervical C6-7 radiculopathy
-
Impingement
-
Rotator cuff tear
-
Brachial neuritis
-
Adhesive capsulitis
PREFERRED RESPONSE 4
The definition of brachial neuritis or Parsonage-Turner syndrome is a rare disorder of unknown etiology that causes pain or weakness of the shoulder and upper extremity. The loss of active motion excludes cervical C6-7 radiculopathy and impingement. A normal MRI scan and full passive motion exclude a rotator cuff tear and adhesive capsulitis, respectively.
(SAE08UE.45) A 59-year-old man underwent interposition arthroplasty for osteoarthritis of the elbow 9 years ago. Over the past year the patient has had
increasing pain and elbow instability. There is no clinical evidence of infection, and radiographs show no new bony process. What is the best option for this patient? Review Topic
-
Bracing
-
Physiotherapy
-
Cortisone injection
-
Conversion to total elbow arthroplasty
-
Revision interposition arthroplasty
PREFERRED RESPONSE 4
In a series reported by Blaine and associates, 12 patients were converted from interposition to total elbow arthroplasty. This procedure was successful in 10 out of 12 patients.
(SAE08UE.18) Figures 10a and 10b show the radiographs of a 47-year-old man who reports pain in both shoulders. He has a history of leukemia that was treated with chemotherapy and high-dose cortisone. What is the most reliable treatment option for pain relief in this patient? Review Topic
-
Arthroscopic debridement
-
Arthrodesis
-
Resection arthroplasty
-
Hemiarthroplasty
-
Cortisone injection
PREFERRED RESPONSE 4
The radiographs reveal osteonecrosis with collapse. The most reliable and durable treatment for osteonecrosis of the humeral head remains prosthetic shoulder arthroplasty. Osteonecrosis of the humeral head may be seen after the use of steroids, and there is an increasing demand for shoulder arthroplasty in young people because of the use of high-dose steroids in chemotherapy regimes for the treatment of malignant tumors. The indications for most shoulder arthrodeses today include posttraumatic brachial plexus injury, paralytic disorders in infancy, insufficiency of the deltoid muscle and rotator cuff, chronic infection, failed revision arthroplasty, severe refractory instability, and bone deficiency following resection of a tumor in the proximal aspect of the humerus. Clearly, the role of arthroscopy and related minimally invasive techniques in the treatment of humeral head osteonecrosis remains unknown.
(SAE11UE.21) A 71-year-old woman reports the insidious onset of shoulder pain at night and when moving her shoulder. She cannot raise her arm above shoulder level. Physical therapy has failed to provide pain relief or improve function. An injection relieved her pain in the office, but she could not raise her arm above shoulder level. A radiograph is shown in Figure 21. What surgical procedure will provide the best chance of restoring above shoulder function and pain relief? Review Topic
-
Reverse total shoulder arthroplasty
-
Hemiarthroplasty of the shoulder
-
Arthroscopic biceps tenolysis
-
Open subacromial debridement
-
Total shoulder arthroplasty
PREFERRED RESPONSE 1
The radiograph shows complete loss of the acromiohumeral space. The glenohumeral joint space is also severely narrowed, which is consistent with rotator cuff tear arthropathy. In patients who have pain that limits elevation, pain-reducing procedures such as biceps tenolysis, open debridement, or hemiarthroplasty may allow the patient to regain the shoulder function. If the patient cannot elevate the arm after a successful local anesthetic injection, then pain is not the reason for the patient's loss of elevation. In this situation, a reverse total shoulder arthroplasty will most reliably restore function and provide pain relief.
(SAE11OS.174) Figures 174a through 174c are the MRI scans of a 16-year-old football player who dislocated his dominant left shoulder 3 weeks ago while landing on his outstretched arm. The dislocation was reduced in the emergency department. He has since had two episodes where he felt like his shoulder slipped partially out of place. Which of the following statements to the athlete and his parents is most accurate regarding treatment options? Review Topic
-
Physical therapy should allow him to return to football with recurrent dislocations unlikely.
-
Immobilization of his shoulder in an external rotation brace will eliminate the chance of further dislocations.
-
Arthroscopic capsulolabral repair is a reasonable option if he wishes to undergo this procedure, despite this being a first-time dislocation.
-
Arthroscopic capsular and labral repair will likely fail in this situation.
-
Open repair definitely provides a better outcome.
PREFERRED RESPONSE 3
Arthroscopic capsulolabral repair is a reasonable option despite this being a first-time dislocation. The patient has had recurrent instability episodes with two subluxations since his dislocation. Outcome studies have shown up to 90% recurrent instability rates in young, active populations. Capsulolabral repair has the best chance to reduce the risk of recurrent instability. Physical therapy is unlikely to significantly reduce the high likelihood of recurrence. While there is controversy regarding immobilization in internal or external rotation, studies have shown that immobilization may not reduce the risk of recurrent instability. While older studies did show that open repairs had lower recurrence rates than arthroscopic repairs, more recent studies have shown similar rates for arthroscopic capsulolabral plication with modern suture anchor techniques and no glenoid bone loss or engaging Hill-Sachs lesion.
(SAE08UE.28) A 68-year-old woman with serologically proven rheumatoid arthritis underwent an open synovectomy and radial head resection 10 years ago. She now has severe pain that has failed to respond to nonsurgical management. Examination reveals a flexion arc of greater than 90 degrees. Radiographs are shown in Figures 15a and 15b. What is the most appropriate management? Review Topic
-
Semiconstrained total elbow arthroplasty
-
Unconstrained total elbow arthroplasty
-
Fascial arthroplasty
-
Open synovectomy
-
Arthroscopic synovectomy
PREFERRED RESPONSE 1
The radiographs reveal severe arthritic changes with no joint space, and the AP view shows a progressive malalignment secondary to the radial head resection. A prosthetic arthroplasty is indicated given the severe arthritis (Larsen grade III). Unconstrained arthroplasties have not performed as well as semiconstrained arthroplasties after previous radial head resections. However, both types of arthroplasties performed better in native elbows. Synovectomies should be reserved for less advanced disease states.
(SAE11UE.29) Which of the following is considered a contraindication to the use of a reverse total shoulder arthroplasty? Review Topic
-
Prior shoulder joint infection
-
Pseudoparalysis
-
Prior partial acromioplasty
-
Absent glenohumeral joint space narrowing
-
Axillary neuropathy
PREFERRED RESPONSE 5
The reverse total shoulder arthroplasty depends on a functional deltoid muscle which is innervated by the axillary nerve to restore elevation for the patient. Pseudoparalysis is an indication for a reverse shoulder arthroplasty. Acromioplasty has not been correlated with poor results with a reverse shoulder arthroplasty. As long as the patient does not have an active infection, prior infections are not a contraindication. Patients can still have pain and pseudoparalysis from a chronic rotator cuff tear, despite having normal cartilage, and they will still benefit from a reverse total shoulder arthroplasty if other treatments have failed.
(SAE11OS.62) Figure 62 shows the radiograph of a 46-year-old man who has had increasing shoulder pain and diminishing motion over the last 10 years. Because his difficulties are severely impacting his quality of life, he is seeking advice and treatment options. Twenty five years ago, he underwent a shoulder stabilization procedure for recurrent shoulder dislocations. Examination reveals he can only elevate his arm to less than shoulder level and his external rotation is no more than 10 degrees. Management consisting of nonsteroidal anti-inflammatory drugs and intra-articular steroid injections has failed to provide relief. What is the most appropriate treatment recommendation? Review Topic
-
Humeral head arthroplasty
-
Total shoulder arthroplasty
-
Reverse shoulder arthroplasty
-
Arthroscopic debridement/capsular release
-
Corticosteroid injection and physical therapy
PREFERRED RESPONSE 2
The patient has classic "arthritis of dislocation." Procedures done years ago were designed to enhance shoulder stability by limiting external rotation. However, it is now understood that limiting external rotation results in significant alteration of joint mechanics and kinematics, thus leading to the development of osteoarthritis. The average age of patients who develop `arthritis of dislocation` is 45 years old. Despite the young age of these patients, total shoulder arthroplasty offers the most predictable improvement in pain and function. However, the patient must be made aware of the need to protect the arm from excessive loads to protect the glenoid implant. Because there is complete loss of articular cartilage and incongruent joint surfaces, there is no role for arthroscopic debridement and capsular release. Injections offer little, if any, chance of improvement with the prior history of nonresponse. Physical therapy predictably makes patients worse because loading the arthritic joint generates more pain. Reverse shoulder arthroplasty is reserved for elderly patients with severe rotator cuff deficiency. A humeral head arthroplasty, while potentially more ideal than a total shoulder arthroplasty because of glenoid concerns, would likely not offer pain relief in the face of the significant glenoid involvement and incongruity.
(OBQ13.131) A 52-year-old man sustained the left elbow injury shown in Figure A while playing basketball 2.5 months ago. He underwent the procedure shown in Figure B. Post-operatively he was mobilized in a hinged brace. On examination today, his arc of elbow flexion is 75 degrees with loss of 45 degrees of full extension. His Disabilities of the Arm, Shoulder and Hand (DASH) Outcome Measure score is 45 points. What initial treatment option will likely provide the greatest improvement in this patients DASH score and functional range of motion? Review Topic
-
Self-directed exercise therapy
-
Supervised exercise therapy
-
Supervised exercise therapy with static progressive elbow splinting
-
Continuous passive motion device
-
Closed manipulation under anesthesia
PREFERRED RESPONSE 3
The clinical presentation is consistent with post-traumatic elbow stiffness following an elbow fracture-dislocation. Supervised exercise therapy with static elbow splinting over a 6 month period has shown to have a significant improvement on DASH scores and functional range of motion (ROM) in patients with post-traumatic elbow stiffness.
Post-traumatic elbow stiffness is often difficult to manage. The ultimate goal of treatment is to restore a functional range of elbow motion (30° to 130°). Nonoperative modalities are considered the first-line of treatment. Aggressive physical therapy has traditionally been advocated. However, the use of static progressive elbow splinting with a turnbuckle, alongside aggressive physical therapy, has shown to provide better functional outcomes. Treatment is usually maintained over a period of 6-12 months. Surgery is considered when nonoperative therapy fails.
Doornberg et al. looked at a retrospective case series of 29 patients with posttraumatic elbow stiffness. They showed that static progressive splinting can help gain additional motion when standard exercises fail to produce additional improvements.
Lindenhovius et al. randomized sixty-six patients with post-traumatic elbow stiffness into static progressive elbow splint therapy or dynamic elbow splinting over a 12 month period. There was no significant difference in outcomes between treatment modalities. ROM increased by 40° vs. 39° at six months, respectively. DASH scores improved from 50 vs 45 at enrollment to 32 vs. 25 at six months, respectively.
Figure A shows a posterior elbow dislocation with an associated medial epicondyle fracture. Figure B shows ORIF of the fracture seen in Figure A. Illustration A shows a static progressive turnbuckle elbow splint used for post-traumatic elbow stiffness.
Incorrect Answers:
Answer 1: Self directed exercise therapy of the elbow has shown to have poor results. Answer 2: Supervised exercise therapy is the first line of treatment for most patients with elbow stiffness. However, clinical response is variable and often unsatisfactory. Answer 3: Continuous passive motion machines have a limited role in treating established contractures. They do not seem to improve end-range mobility in these patients.
Answer 4: Closed manipulation under anesthesia may worsen elbow stiffness and cause intra-articular damage. Manipulation causes significant swelling and inflammation with tearing of soft tissues, causing hemarthrosis and additional fibrosis in the joint.
(SAE13SM.72) Figure 72 is the MRI scan of a 61-year-old man who had left shoulder pain with a massive rotator cuff tear. Active forward elevation was 120 degrees. Arthroscopic examination revealed that the rotator cuff tear was irreparable. The articular surfaces of the glenohumeral joint have a normal appearance without significant degenerative changes. What is the most appropriate treatment option? Review Topic
-
Biceps tenotomy
-
Loose body removal
-
Latissimus dorsi transfer
-
Reverse total shoulder arthroplasty
PREFERRED RESPONSE 1
The MRI scan shows medial subluxation of the biceps tendon. Biceps tenotomy has been an effective treatment option for patients with large to massive rotator cuff tears when the tear is irreparable and pain is the main symptom. The MRI scan does not show a loose body. Patients with severe external rotation deficit and a deficient teres minor may experience a better functional result with latissimus dorsi transfer. Reverse total shoulder arthroplasty is an option in patients with cuff tear arthropathy and pseudoparalysis.
(OBQ15.66) A 25-year-old athlete presents with symptoms attributed to injury to ligament D in Figure A. Which of the following symptoms and signs is characteristic of this injury? Review Topic
-
Pain during late cocking and acceleration; milking maneuver.
-
Painful clicking during pushoff from armrests of a chair; milking maneuver.
-
Painful clicking during pushoff from armrests of a chair; lateral pivot shift.
-
Pain during late cocking and acceleration; lateral pivot shift.
-
Painful clicking during pushoff from armrests of a chair; moving valgus stress test.
PREFERRED RESPONSE 3
This patient has rupture of the lateral ulnar collateral ligament (LUCL), producing posterolateral rotatory instability (PLRI). This is best demonstrated with a positive lateral pivot shift test.
PLRI can be diagnosed using the lateral pivot shift or posterolateral drawer. According to O’Driscoll, the elbow dislocates in 3 stages from lateral to medial (circle of Horii). Stage 1 involves disruption of the LUCL and partial/total disruption of the LCL complex (creating PLRI). Patients have pain with varus stress. Stage 2 includes disruption of the anterior capsule from incomplete elbow posterolateral dislocation. Stage 3 is divided into:
(a) Disruption of all soft tissues surrounding/ including the posterior MCL except for the anterior bundle. This bundle forms the pivot around which the elbow dislocates in a posterior direction by way of a posterolateral rotatory mechanism; and (b) complete disruption of the MCL.
O'Driscoll et al. describe PLRI diagnosed in 5 patients who had elbow dislocation using the posterolateral rotatory instability test, which they describe as being analogous to the test for lateral rotatory instability of the knee after ACL rupture. They believed the condition was laxity of the LUCL, which allowed transient rotatory subluxation of the ulnohumeral joint and secondary dislocation of the radiohumeral joint, without radio-ulnar joint dislocation. They recommended repair of the LUCL to eliminate PLRI.
Sanchez-Sotelo et al. retrospectively described 12 cases of direct repair and 33 ligamentous reconstructions for PLRI. 86% were satisfied with the procedure. Better results were obtained with patients with post-traumatic etiology, instability at presentation, and those with augmented reconstruction with tendon graft (compared with ligament repair alone).
Figure A shows structures on the lateral side of the elbow. The corresponding labels are seen in Illustration A. Illustration B shows the lateral pivot shift (also known as the posterolateral rotatory instability test).
Incorrect Answers:
Answers 1, 2, 4, 5: Pain during late cocking/early acceleration is characteristic of valgus instability. The milking maneuver and moving valgus stress tests are valgus tests used to test for this.
(SAE07SM.60) Nonsurgical management of pectoralis major tears is likely to result in weakness of glenohumeral Review Topic
-
abduction and external rotation.
-
abduction and internal rotation.
-
adduction and external rotation.
-
adduction and internal rotation.
-
external rotation and forward flexion.
PREFERRED RESPONSE 4
Nonsurgical management is considered for proximal tears as well as partial tears in some individuals. Surgical management is often not appropriate in older or sedentary patients. However, patients treated nonsurgically will have a significant cosmetic defect, as well as weakness in adduction and internal rotation.
(SAE11UE.10) Atraumatic suprascapular nerve compression usually occurs at which of the following anatomic locations if it develops atraumatically? Review Topic
-
Scalenus anterior
-
Suprascapular and spinoglenoid notches
-
Cervical rib
-
Conjoined tendon
-
Subcoracoid
PREFERRED RESPONSE 2
The suprascapular nerve has the potential to be compressed as it passes through the suprascapular and spinoglenoid notches. If the site of compression occurs at the suprascapular notch, both the supraspinatus and infraspinatus muscles will be affected. If the site of compression occurs at the spinoglenoid notch, only the infraspinatus muscle will be affected. Fascial bands and ganglion cysts often compress the nerve in these areas. The other anatomic areas are not associated with suprascapular nerve compression.
(SAE11UE.120) A 74-year-old patient is seen for follow-up 6 weeks after undergoing a total shoulder arthroplasty for glenohumeral osteoarthritis. The patient missed the 2-week follow-up appointment and is currently wearing a sling. The incision is well healed with no signs of breakdown. Examination reveals that passive range of motion is forward elevation of 90 degrees, external rotation at the side 0 degrees, and internal rotation up the back is to the level of the greater trochanter. A radiograph shows no signs of fracture or dislocation. What is the next most appropriate management for this patient? Review Topic
-
Physical therapy for range-of-motion exercises
-
Aspiration for possible infection
-
MRI to evaluate for possible rotator cuff tear
-
Sling immobilization and reevaluation in 4 weeks
-
Duplex ultrasound for possible upper extremity deep venous thrombosis
PREFERRED RESPONSE 1
The patient has a postoperative stiff shoulder. The patient missed follow-up appointments and has not been participating in physical therapy for stretching. Based on normal radiographic findings, the shoulder is not dislocated; therefore, physical therapy should begin immediately. Continued sling immobilization will further worsen the stiffness. There is no indication of an infection or rotator cuff tear. Deep venous thrombosis would present with abnormal swelling and pain.
(SAE08UE.81) Figure 38 shows the radiograph of a 75-year-old woman who has had right shoulder pain, difficulty sleeping on the affected arm, and difficulties performing activities of daily living for the past 6 weeks. Initial nonsurgical management includes analgesics, a subacromial cortisone injection, and gentle range-of-motion exercises. However, these modalities have failed to provide relief, and the patient reports that she is unable to elevate her arm. Her pain is worse and she would like the most reliable treatment method for pain relief and functional improvement. What is the best surgical treatment? Review Topic
-
Reverse shoulder arthroplasty
-
Hemiarthroplasty
-
Resurfacing of the humeral head
-
Arthroscopic debridement
-
Shoulder fusion
PREFERRED RESPONSE 1
The authors of several studies conducted in Europe have reported promising results in the short- and medium-term with use of a reversed or inverted shoulder implant. The most recent investigation, a multicenter study in Europe in which 77 patients (80 shoulders) with glenohumeral osteoarthritis and a massive rupture of the rotator cuff were treated with the Delta III prosthesis, described an improvement in the mean constant score of 42 points, an increase of 65 degrees in forward elevation, and minimal or no pain in 96% of the patients. Hemiarthroplasty, the “nonconstrained”
option, has long been the standard of care for rotator cuff tear arthropathy. However, careful examination of the literature reveals that the results have not been uniform.
(SAE08UE.7) Osteonecrosis of the humeral head is a rare complication seen after dislocation of the glenohumeral joint in skeletally immature patients. When this complication is encountered, treatment should consist of Review Topic
-
humeral head arthroplasty.
-
observation.
-
arthroscopic capsular release.
-
grafting of the humeral head defect.
-
electrical stimulation.
PREFERRED RESPONSE 2
This rare complication occurs after fracture-dislocation and has been seen after surgical stabilization in the adolescent. In most reported cases, prolonged observation has been shown to result in revascularization.
(SAE11UE.42) The MRI scan of a patient with symptomatic shoulder pain reveals subacromial bursitis. What markers have been shown to be significant contributors to this pain? Review Topic
-
Metalloproteases
-
Alpha fetoprotein
-
Prostate-specific antigen (PSA)
-
Carcinoembryonic antigen (CEA)
-
CA-125
PREFERRED RESPONSE 1
Several inflammatory markers have been shown to be elevated in subacromial bursitis. These include metalloproteases, tumor necrosis factors, and cyclooxygenase 1 and 2. The other answers provided are all tumor markers and not typically present in routine subacromial bursitis ("impingement syndrome").
(OBQ15.41) A 45-year-old right hand dominant female sustains a left shoulder injury after falling on ice 2 weeks ago. She is brought to the operating room for surgical intervention and positioned upright in a beach chair. Figures A and B are images taken from a posterior viewing portal with a 70 degree arthrosope. Figure C demonstrates the surgically repaired structure. Which of the following physical examination maneuvers would have been most likely abnormal in this patient PREOPERATIVELY? Review Topic
-
Jerk Test
-
Wright's Test
-
Lift-off test
-
Jobe's test
-
Hornblower's test
PREFERRED RESPONSE 3
The repaired structure is the subscapularis tendon, tested frequently with the Lift-off test.
Figure A as viewed from a posterior portal in the beach chair position of the left shoulder demonstrates a subscapularis tear with Figure B demonstrating medial instability of the long head of the biceps tendon. Figure C shows a repaired subscapularis to its footprint with double loaded screw-in anchors. A biceps tenodesis was concomitantly performed.
The key physical exam findings are positive Belly-press and Lift-off maneuvers, as well as weakness in internal rotation and increased passive external rotation. An MRI would confirm the diagnosis with discontinuity of the subscapularis and long-head of the biceps not located in the bicipital groove.
Faruqui et al present a Level 4 retrospective analysis of 52 patients. They found that the use of the belly press, lift-off, and bear hug tests had a collective sensitivity of 81%.
Yoon et al author a Level 2 prospective evaluation of 312 patients that underwent arthroscopic rotator cuff tear surgery. They investigated the sensitivity and specificity of the lift-off, internal rotation lag sign, belly-press, and bear-hug tests to evaluate subscapularis integrity. They found that a positive lift-off test was specific for the finding a full-thickness subscapularis tear. The lift-off, internal rotation lag sign, belly-press, and bear-hug tests show internal rotation strength deficit.
Figure A depicts an abnormal belly-press examination whereby the wrist flexes, and the elbow will fall posteriorly as the patient recruits the posterior deltoid to compensate for lack of the subscapularis. Illustration B demonstrates a normal belly-press examination. Illustration C shows a normal lift off test.
Incorrect Answers:
1: Jerk Test is positive if there is a 'clunk' or pain with the maneuver and is associated with posterior labral pathology. 2: Wright's test is where the shoulder is passively externally rotated and abducted while the patient turns their neck away from the tested extremity. It is a test for thoracic outlet syndrome. 4: Jobe's test is positive with supraspinatus weakness and or impingement. 5: Hornblower's test is positive if the arm falls into internal rotation. It may represent teres minor pathology.
(OBQ14.89) A 49-year-old male presents with right shoulder pain and weakness after undergoing open cervical lymph node biopsy approximately one year ago. A pertinent finding from the physical exam is seen in Figure A, with the patients arms by his side. Physical exam finding with the arms in a position of 90 degrees of forward elevation and 10 degrees of external rotation are shown in Figure B. What nerve is most likely injured? Review Topic
-
Long thoracic
-
Suprascapular
-
Spinal accessory
-
Axillary
-
Thoracodorsal
PREFERRED RESPONSE 3
The patient is presenting with LATERAL scapular winging which is a result of injury to the spinal accessory nerve and resultant trapezius muscle palsy.
The spinal accessory nerve is fundamental to scapulothoracic function and essential for scapulohumeral rhythm. This nerve is vulnerable along its superficial course. The majority of injuries to the spinal accessory nerve are iatrogenic and occur secondary to head and neck surgery. There is often a marked delay in recognition and initiating treatment. Surgical treatment with the Eden-Lange transfer lateralizes the levator scapulae and rhomboids (transfer from medial border to lateral border)
Camp et al. reviewed the results of 111 patients who underwent operative management of a lesion to the spinal accessory nerve. They found that the majority (~80%) of injuries were sustained iatrogenically and that diagnosis was delayed for approximately 12 months.
Pikkarainen et al. reviewed the natural history of isolated serratus palsy. They found that symptoms mostly recover in 2 years, but at least one-fourth of the patients will have long-lasting symptoms, especially pain.
Figure A depicts a patient with lateral scapular winging. Figure B demonstrates physical exam of this patient with their arms in a position of 90 degrees of forward elevation and 10 degrees of external rotation. Illustration A highlights the difference between medial and lateral scapular winging. Illustration B depicts another example of a patient with lateral scapular winging.
Incorrect Answers:
-
An injury to the long thoracic nerve would result in serratus anterior palsy which would lead to MEDIAL scapular winging.
-
An injury to the suprascapular nerve would result in weakness and wasting of the supraspinatus and/or infraspinatus.
-
An injury to the axillary nerve would result in deltoid muscle weakness.
-
An injury to the thoracodorsal nerve would result in latissimus dorsi weakness and would not cause scapular winging
(SAE08UE.85) Osteochondritis dissecans of the capitellum is a source of elbow pain and most commonly occurs in what patient population? Review Topic
-
Swimmers and divers
-
Football lineman
-
Rugby players
-
Gymnasts and throwing athletes
-
Cyclists
PREFERRED RESPONSE 4
The etiology of osteochondritis dissecans of the capitellum is somewhat unclear. However, trauma has been implicated in this disease process. Gymnasts who load their upper extremities during tumbling and throwing athletes with repetitive trauma during the throwing motion are common patient subgroups in which osteochondritis dissecans of the elbow is seen. This often occurs in the adolescent age population.
(SAE08UE.77) With the arm abducted 90 degrees and fully externally rotated, which of the following glenohumeral ligaments resists anterior translation of the humerus? Review Topic
-
Coracohumeral
-
Superior glenohumeral
-
Middle glenohumeral
-
Anterior band of the inferior glenohumeral ligament complex
-
Posterior band of the inferior glenohumeral ligament complex
PREFERRED RESPONSE 4
With the arm in the abducted, externally rotated position, the anterior band of the inferior glenohumeral ligament complex moves anteriorly, preventing anterior humeral head translation. Both the coracohumeral ligament and the superior glenohumeral ligament restrain the humeral head to inferior translation of the adducted arm, and to external rotation in the adducted position. The middle glenohumeral ligament is a primary stabilizer to anterior translation with the arm abducted to 45 degrees. The posterior band of the inferior glenohumeral ligament complex resists posterior translation of the humeral head when the arm is internally rotated.
(SAE08UE.22) What is the most common cause for poor outcomes in patients who undergo total shoulder arthroplasty? Review Topic
-
Loosening of the humeral component
-
Loosening of the glenoid component
-
Infection
-
Brachial plexus injury
-
Rotator cuff tear
PREFERRED RESPONSE 5
In an article in the Journal of Shoulder and Elbow, 431 total shoulder arthroplasties were performed with a cemented all-polyethylene glenoid component between 1990 and 2000. Follow-up averaged 4.2 years. In total, 53 surgical complications occurred in 53 patients (12%). Of these, 32 were major complications (7.4%), with 17 of these requiring reoperation. Index complications in order of frequency included rotator cuff tearing, postoperative glenohumeral instability, and periprosthetic humeral fracture. Notably, glenoid and humeral component loosening requiring reoperation occurred in only one shoulder. Data from the contemporary patient group suggest that there are fewer complications of shoulder arthroplasty and less need for reoperation. Especially striking is the near absence of component revision because of loosening or other mechanical factors. Complications involving the brachial plexus have been reported following total shoulder arthroplasty but are not as common of a cause for failure.
(SAE08UE.38) A 72-year-old woman with diabetes mellitus who underwent a total shoulder arthroplasty for degenerative arthritis 5 years ago now reports the sudden onset of shoulder pain following recent hospitalization for pneumonia. Laboratory values show a WBC count of 11,400/mm3 and an erythrocyte sedimentation rate of 52mm/h. What is the most appropriate action? Review Topic
-
Begin a stretching program.
-
Obtain shoulder radiographs and aspirate the shoulder joint.
-
Obtain an MRI scan to evaluate for a rotator cuff tear.
-
Schedule for irrigation and debridement.
-
Schedule for revision shoulder arthroplasty.
PREFERRED RESPONSE 2
The patient has the preliminary diagnosis of an infected shoulder arthroplasty; therefore, shoulder radiographs and joint aspiration for organism identification should be the first steps in the work-up. The patient is at risk for hematogenous spread given the recent history of pneumonia and her history of diabetes mellitus. Although she has stiffness, a stretching program is not indicated with the possibility of infection. Scheduling for revision arthroplasty, or irrigation and debridement will depend on multiple factors including identification of the infecting organism, the organism’s
susceptibility to antibiotics, and implant stability. An MRI scan to evaluate for a rotator cuff tear is not indicated at this time.
(OBQ13.41) Reverse total shoulder arthroplasty combined with latissimus dorsi transfer would be most appropriate for which of the following patients? Review Topic
-
75-year-old male with post-traumatic shoulder arthritis after a four-part proximal humerus fracture with no motor dysfunction
-
63-year-old male with grade 4 shoulder arthritis with severe deltoid muscle dysfunction secondary to a stroke
-
80-year-old female with significant rotator cuff arthropathy, a negative Hornblower sign and less than 5 degrees of external rotation lag
-
70-year-old female with pseudoparesis of anterior elevation and external rotation, narrowing of gleno-humeral joint and acetabularization of the acromion
-
82-year-old male with grade 4 shoulder arthritis and an isolated supraspinatus tear
PREFERRED RESPONSE 4
Reverse total shoulder arthroplasty combined with latissimus dorsi transfer would be most appropriate in a patient with pseudoparesis of anterior elevation and external rotation, in the setting of shoulder arthritis (narrowing of glenohumeral joint and acetabularization of the acromion).
Combining a latissimus dorsi tendon transfers with reverse total shoulder arthroplasty (R-TSA) helps to restore control of active external rotation. Dysfunction with external rotation can be determined clinically with external rotation lag sign, a positive Hornblower's sign, and radiographically with fatty degeneration of the teres minor classified as stage 2 or greater according to the system of Goutallier et al. or Fuchs et al.
Gerber et al. found that R-TSA with combined lat dorsi transfer yielded minimal improvements in external rotation ROM (13 deg to 19 deg) compared to increases in shoulder ROM in flexion (94 deg to 137 deg) and abduction (87 deg to 145 deg), with this procedure.
Boileau et al. examined 17 consecutive patients treated with reverse shoulder arthroplasty and latissimus dorsi and teres major transfer (L'Episcopo). They found
that external rotation increased from -21 degrees to 13 degrees (+34 degrees ). They recommend transferring both the LD and TM, rather than the LD alone as it results in better active external rotation.
Illustration A is a radiograph showing a right reverse total shoulder replacement. Illustration B shows a cadaveric image of the positioning of the latissimus dorsi tendon transfer prior to implantation of the reverse total shoulder components.
Incorrect Answers
Answer 1: Post-traumatic arthritis without motor dysfunction may be treated with total shoulder arthroplasty and there is no indication for tendon transfer. Answer 2: Severe dysfunction of the deltoid is typically considered a contraindication rather than an indication for R-TSA. Answer 3: This patient may benefit from a R-TSA. However, there is no clinical finding of external rotation dysfunction. A positive Hornblower's sign and external rotation lag >15 degrees to the contralateral shoulder would be indicative of external rotation dysfunction.
Answer 5: Isolated supraspinatus tears are not a surgical indication for reverse total shoulder arthroplasty.
(SAE10SM.57) A 57-year-old man who plays recreational sports reports pain in his dominant shoulder. An MR arthrogram is shown in Figure 57. During arthroscopy of the shoulder, what pathology is most likely to be found? Review Topic
-
Complete disruption of the transverse humeral ligament
-
Acromioclavicular joint arthritis
-
Absent coracohumeral ligament
-
Subscapularis tear and biceps subluxation
-
Complete rupture of the short head of the biceps
PREFERRED RESPONSE 4
The MR arthrogram shows medial subluxation of the biceps tendon out of the bicipital groove and a subscapularis tendon tear. Biceps tendon subluxation is almost always associated with subscapularis tears. Whereas other diagnoses can be associated, none of them is directly related to this finding or seen on the MR arthrogram.
(SAE07SM.99) Which of the following anatomic structures are in contact with internal impingement in the throwing athlete? Review Topic
-
Humerus and posterior-superior glenoid
-
Humerus and anterior inferior glenoid
-
Humerus and acromion
-
Biceps and acromion
-
Rotator cuff and acromion
PREFERRED RESPONSE 1
Internal impingement occurs in the late cocking phase of throwing with humeral head abduction and maximal external rotation. It is a physiologic phenomenon occurring in 85% of patients undergoing arthroscopy for various indications in one study. Internal impingement is defined as impingement of the posterior-superior rotator cuff between the humerus and posterior-superior glenoid rim. Symptomatic internal impingement is felt to be due to the frequency and magnitude of the impingement in throwers.
(SAE08UE.12) A 25-year-old woman returns for her first postoperative visit after arthroscopic thermal capsulorrhaphy for recurrent multidirectional instability. Examination reveals that the portals are healed, there is no swelling; and passive range of motion is within the normal range. However, she is unable to actively raise her arm. Shoulder radiographs are normal. What is the most likely cause of these findings? Review Topic
-
Adhesive capsulitis
-
Sling immobilization
-
Thermal chondrolysis
-
Subacromial impingement
-
Axillary nerve injury
PREFERRED RESPONSE 5
Treatment of shoulder instability with thermal devices has lead to numerous complications including recurrent instability, chondrolysis, stiffness, and capsular necrosis. This patient’s findings are consistent with a heat-induced axillary nerve injury. Normal radiographs exclude extensive chondrolysis.
(SAE08AN.44) Figure 28 shows an arthroscopic view of a right shoulder in the lateral position through a posterior portal. What is the area between structure B (biceps) and SS (subscapularis tendon)? Review Topic
-
Inferior glenohumeral ligament
-
Superior glenohumeral ligament
-
Rotator cuff interval
-
Subscapularis recess
-
Interior recess
PREFERRED RESPONSE 3
The rotator cuff interval is located between the supraspinatus and subscapularis and the biceps tendon is deep to the interval. It is a triangular area where the base is the coracoid process and the apex is the transverse humeral ligament at the biceps sulcus. Closure or tightening of this area is often helpful in patients with shoulder instability. Conversely, this area is often contracted in patients with adhesive capsulitis and may need to be released.
(SAE08OS.8) What is the most common neurologic problem associated with a simple shoulder dislocation? Review Topic
-
A neurapraxic brachial plexus injury
-
A neurapraxic axillary nerve injury
-
A neurapraxic musculocutaneous nerve injury
-
A neurotmetic axillary nerve injury
-
An axonotmetic musculocutaneous nerve injury
PREFERRED RESPONSE 2
The most common nerve injury associated with dislocation of the shoulder involves the axillary nerve. This is typically a stretch injury, or neurapraxia, that occurs with anterior displacement of the humeral head out of the glenoid. The suspected diagnosis can be confirmed with neurodiagnostic testing after the first 2 to 3 weeks. A gradual return to normal function is the expected result, though mild deficits may remain. A neurotmetic injury, in which there is complete disruption of the entire nerve, would show no return of function. This type of injury is more likely associated with a penetrating injury, a laceration secondary to a fracture fragment, or occasionally with a direct blow of sufficient force.
(SAE08AN.51) In the most common condition causing a winged scapula, which of the following nerves is affected? Review Topic
-
Long thoracic nerve
-
Spinal accessory nerve
-
Suprascapular nerve
-
Dorsal scapular nerve
-
Thoracodorsal nerve
PREFERRED RESPONSE 1
A winged scapula is most often associated with Parsonage-Turner syndrome, a condition thought to be due to an inflammatory or immune-mediated mechanism. Certain muscles are predisposed, particularly the serratus anterior muscle innervated by the long thoracic nerve. Other less common nerve lesions (eg, the spinal accessory and dorsal scapular nerves) may also cause winged scapulae.
(SAE08AN.71) Figure 41 shows the MRI scan of a 38-year-old weightlifter. What does the arrow on the MRI scan indicate? Review Topic
-
Biceps tear
-
Pectoralis minor tear
-
Pectoralis major tear
-
Subscapularis tear
-
Abscess formation
PREFERRED RESPONSE 3
Pectoralis major ruptures typically occur in avid weightlifters (often on supplements) and typically while bench-pressing. Clinically there is significant discoloration/bruising over the pectoralis and into the axilla. MRI helps confirm the diagnosis and may help determine if the tear is in the muscle belly or at the bone-tendon junction.
(SAE10SM.42) A 20-year-old college pitcher reports the recent onset of decreased velocity and posterior shoulder pain. He states that it takes him longer to loosen up but denies any mechanical symptoms. When compared to his non-throwing shoulder, glenohumeral examination of his throwing shoulder will most likely reveal which of the following findings? Review Topic
-
Coracoid tenderness
-
Supraspinatus muscle atrophy
-
Decreased internal rotation of greater than 25 degrees
-
Decreased external rotation of greater than 40 degrees
-
Decreased abduction of greater than 30 degrees
PREFERRED RESPONSE 3
In symptomatic throwing shoulders, loss of internal rotation in abduction resulting from posteroinferior capsular contraction exceeds adaptive gains in external rotation. Glenohumeral internal rotation deficit (GIRD) is defined as the loss in degrees of glenohumeral internal rotation of the throwing shoulder compared with the nonthrowing shoulder. The pathologic cascade initially begins with decreased velocity and command, followed by posterior stiffness and trouble loosening up. Posterior shoulder pain without mechanical symptom occurs during late cocking and early acceleration phases due to the contracture of the posterior-inferior capsule. This results in a posterosuperior shift of the glenohumeral contact, resulting in internal impingement on the undersurface of the posterior superior rotator cuff and strain on the posterior superior glenoid labral interface. The "slap event" is when the posterior superior labrum and biceps anchor fail in tension. After the "slap event", surgery is the likely solution. Prior to this event, however, posterior inferior capsular stretches may result in resolution of symptoms.
(SAE11UE.6) A patient has multidirectional instability of the shoulder that has not responded to nonsurgical management. Successful surgical treatment will most likely include which of the following? Review Topic
-
Abrasion arthroplasty of the anterior glenoid rim
-
Posterior capsular advancement into a Hill-Sachs defect
-
Repair of an inferior glenohumeral ligament detachment
-
Repair of an unstable SLAP tear
-
Closure of the rotator interval
PREFERRED RESPONSE 5
Published reports establish the importance of the rotator interval in shoulder stability and improvements achieved through suture closure of the interval. Multidirectional
instability treated surgically following failure to respond to nonsurgical management has been shown to be associated with classic Bankart lesions, Hill-Sachs defects, glenoid chondral lesions, and even SLAP lesions (Werner). However, these lesions were seen in a lower percentage than that found for unidirectional anterior dislocations. Likewise, these lesions do not appear to be significant in influencing treatment in the majority of patients.
(SAE11UE.27) Elbow distraction interposition arthroplasty may be most appropriate treatment for which of the following patient profiles? Review Topic
-
25-year-old woman with destructive juvenile rheumatoid arthritis
-
41-year-old male laborer with posttraumatic arthritis of the elbow
-
44-year-old woman with distal humerus osteonecrosis and collapse
-
65-year-old man with painful primary elbow osteoarthritis
-
70-year-old sedentary woman with end-stage rheumatoid arthritis
PREFERRED RESPONSE 2
Elbow interposition arthroplasty is reserved for younger, active patients who may otherwise be candidates for prosthetic replacement. Osteoarthritis, posttraumatic arthritis, and rheumatoid arthritis patients may all be candidates for interposition arthroplasty if bone stock is preserved and the elbow maintains inherent stability. Primary osteoarthritis may also be treated with ulnohumeral arthroplasty (ie, Outerbridge) or arthroscopic debridement with release. Patients with destructive juvenile rheumatoid arthritis and distal humerus osteonecrosis would better benefit from prosthetic replacement because of bone loss issues.
(SAE08UE.15) A 39-year-old man has had persistent right shoulder pain for the past 6 months. A formal physical therapy program has failed to provide relief, and an injection several months ago provided only short-term relief. Examination reveals a positive Neer and Hawkins test. There is no instability and the neurovascular
examination is normal. Arthroscopy reveals a partial rotator cuff tear on the bursal side measuring 60% of the tendon thickness. What is the next most appropriate step in management? Review Topic
-
Arthroscopic debridement alone of the partial rotator cuff tear
-
Repair of the partial rotator cuff tear and subacromial decompression
-
Arthroscopic debridement combined with subacromial decompression
-
Arthroscopic subacromial decompression
-
Biceps tenotomy
PREFERRED RESPONSE 2
Although arthroscopic debridement with or without subacromial decompression is a reasonable response, the patient has positive impingement signs. Several recent studies regarding the surgical treatment of partial rotator cuff tears have demonstrated good to excellent results after repair of tears involving more than 50% of the tendon thickness. This was shown specifically for bursal-sided tears and joint-side tears. Biceps tenotomy is not indicated in a young patient.